■戻る■ 全部 1- 101- 201- 301- 401- 501- 601- 701- 801- 901- 1001- 最新50  

面白い問題おしえて〜な 十四問目

1 名前:132人目の素数さん:2008/05/02(金) 21:53:23
面白い問題、教えてください

2 名前:132人目の素数さん:2008/05/02(金) 21:54:49
過去ログ
http://www3.tokai.or.jp/meta/gokudo-/omoshi-log/
まとめwiki
http://www6.atwiki.jp/omoshiro2ch/

1 http://cheese.2ch.net/test/read.cgi/math/970737952/
2 http://natto.2ch.net/test/read.cgi/math/1004839697/
3 http://science.2ch.net/test/read.cgi/math/1026218280/
4 http://science.2ch.net/test/read.cgi/math/1044116042/
5 http://science.2ch.net/test/read.cgi/math/1049561373/
6 http://science.2ch.net/test/read.cgi/math/1057551605/
7 http://science2.2ch.net/test/read.cgi/math/1064941085/
8 http://science3.2ch.net/test/read.cgi/math/1074751156/
9 http://science3.2ch.net/test/read.cgi/math/1093676103/
10 http://science4.2ch.net/test/read.cgi/math/1117474512/
11 http://science4.2ch.net/test/read.cgi/math/1134352879/
12 http://science6.2ch.net/test/read.cgi/math/1157580000/
13 http://science6.2ch.net/test/read.cgi/math/1183680000/

3 名前:132人目の素数さん:2008/05/02(金) 22:11:08
数学板の代表スレ乙

4 名前:132人目の素数さん:2008/05/03(土) 01:00:36
0.68 < log2 < 0.71を証明せよ

なんてのがメモ帳の隅にあったんだが、このスレで出た問題でしたっけ?

5 名前:132人目の素数さん:2008/05/03(土) 01:03:43
>>1


>>4
電卓で済む問題だなぁ……

6 名前:132人目の素数さん:2008/05/03(土) 01:27:35
>>4
ここ3,4年の東大過去問検索しる

7 名前:132人目の素数さん:2008/05/03(土) 06:56:52
自作問題。誰も(2)を解いてくれない(^o^)

(1) cを自然数とし、自然数列a(n)(n=1,2,3,…)を
a(1)=c , a(n+1)=[ a(n)|sin a(n)| ]+1 (n≧1)
で定義する。どんなcに対しても、a(n)は発散しないことを示せ。
ただし、[ ]はガウス記号とする。

(2) cを自然数とし、自然数列a(n)(n=1,2,3,…)を
a(1)=c , a(n+1)=[ a(n)|sin a(n)| ]+2008 (n≧1)
で定義する。どんなcに対しても、a(n)は発散しないことを示せ。
ただし、[ ]はガウス記号とする。

8 名前:132人目の素数さん:2008/05/04(日) 04:02:02
1でも2でも100でも2008でも正の数なら何でも発散しない事を示せばいいのか
ちょっと待ってれ

9 名前:132人目の素数さん:2008/05/04(日) 07:26:49
(1)は、自然数a(n)に対して|sin a(n)| < 1だから[ a(n)|sin a(n)| ]<a(n)-1となって、a(n+1) ≦ a(n)

10 名前:132人目の素数さん:2008/05/04(日) 07:54:36
(2)は、あるnについて|sin n| > dならば、m=n+1,n+2,…,n+2008 に対して|sin m| < dとなるようなdが存在することを示せばいいのかな?

11 名前:132人目の素数さん:2008/05/04(日) 15:16:28
>>7
b_n := a_n / 2008

12 名前:11:2008/05/04(日) 15:37:24
そんなわけ無いな。すまそ。

13 名前:132人目の素数さん:2008/05/04(日) 15:42:01
2008って入ってんだな
数学オリンピックの問題みたいだ

14 名前:132人目の素数さん:2008/05/04(日) 16:34:24
|sin(π/2 + n)| (n=1,2,…,2008)の最大値をMとする。
0<θ<π/2の範囲で、M=sin(π/2-θ)となるものをθと置く。
d=sin(π/2-θ/2)とする。このとき、あるnについて|sin n| > dならば、m=n+1,n+2,…,n+2008 に対して|sin m| < dとなる。

|sin k| > dとなるkは無限にあるので、そのうちでk > max{c, 2008/(1-d)} となるものをひとつ取る。
a(n+1) > k+2008 となるようなnが存在するとして、そのようなnのうち最小のものに対しては、k< a(n) ≦ k + 2008となるはずであるが、
その時、|sin a(n)| < dなので、a(n+1) < a(n)d + 2008 < kd+ 2008d + 2008 < k + 2008d となり、
a(n+1) > k+2008であることに矛盾するので、a(n)は有界

15 名前:132人目の素数さん:2008/05/04(日) 19:08:43
x-y平面上の四点(0,0)、(1,0)、(0,1)、(1,1)からの距離が全て有理数になるようなx-y平面上の点を求めよ。

16 名前:132人目の素数さん:2008/05/04(日) 19:21:52
>>15
既出じゃないか?

17 名前:132人目の素数さん:2008/05/04(日) 20:15:10
RからRへの連続写像からなる集合Sを考える。
f_1、f_2∈Sに対し、S^2からR+(非負実数の集合)への写像d(f_1,f_2)を考える。

この時、S,dが距離空間になるようなdの具体例を一つ与えよ。

18 名前:132人目の素数さん:2008/05/04(日) 20:19:35
>>14
前半の議論が分からない。PCで、2008じゃなく10の場合で試してみたら反例が
あったのだが(誤差が丸め込まれていて、実は反例ではないかもしれないが)。

19 名前:132人目の素数さん:2008/05/04(日) 20:26:21
全ての項が整数である数列A(n)はA(n+2)=A(n+1)+A(n)を満たす。
任意の自然数mに対し、m|A(n)なるnが少なくとも一つ存在するとき、A(n)=aF(n+b)、(a,bはある整数、Fはフィボナッチ数列)
であると言えるか。

20 名前:132人目の素数さん:2008/05/04(日) 20:27:48
>>17
ガッコウの宿題か?
f∈Sに対して、pn(f)=sup[|x|≦n]|f(x)|と定義する。
d(f,g)=Σ[n=1〜∞]pn(f−g)/{(2^n)+(2^n)pn(f−g)}
とおけば(S,d)は距離空間。しかも完備。

21 名前:132人目の素数さん:2008/05/04(日) 21:34:37
>>18
単位円を思い浮かべたときに、(cos(π/2 + n),sin(π/2 + n)) (n=1,2,…,2008)をプロットして
一番y軸に近いものとy軸との角度をθとする。その時π/2-θ 〜 π/2+θと、π+π/2-θ 〜 π+π/2+θの範囲には
点が入ってない。(π/2-θなどはx軸正方向からはかった角度)
それを少し回転させて考えると、φ+n (n=1,2,…2008)に対して(cos(φ+n ),sin(φ+n ))をプロットすると
φ±θとπ+φ±θの範囲には点が入ってこなくなる。
φがπ/2±θ/2か、π+π/2±θ/2の範囲にあれば、π/2±θ/2とπ+π/2±θ/2の範囲には点が入ってこない。
d=sin(π/2-θ/2)とすると、|sin(φ)| > dとなるのは、φがπ/2±θ/2か、π+π/2±θ/2の範囲にある時だから
その時、π/2±θ/2とπ+π/2±θ/2の範囲には点が入ってこない⇒n=1,2,…2008で|sin(φ+n)| ≦ dが成り立つ
という考え

22 名前:132人目の素数さん:2008/05/04(日) 21:40:36
>一番y軸に近いものとy軸との角度をθとする。その時π/2-θ 〜 π/2+θと、π+π/2-θ 〜 π+π/2+θの範囲には
>点が入ってない。(π/2-θなどはx軸正方向からはかった角度)
↑ここまではその通りだけど、

>それを少し回転させて考えると、φ+n (n=1,2,…2008)に対して(cos(φ+n ),sin(φ+n ))をプロットすると
>φ±θとπ+φ±θの範囲には点が入ってこなくなる。
>φがπ/2±θ/2か、π+π/2±θ/2の範囲にあれば、π/2±θ/2とπ+π/2±θ/2の範囲には点が入ってこない。
↑これはどうして?

23 名前:132人目の素数さん:2008/05/04(日) 21:49:36
>それを少し回転させて考えると、φ+n (n=1,2,…2008)に対して(cos(φ+n ),sin(φ+n ))をプロットすると
>φ±θとπ+φ±θの範囲には点が入ってこなくなる。
は、最初に描いた(cos(π/2 + n),sin(π/2 + n))を全部いっせいに回転したのと同じだから。
ようするに、φ=π/2 + δとした時に、最初に描いたものをδ回転したことになる

>φがπ/2±θ/2か、π+π/2±θ/2の範囲にあれば、π/2±θ/2とπ+π/2±θ/2の範囲には点が入ってこない。
のほうは、例えばφがπ/2+θ/2の時π/2-θ/2 〜 π/2 + 3θ/2に点が来ないことを考えればわかると思う

24 名前:132人目の素数さん:2008/05/04(日) 21:56:01
>ようするに、φ=π/2 + δとした時に、最初に描いたものをδ回転したことになる
y軸の「近く」には無かった点がφ±θとπ+φ±θの範囲に入る可能性があるのでは?

一応、PCでの計算結果を貼っておく。

cos(1)=0.5403023058681398
cos(2)=-0.4161468365471424
cos(3)=-0.9899924966004454
cos(4)=-0.6536436208636119
cos(5)=0.28366218546322624
cos(6)=0.960170286650366
cos(7)=0.7539022543433046
cos(8)=-0.14550003380861354
cos(9)=-0.9111302618846769
cos(10)=-0.8390715290764524

M   =0.960170286650366 (t=6)
cosθ=0.960170286650366 (θ=0.2831853071795866)

cosθ+cos(2n)=-0.03979053974427116 (n=11)

i=1:cosθ+cos(2n+2i)=1.384349293987363
i=2:cosθ+cos(2n+2i)=1.6070896089790063
i=3:cosθ+cos(2n+2i)=-0.0024355796632006265 ←ここで既に反例
i=4:cosθ+cos(2n+2i)=1.11442173653795



25 名前:132人目の素数さん:2008/05/04(日) 22:00:36
なお、>>14は次のように言い換えられることに注意する(p=2008)。

0<θ<π/2なるθで、max[1≦n≦p]|cos n|=cosθが成り立つものを取る。
もし、cos(2n)+cosθ<0なる自然数nが存在したら、1≦i≦pに対して
cos(2n+2i)+cosθ>0である。

↑例えば、|sin n|>dという式は、両辺2乗して半角の公式・2倍角の公式を使って整理すると
cos(2n)+cosθ<0と変形できる。

>>24の計算は、p=10のときのもの。

26 名前:132人目の素数さん:2008/05/04(日) 22:00:40
f(x)をxの多項式とする。この多項式f(x)に対し、P(f)をf(x)の0でない項の個数とする。
つまり、
f(x)=x^3+2x+1の時、P(f)=3、
f(x)=x^10+2x^3+x+1の時、P(f)=4
である。

この時、P(f^2)≧P(f)は成り立つといえるか。ただし、f^2とは(f(x))^2のことである。


27 名前:132人目の素数さん:2008/05/04(日) 22:12:26
うーむ。
>それを少し回転させて考えると、φ+n (n=1,2,…2008)に対して(cos(φ+n ),sin(φ+n ))をプロットすると
>φ±θとπ+φ±θの範囲には点が入ってこなくなる。
ここはさ、少し回転させる角度がφなのだから、特にφ=0(つまり回転させない)を代入すると、範囲がおかしい。
±θとπ±θの範囲に点が入らないことになってしまう。
π/2−θ+φ〜π/2+θ+φ の範囲と、π+π/2−θ+φ〜π+π/2+θ+φの範囲の間違いでは?


28 名前:132人目の素数さん:2008/05/04(日) 22:20:25
なんか間違えてたかな
自分では何が違うかわからないけど。
>>27はφは回転角じゃないのでφ=π/2が回転させない状態です。わかりにくくてすまん
もう一回考えてみる

29 名前:132人目の素数さん:2008/05/04(日) 22:26:35
>>24
>M   =0.960170286650366 (t=6)
M   =0.9899924966004454 (t=3)では?

30 名前:132人目の素数さん:2008/05/04(日) 22:36:22
>>27はφは回転角じゃないのでφ=π/2が回転させない状態です。わかりにくくてすまん
('A`)オイオイ……
そのφでやってみたら理解できた。これなら合ってると思う。

>>29
そういうことのようです/(^o^)\

31 名前:132人目の素数さん:2008/05/06(火) 11:36:39
転載
http://science6.2ch.net/test/read.cgi/math/1208602546/740

整列集合 X において、切片 X(a)={x∈X | x<a}が非可算集合となる a∈X が
存在するとき、a'∈X を以下のようにとる。
a' = min{a∈X | X(a):非可算}
このとき、Xにおける点列 (x[n])(nは自然数) が a' に収束するならば、
次が成り立つことを示せ。
∃n'(自然数) such that n≧n' ⇒ x[n] = a'

32 名前:132人目の素数さん:2008/05/08(木) 22:05:18
(1)ここにいびつな形のコインが1枚がある。これを使って勝率1/2の賭けをするにはどうしたらいいか
(2)ここにいびつな形のサイコロが1個ある。これを使って勝率1/6の賭けをするにはどうしたらいいか

33 名前:132人目の素数さん:2008/05/08(木) 22:30:56
いびつなコインのほうは2回セットで投げて裏表の順で出るか表裏の順で出るかどちらかにかければいいかな。
裏裏、表表の場合はやり直し。


34 名前:132人目の素数さん:2008/05/08(木) 22:49:58
と思ったけど、表(か裏)が出る確率が0だったら成立しないなぁ。
一応、裏表とも出る確率が0じゃないってことは問題の条件として必要なんじゃない?



35 名前:132人目の素数さん:2008/05/08(木) 22:55:48
>>32
(1) そのコインがどのようにいびつで裏表どちらに偏るようにできているのか解らない以上
裏、または表に賭けて、勝つ確率は1/2
ただし、この方法では同じコインを繰り返し賭けに用いることはできない。

(2) そのサイコロがどのように(ry
1〜6のどれかにかに賭けて勝つ確率は1/6
ただし、この方法では(ry


賭ける先を、裏表ではなくABにし、ABと裏表の対応はディーラーがひと勝負毎に事前に決めておく
という方法なら、同じコインで何度でも賭けができる。
サイコロもA〜Fでやれば同じ。

36 名前:132人目の素数さん:2008/05/08(木) 22:57:47
35の方法は、裏または表どちらかの出る確率が0でも有効

37 名前:132人目の素数さん:2008/05/08(木) 23:01:03
>>32
(1)「どっちの手にコインが入っているか当ててみな」

38 名前:132人目の素数さん:2008/05/08(木) 23:06:24
>>32
(2)「(両手を背後に回して座り込み)サイコロにどの指が触れているか当ててみな」

右手の親指と、人差し指/中指/薬指のいずれか
左手の親指と、人差し指/中指/薬指のいずれか


39 名前:132人目の素数さん:2008/05/08(木) 23:06:49
誰か解ける方、ご教授下さい!
財務関係の利子率rを用いた問題です。

10000×(1+r)^-1 + 10000×(1+r)^-2 + 10000×(1+r)^-3 = 27000

上記が成立するときの、rの値を求めなさい。

答えが何%になるのでしょうか。
どうかよろしくお願いします。

40 名前:132人目の素数さん:2008/05/09(金) 00:48:59
>>39
質問スレに行きなさい。 ここはそういうスレではない。

41 名前:132人目の素数さん:2008/05/11(日) 00:08:39
10個の点
http://puz.hp.infoseek.co.jp/hirameki/suuri_4.html

この問題が解答を見ても分かりません。どなたか解説お願いします。

42 名前:132人目の素数さん:2008/05/11(日) 01:26:32
>>41
今10個の点を固定する。
それらをシートで覆うと必ず一点がはみ出ると仮定すると
各点がはみ出る可能性は1/10、
つまり覆われる可能性は90%ととなる。
それらがシートで覆えない、つまり必ず一つ以上はみ出ると仮定すると
各点がはみ出る可能性は90%以下


43 名前:132人目の素数さん:2008/05/11(日) 01:27:29
訂正
>各点がはみ出る可能性は90%以下
→各点が覆われる可能性は90%以下

44 名前:132人目の素数さん:2008/05/11(日) 01:46:21
必ず少なくとも1点がはみ出てしまうような10個の点の配置があったとする。
10個の点に1,2,…,10と番号をつける。番後iの点が覆われない確率をAiとすると、
P(A1∪A2∪…∪A10)=1が成り立つことになる。P(A1)=P(A2)=…=P(A10)=pと
おくとp<0.1 であるから、P(A1∪A2∪…∪A10)≦P(A1)+P(A2)+…+P(A10)=10p<1
となり、P(A1∪A2∪…∪A10)=1に矛盾する。

45 名前:132人目の素数さん:2008/05/11(日) 01:49:17
>>41
検索したら出てきたほかの解法

10個の点a[1],a[2],…,a[10]を任意に固定する。
ここにシートを適当にかぶせた時、はみ出る点がある確率を考える
点a[i]がはみ出る事象をAiとおくと P[Ai]=100-90.69%=9.31%
したがって
P[少なくとも一つの点がはみ出す事象]
=P[A1∪A2∪…∪A10]
≦P[A1]+P[A2]+…+P[A10]
=93.1%<100%
したがって少なくとも6.9%分、全ての点を覆う事象が存在する。

46 名前:132人目の素数さん:2008/05/11(日) 05:29:59
>>39

http://science6.2ch.net/test/read.cgi/math/1209929873/167
さくらスレ242


47 名前:132人目の素数さん:2008/05/12(月) 16:36:07
>>46
全然面白くない

48 名前:132人目の素数さん:2008/05/12(月) 17:41:20
>>41です。まだ良く分かってませんが、のんびりと考えてみます。ありがとうございます。

49 名前:132人目の素数さん:2008/05/13(火) 01:10:51
一辺の長さ30の正方形をいくつかの多角形に分割する。
それぞれの多角形はその中のどの2点を取っても、二点間の距離が1以下になると言う。
この時、ある多角形Pが存在し、Pに隣接する多角形が六つあることを示せ。
ただし、隣接する多角形A,Bとは、A,Bに属する点をそれぞれ適切に選べば、0に出来るような多角形A,Bのことを言う。

50 名前:132人目の素数さん:2008/05/13(火) 11:28:32
>>49
> ただし、隣接する多角形A,Bとは、A,Bに属する点をそれぞれ適切に選べば、0に出来るような多角形A,Bのことを言う。 

これの意味がわからん

51 名前:132人目の素数さん:2008/05/13(火) 11:30:30
ただし、隣接する多角形A,Bとは、A,Bに属する点をそれぞれ適切に選べば、0に出来るような多角形A,Bのことを言う。

ごめん、
ただし、隣接する多角形A,Bとは、A,Bに属する点をそれぞれ適切に選べば、それらに転換の距離を0に出来るような多角形A,Bのことを言う。

52 名前:132人目の素数さん:2008/05/14(水) 20:18:04
>32 (2) 普通に手本引きができる予感

53 名前:132人目の素数さん:2008/05/16(金) 23:22:40
昔どっかの本で見た問題。

回転する円形のテーブルの周上に、区別のつかないn個の小箱が、
正n角形の頂点をなす配置で固定されている。小箱にはそれぞれ
1枚ずつコインが入っていて、それらが「全部表」もしくは「全部裏」の
状態になった瞬間にチャイムが鳴る仕掛けになっている。
このチャイムを鳴らすことが目的である。

さて、あなたは同時に好きな2つの箱を開けて中を確認し、コインの
状態(裏/表)を自由に変えることができる。これを「一手」とする。
一手が済んだらあなたには目をつぶってもらい、その間に誰かが
テーブルを無作為に回転させる。テーブルが止まったら二手目をやる。
また回転させる‥‥

これを繰り返し、有限手のうちにチャイムが鳴ればあなたの勝ち。

(1) n=3 のときの必勝戦術を考えよ。
(2) n=4 のときの必勝戦術を考えよ。
(3) n=5 のとき、必勝戦術はあるか?

※(3)はよくわかりません。

54 名前:132人目の素数さん:2008/05/17(土) 01:09:37
>>53
(1)
Step 1: 選んだ2つを両方表にする
Step 2: 選んだ2つが両方表なら両方裏にする
     表と裏なら両方表にする

(2)
Step 1: 隣り合う2つを選び、両方表にする
Step 2: 対角にある2つを選び、両方表にする。
Step 3: 隣り合う2つを選び、表と裏なら両方表にして終了。
     両方表なら片方を裏にする。
Step 4: 対角にある2つを選び、両方同じなら両方を反転して終了。
     表と裏ならそのまま。
Step 5: 隣り合う2つを選び、両方を反転。
Step 6: 対角にある2つを選び、両方を反転。

(3)は、5つの箱のうち2つを全く選択できないという
可能性があり得て、そこに裏と表が入っていれば
クリアできない。よって必勝戦術はない。

55 名前:132人目の素数さん:2008/05/18(日) 09:36:34
age

56 名前:53:2008/05/18(日) 20:31:57
>>54
はやっ
お見事。

57 名前:132人目の素数さん:2008/05/19(月) 16:38:24
{1,2,3,....p-1 |p素数 }をp-1個の変数と見なしてk (=1.2...p-2)次基本対称式をつくると
mod pで0となることを示せ。

58 名前:132人目の素数さん:2008/05/19(月) 19:32:36
>>57
p元体F_p上の1変数多項式環F_p[x]を考える。この環はUFD。
多項式x^(p-1)-1について、フェルマーの小定理から、
この多項式に1,2…,p-1を代入すると0になる。ゆえ因数定理から
x^(p-1)-1=(x-1)(x-2)…{x-(p-1)}と因数分解できる。
右辺を展開したものと左辺を比較すれば>>57が示される。

なお、定数項の比較からはWillsonの定理が得られる。

59 名前:132人目の素数さん:2008/05/25(日) 05:38:36
最近どっかで見た問題の変形。

回転する円形のテーブルの周上に、区別のつかないn個の小箱が、
正n角形の頂点をなす配置で固定されている。小箱にはそれぞれ
1個ずつサイコロが入っていて、それらが「全部1」もしくは「全部2」もしくは…もしくは「全部6」の
状態になった瞬間にチャイムが鳴る仕掛けになっている。
このチャイムを鳴らすことが目的である。

さて、あなたは同時に好きな2つの箱を開けて中を確認し、サイコロの
状態(1/2/3/4/5/6)を自由に変えることができる。これを「一手」とする。
一手が済んだらあなたには目をつぶってもらい、その間に誰かが
テーブルを無作為に回転させる。テーブルが止まったら二手目をやる。
また回転させる‥‥

これを繰り返し、有限手のうちにチャイムが鳴ればあなたの勝ち。

(1) n=3 のときの必勝戦術を考えよ。
(2) n=4 のとき、必勝戦術はあるか?

※(2)はよくわかりません。

60 名前:132人目の素数さん:2008/05/25(日) 05:50:37
>>59
(1)はたいして変わらん

・1,1にする。
・ぞろ目が出たら、両方+1にする、ぞろ目でなければ小さいほうの目にそろえる。




61 名前:132人目の素数さん:2008/05/25(日) 17:21:43
□に−,×,÷の何れかを入れて等式を成立させて下さい
+の使用や空白(12にする等)は不可

 1□2□3□4□5□6□7□8□9=1

(全通り求めて下さい)

62 名前:132人目の素数さん:2008/05/25(日) 17:23:05
>>61
パズル板いけ

63 名前:132人目の素数さん:2008/05/25(日) 19:43:40
>>61みたいな虫食い系は問題数も多く粗製乱造になりがち。
前スレは酷い有様であった。

要するにパズル板行け。

64 名前:132人目の素数さん:2008/05/25(日) 22:38:07
直径5kmのトーチタスがワシントンに落ちたら人類滅亡までどれくらいかかるか?

65 名前:132人目の素数さん:2008/05/25(日) 22:44:59
VIPで糞スレ立てて教えてもらえ

66 名前:132人目の素数さん:2008/05/26(月) 00:34:52
>>59 (2)
「出目が2種類のみの状態であり、かつその2種類の数値が判明している」★
この状態は、コインの場合に帰着されるのでクリアできる。

[手順1](任意の状態から)
まず対角を11とし、続けて隣接を11とする。
これでクリアしないなら、111X(2≦X)になっている。

[手順2](状態111X, 2≦X から)
隣接をとり、Xが出たらそれを1にしてクリア。
11が出たら22にする。これで221X(2≦X)になる。

[手順3](状態221X, 2≦X から)
対角をとり、22ならX=2であり、★の状態なのでクリア。
2X(3≦X)ならX→2で★になるのでクリア。
12なら、1→2で222X(3≦X)になる。

あとは[手順2][手順3]を、数値を1つずつ上げながら実行することで、
Xはどんどん追いつめられていって、最長まで粘っても555X(6≦X)、
すなわち5556となる。これは★なのでクリア。おしまい。

どっかに穴があるかも。


67 名前:132人目の素数さん:2008/05/26(月) 20:51:40
>>61
1*2/3*4*5*6-7-8*9=1
これ以外ある?

68 名前:132人目の素数さん:2008/05/26(月) 22:59:14
1 = 1+2+3+4+5-6-7+8-9 = 1+2+3+4-5+6+7-8-9 = 1+2+3+4*5-6*7+8+9
= 1+2+3-4-5-6-7+8+9 = 1+2+3*4*5/6*7-8*9 = 1+2-3+4-5-6+7-8+9
= 1+2-3-4+5+6-7-8+9 = 1+2-3-4+5-6+7+8-9 = 1+2-3*4*5/6+7-8+9
= 1+2-3/4-5-6*7/8+9 = 1+2*3-4*5+6+7-8+9 = 1+2*3*4-5*6+7+8-9
= 1-2+3+4-5+6-7-8+9 = 1-2+3+4-5-6+7+8-9 = 1-2+3-4+5+6-7+8-9
= 1-2+3*4*5/6-7+8-9 = 1-2+3/4+5+6*7/8-9 = 1-2-3+4+5+6+7-8-9
= 1-2-3-4+5-6-7+8+9 = 1-2-3-4-5+6+7-8+9 = 1-2-3-4*5+6*7-8-9
= 1-2-3*4*5/6*7+8*9 = 1-2*3+4*5-6-7+8-9 = 1-2*3*4+5*6-7-8+9
= 1*2+3-4*5+6-7+8+9 = 1*2+3*4+5+6-7-8-9 = 1*2-3+4*5+6-7-8-9
= 1*2-3-4+5*6-7-8-9 = 1*2-3*4-5+6-7+8+9 = 1*2-3*4*5+6*7+8+9
= 1*2-3*4*5-6+7*8+9 = 1*2-3*4*5-6-7+8*9 = 1*2*3+4+5-6-7+8-9
= 1*2*3+4-5+6+7-8-9 = 1*2*3+4*5-6*7+8+9 = 1*2*3-4-5-6-7+8+9
= 1*2*3*4-5+6-7-8-9 = 1*2/3*4*5*6-7-8*9 = 1/2-3/4+5+6*7/8-9
= 1/2*3*4-5+6-7-8+9 = 1/2*3*4-5-6+7+8-9 = 1/2/3-4+5/6*7+8-9

69 名前:132人目の素数さん:2008/05/26(月) 23:17:30
10 = 1+2+3+4*5-6+7-8-9 = 1+2+3*4-5+6-7-8+9 = 1+2+3*4-5-6+7+8-9
= 1+2+3*4*5-6-7*8+9 = 1+2-3+4+5*6-7-8-9 = 1+2-3-4*5+6+7+8+9
= 1+2-3-4*5-6*7+8*9 = 1+2-3*4+5+6+7-8+9 = 1+2*3+4-5-6-7+8+9
= 1+2*3-4+5-6+7-8+9 = 1+2*3-4-5+6+7+8-9 = 1+2/3/4+5/6+7-8+9
= 1-2+3+4*5-6-7-8+9 = 1-2+3*4-5-6-7+8+9 = 1-2-3*4+5-6+7+8+9
= 1-2-3*4*5+6+7*8+9 = 1-2-3*4*5+6-7+8*9 = 1-2*3+4-5+6-7+8+9
= 1-2*3-4+5+6+7-8+9 = 1-2/3/4-5/6-7+8+9 = 1*2+3+4+5+6+7-8-9
= 1*2+3-4+5-6-7+8+9 = 1*2+3-4-5+6+7-8+9 = 1*2+3-4*5+6*7-8-9
= 1*2-3+4+5-6+7-8+9 = 1*2-3+4-5+6+7+8-9 = 1*2-3*4+5*6+7-8-9
= 1*2-3*4-5+6*7-8-9 = 1*2-3/4+5-6*7/8+9 = 1*2*3+4*5-6+7-8-9
= 1*2*3*4*5/6+7-8-9 = 1*2/3+4*5/6+7+8-9 = 1/2+3/4+5-6*7/8+9
= 1/2/3*4*5*6+7-8-9

100 = 1+2+3+4+5+6+7+8*9 = 1+2+3-4*5+6*7+8*9 = 1+2-3*4+5*6+7+8*9
= 1+2-3*4-5+6*7+8*9 = 1+2*3+4*5-6+7+8*9 = 1+2*3*4*5/6+7+8*9
= 1-2+3*4*5+6*7+8-9 = 1-2+3*4*5-6+7*8-9 = 1-2*3+4*5+6+7+8*9
= 1-2*3-4+5*6+7+8*9 = 1-2*3-4-5+6*7+8*9 = 1*2*3+4+5+6+7+8*9
= 1*2*3-4*5+6*7+8*9 = 1*2*3*4+5+6+7*8+9 = 1*2*3*4+5+6-7+8*9

70 名前:132人目の素数さん:2008/05/26(月) 23:34:46
なんでこう問題すらろくに読まないやつが得意げにレスしてるわけ?

71 名前:132人目の素数さん:2008/05/27(火) 00:09:34
>>70
>>62>>63
レスがついたことをありがたく思え

72 名前:132人目の素数さん:2008/05/27(火) 00:55:07
なんで出題者でもないのに有り難がらないといけないのか?

73 名前:132人目の素数さん:2008/05/27(火) 00:55:58
もし出題者だとしても有り難くないよな、問題も読まないような奴は

74 名前:132人目の素数さん:2008/05/27(火) 01:41:39
61のたぐいのパズルは出題者しか有難くない。パズル板でやれ。

75 名前:132人目の素数さん:2008/05/27(火) 02:07:02
読みもしないで得意げにレスする奴には有り難いみたいだよ

76 名前:132人目の素数さん:2008/05/27(火) 02:21:40
昔、他所で未解決だった問題

1辺が 1 の正四面体 OPQR の辺 OP, OQ, OR 上にそれぞれ
動点 X, Y, Z が存在し、OX*OY*OZ = 1/3 を満たしながら動くとき、
△XYZ(内部を含む)が動く領域の体積を求めよ。

77 名前:132人目の素数さん:2008/05/27(火) 18:56:27
蟻=鯛

78 名前:132人目の素数さん:2008/05/28(水) 10:35:06
1×1の正方形を4個縦横につなげた図形をテトロミノと言い5種類あります
(回転,裏返しで重なるものは同じ図形と見なします)
この5種類のテトロミノを以下の条件で組み合わせて6×6の正方形にして下さい
(1)パーツの回転,裏返し可
(2)パーツを重ねたり隙間があるものは不可
(3)どの種類のテトロミノも最低1つは使って下さい
(4)同じ種類のテトロミノが縦あるいは横で隣り合ってはいけません

全部で何通りあるか?

79 名前:132人目の素数さん:2008/05/28(水) 17:20:16
nPk < (k! 2^n)/√n が成り立つことを示せ
ただしnPkは順列の個数を意味する

80 名前:132人目の素数さん:2008/05/29(木) 20:35:33
AP+PBを最小にする円周上のPの位置を求めよ
http://i.pic.to/q71vu

81 名前:132人目の素数さん:2008/05/29(木) 20:50:14
>>80
PCからでも見れる所にupしねーとわからねー!

82 名前:132人目の素数さん:2008/05/29(木) 23:29:27

落書きで困っている商店街がある。

その商店街のシャッターに『落書きするな』と書くのは

落書き であろうか。


83 名前:80:2008/05/29(木) 23:45:34
>>81
見れるようにしました
http://i.pic.to/q71vu

84 名前:132人目の素数さん:2008/05/30(金) 01:05:23
>>82
落書きの定義による。
通常は、権利者(たいていはシャッターの持ち主)に無断で書いたものは落書き。

85 名前:132人目の素数さん:2008/05/30(金) 01:05:57
>>83
∠APO=∠BPOになるの点P

86 名前:132人目の素数さん:2008/05/30(金) 12:09:18
>>85
証明は?

87 名前:132人目の素数さん:2008/05/30(金) 20:53:01
>>86
つ「接線について線対称の位置にA'を置く」

88 名前:132人目の素数さん:2008/05/30(金) 20:54:41
どこの接線?

89 名前:132人目の素数さん:2008/05/30(金) 20:57:15
釣りか? 普通に考えりゃ円のだろ。
他に接線が引けそうなとこがあるか?

90 名前:132人目の素数さん:2008/05/30(金) 21:05:52
でも、それで証明になるのか?

91 名前:132人目の素数さん:2008/05/30(金) 22:12:53
この場合接線も動くのでその方法だと証明にならないと思う

92 名前:132人目の素数さん:2008/05/31(土) 20:41:10
>>79

ここら辺↓に解答…

http://science6.2ch.net/test/read.cgi/math/1179000000/333-334
不等式スレ3

93 名前:132人目の素数さん:2008/06/01(日) 18:47:47
nは3以上の自然数とする。1辺の長さが1の正方形を碁盤の目のように
縦にn個ずつ、横にn個ずつ、全部でn^2個敷き詰める(一辺の長さがnの
大きな正方形が出来上がる)。このn^2個の正方形のうち、k個の正方形を
黒で塗りつぶす。ただし、次のような配置が出来ないように塗りつぶす。

 ■ … ■

  :    :  (4つの■が 正 方 形 の4頂点を形作るような配置)

 ■ … ■

例:n=3,k=4のとき。

これは当然OK   これもOK   これは×  これも×
  □■■      □■■    ■□■   ■■□
  ■□□      □□□    □□□   ■■□
  □□■      □■■    ■□■   □□□

k≦(1/√3)*(8n^5)^(1/4)のとき、このような塗りつぶしは可能であることを示せ。

94 名前:132人目の素数さん:2008/06/01(日) 21:25:36
>>91
接線を動かす必要はないんじゃないか?

点x、y、zを順に通る折れ線をx〜y〜zと書くこととする。

円周上に∠APO=∠BPOになるように点Pを置く(ただし∠APO>π/2)。
点Pに円の接線Lを引く。

ここで、A〜接線L〜Bをつなぐ線の最短は、例の線対称点A’を考えれば
A〜P〜Bであることは容易にわかる。

円周上のP以外の点Qをとる、QをどこにとってもA〜Q〜Bよりも短い
A〜R〜Bとなるような点Rが接線L上に存在する。 


95 名前:132人目の素数さん:2008/06/04(水) 05:07:01
penis out

96 名前:132人目の素数さん:2008/06/07(土) 18:09:46
直線を6本引き1辺の長さが1,2,3,4,5,6,7,8の正三角形(計8個)を同時に作って下さい

97 名前:132人目の素数さん:2008/06/07(土) 20:13:47
立体的に考えれば余裕

98 名前:132人目の素数さん:2008/06/07(土) 21:21:28
http://www.iis.it-hiroshima.ac.jp/~ohkawa/math/math_problem_all.htm#103
簡単じゃないよ。

99 名前:132人目の素数さん:2008/06/08(日) 07:53:17
>>96
2本の線が√2離れた平行線を用意する。
そこに60°の角度で2本の線が2√2離れた平行線を引く。
以上の交差部分を挟むように(9√2)/2離れた平行線を
最初の線から-60°の角度に引く。

100 名前:132人目の素数さん:2008/06/10(火) 23:23:57
任意の円錐は、任意の楕円を断面としてもつ。真or偽?

ただし断面とは平面による切断面とし、
円錐の高さに制限は設けない。

101 名前:132人目の素数さん:2008/06/11(水) 01:09:29
直感的には真だな

102 名前:132人目の素数さん:2008/06/11(水) 02:48:55
円と放物線が作れるから、その間の任意の離心率の楕円が作れる
切る高さを調節すれば、任意の大きさにできるってことか

103 名前:132人目の素数さん:2008/06/12(木) 17:16:44

対偶で考えたら明らか

104 名前:132人目の素数さん:2008/06/12(木) 18:04:21
対偶で考えたら明らかになるようなものなのか?

105 名前:132人目の素数さん:2008/06/12(木) 19:19:29
◆ わからない問題はここに書いてね 244 ◆
http://science6.2ch.net/test/read.cgi/math/1211985803/866
から

表と裏が等確率で出るコインを連続して投げて、
1000回連続で表、もしくは1000回連続で裏が出る事象が
99% 以上の確率で起こるためには何回コインを投げればいいか。
有効数字1桁で求めよ。

106 名前:132人目の素数さん:2008/06/13(金) 19:01:22
■が縦横斜(対角線以外も含む)に4個連続して並ばない様にするには
最低何個の■を取り除けばよいでしょう?
又,取り除く箇所は?
■■■■■■
■■■■■■
■■■■■■
■■■■■■
■■■■■■
■■■■■■

107 名前:132人目の素数さん:2008/06/13(金) 19:46:20
■■■□■■
■■□■■■
□■■■□■
■□■■■□
■■■□■■
■■□■■■

後2個置くのだが美しくないな、
だからきっとここまでも正しくないのであろう。

108 名前:132人目の素数さん:2008/06/13(金) 21:32:48
俺は正しそうな気がするなぁ。
8個じゃ無理っぽいし、答えが奇数の9になるというのもなんとなく考えにくいし。

109 名前:132人目の素数さん:2008/06/13(金) 22:05:14
一一一一三四
二二二二三四
五六■■三四
五六■■三四
五六七七七七
五六八八八八

各々最低1つは取り除かなきゃいやん

110 名前:132人目の素数さん:2008/06/13(金) 22:06:33
ごめん8個じゃ無理的な話をしてたのか
俺カッコワルイ
吊ってくる


111 名前:132人目の素数さん:2008/06/13(金) 22:35:40
とりあえず、鳩ノ巣原理で考えてみる。

まず、4の並びが幾つあるか数える。
縦18横18斜め18計54個ある。

升目を次のようにグループ分けする。

ABCCBA
BDEEDB
CEFFEB
CEFFEB
BDEEDB
ABCCBA


112 名前:132人目の素数さん:2008/06/13(金) 22:36:16
それぞれのグループを一つ消したとき並ばなくなる4の並びの数は

A 3
B 4
C 5
D 6
E 8
F 12

である。

113 名前:132人目の素数さん:2008/06/13(金) 22:36:54
>>109より
ABCから4個、BDEから4個、最低取り除かなければならない。
もっとも効率がよいのはC4個、E4個取り除いたときである。
このとき並ばなくなる4の並びは(5+8)*4=52である。

よって最低でも9個は取り除かなければならない。

までしかわからんカッタ。
最後の詰めは頼んだ。



114 名前:132人目の素数さん:2008/06/13(金) 23:02:53
>>111に間違いがありました。すいません。
正しくは以下。

ABCCBA
BDEEDB
CEFFEC
CEFFEC
BDEEDB
ABCCBA



115 名前:132人目の素数さん:2008/06/14(土) 10:54:35
0個以上消す必要があることを証明する。

中央の4マスの中からいくつ消すかで場合わけする。

(1)中央の4マスから2つ以上消した場合。
>>109より中央の4マス以外で8個消す必要がある。
中央の4マスで2マス以上消すので合計10個以上消さなければいけない。


116 名前:132人目の素数さん:2008/06/14(土) 10:55:07
(2)中央の4マスから一つも消さなかった場合。
A〜Lのグループから少なくとも1つ以上消さなければならない。
よって12個以上消さなければならない。

AEEEEB
FAIJBG
FK■■KG
FL■■LG
FCIJDG
CHHHHD


117 名前:132人目の素数さん:2008/06/14(土) 10:56:25
(3)中央の4マスから一つ消した場合。

以下のようにグループ分けすると
A〜Iから少なくとも一つ消さなくてはいけない。
よって10個以上消さなくてはいけない。

■AAAAD
B■■EDG
B■□■GI
BF■■FI
BCGE■I
CHHHHI

以上より証明された。


118 名前:132人目の素数さん:2008/06/14(土) 10:58:46
>>115の一行目は
10個以上消す必要があることを証明する。
が正しいです。

図もずれてますね。
すいません。


119 名前:132人目の素数さん:2008/06/14(土) 11:14:45
なるほど、乙

120 名前:132人目の素数さん:2008/06/14(土) 13:56:59
>>106の類似問題
■が縦横斜(対角線以外も含む)に5個連続して並ばない様にするには最低何個の■を取り除けばよいでしょう?(証明も)

■■■■■■■
■■■■■■■
■■■■■■■
■■■■■■■
■■■■■■■
■■■■■■■
■■■■■■■

121 名前:132人目の素数さん:2008/06/14(土) 13:59:38
■ おすすめ2ちゃんねる 開発中。。。 by FOX ★
このスレを見ている人はこんなスレも見ています。(ver 0.20)
☆明晰夢☆ピカッ☆ [心理学]
♂容姿を女の子っぽくしたい part12♂ [美容]

122 名前:132人目の素数さん:2008/06/14(土) 14:08:32
>>120
正解の形はきれいだけど、証明は前と同じにできるし面白みがない

123 名前:132人目の素数さん:2008/06/14(土) 19:39:40
>>120
■の間に灰色の点が見えるぞ!

124 名前:132人目の素数さん:2008/06/14(土) 19:41:57
>>123
すげ

125 名前:132人目の素数さん:2008/06/14(土) 22:12:03
[問題]
>>120の■の間に灰色の点が見えるのはなぜか?

126 名前:132人目の素数さん:2008/06/14(土) 22:14:36
>>125
板違い。生物板などへどうぞ。

127 名前:132人目の素数さん:2008/06/14(土) 23:14:26
>>126
あれ、むしろ哲学板じゃねえ?

128 名前:132人目の素数さん:2008/06/14(土) 23:15:21
認識学

129 名前:132人目の素数さん:2008/06/15(日) 04:36:17
コラッツ予想の論文で使われていた手法を応用して出来た問題。
俺の勘違いかもしれないので、解けなくても責任は負わぬ(^o^)

f:N→N を f(n)=n/2 (nは偶数),2007n+1 (nは奇数)
として定義する。このとき、lim[k→∞]f^k(n)=+∞ を満たす
自然数nが存在することを示せ。ただしf^kはfのk回合成関数とする。

130 名前:132人目の素数さん:2008/06/15(日) 14:40:59
>>125
俺は何回見ても見えないんだけど……

131 名前:132人目の素数さん:2008/06/15(日) 16:35:28
■■■■
■■■×
××■×

↑この図形をいくつか組み合わせて正方形にして下さい
ただし正方形の面積ができるだけ小さくなる様にして下さい
(重ねたり隙間があるものは不可)

132 名前:132人目の素数さん:2008/06/15(日) 18:42:16
>>131
×のところに■を重ねるのはおk?

133 名前:132人目の素数さん:2008/06/15(日) 20:49:51
裏返すのはありなのか?

134 名前:132人目の素数さん:2008/06/15(日) 21:01:31

■■■
■■
■■
って書けばいいのに

最低でも8こ使わないとできないな
結構面倒くさい

135 名前:132人目の素数さん:2008/06/15(日) 21:21:55
8個で出来ないと次の候補は18個かのう。






136 名前:132人目の素数さん:2008/06/15(日) 22:13:23
あー裏返しありでも多分8こじゃ無理だわ
18こもやってらんねー

137 名前:132人目の素数さん:2008/06/15(日) 22:16:19
プログラム書いてやったら、18個もダメ
32個だと大量の解がある
全く自信ないが

138 名前:元も子もない:2008/06/15(日) 22:23:15
切っちゃダメなのか?

139 名前:132人目の素数さん:2008/06/15(日) 22:34:30
この形がキーとなるか?

×■■■■×
□■■■□□
□□□■□□
□□■□□□
□□■■■□
×■■■■×


140 名前:132人目の素数さん:2008/06/15(日) 22:37:26
その形を数片に切り分けて並べなおし正方形にしたい。
最低何篇に切り分ければよいか。

141 名前:132人目の素数さん:2008/06/15(日) 23:09:20
>>137
詳しく

142 名前:132人目の素数さん:2008/06/15(日) 23:45:45
32個で急に大量の解が見つかるってのも凄いな。

143 名前:132人目の素数さん:2008/06/16(月) 00:49:57
等幅フォント使えないとこでどう書くかのほうが難しいんだが…
32個だと、回転、反転したものは別の解と数えて 512解ある
下が解の一例

以以以以留留留留闘闘闘闘於於於於
以以以知知留留留闘闘闘為為於於於
呂呂以知知留与与与与闘為為於末末
呂呂知知知遠遠与与与称為為為末末
呂呂呂反知遠遠与称称称為乃末末末
呂反反反遠遠遠太称称称称乃乃乃末
波反反反反奴遠太太太武乃乃乃乃計
波波波奴奴奴太太太太武武武計計計
波波止止奴奴礼礼礼礼武武久久計計
波波止止奴奴礼礼礼奈武武久久計計
仁止止止和和和和礼奈奈奈久久久不
仁仁仁止和和和曽奈奈奈奈久不不不
仁仁利利利利和曽曽曽宇宇宇宇不不
仁仁保利利利加曽曽良宇宇宇也不不
保保保利加加加曽曽良良良宇也也也
保保保保加加加加良良良良也也也也

144 名前:132人目の素数さん:2008/06/16(月) 06:09:04
なんか見づらい…
平面地図は4色で塗れるだろうに

145 名前:132人目の素数さん:2008/06/16(月) 06:44:05
>>131の類似問題


■■



↑この図形をいくつか組み合わせてこれと相似な図形にして下さい
ただし面積ができるだけ小さくなる様にして下さい
(重ねたり隙間があるものは不可)

146 名前:132人目の素数さん:2008/06/16(月) 07:26:37
>>125は「ヘルマン格子」でググるとわかる

147 名前:132人目の素数さん:2008/06/16(月) 14:43:05
ヘルマン格子でぐぐってもおそらく「なぜ起こるのか」はわからない。
(幾つかの仮説は提示されているが、どれもまだ決定的なものではない

148 名前:132人目の素数さん:2008/06/16(月) 18:24:14
>>143
┏━━━┳━━━┳━━━┳━━━┓
┃    ┏┻┓    ┃    ┏┻┓    ┃
┣━┓┃  ┃┏━┻━┓┃  ┃┏━┫
┃  ┣┛  ┣┻┓    ┣┫  ┗┫  ┃
┃  ┗┳┓┃  ┃┏━┛┃┏┳┛  ┃
┃┏━┛┣┛  ┣┫    ┗┫┗━┓┃
┣┫    ┗┳┓┃┗━┳┳┛    ┣┫
┃┗━┳━┛┣┛    ┃┗━┳━┛┃
┃  ┏┻┓  ┣━━━┫  ┏┻┓  ┃
┃  ┃  ┃  ┃    ┏┫  ┃  ┃  ┃
┣┳┛  ┣━┻━┓┃┗━┫  ┗┳┫
┃┗━┓┃    ┏╋┛    ┃┏━┛┃
┃  ┏┻┻━┓┃┗━┳━┻┻┓  ┃
┃  ┣┓    ┣┫  ┏┫    ┏┫  ┃
┣━┛┃┏━┛┃  ┃┗━┓┃┗━┫
┃    ┗┫    ┗┳┛    ┣┛    ┃
┗━━━┻━━━┻━━━┻━━━┛

149 名前:132人目の素数さん:2008/06/16(月) 18:26:54
GJ
ついでにage

150 名前:132人目の素数さん:2008/06/16(月) 18:49:28
>>148
ぜんぜん正方形じゃないじゃん

151 名前:132人目の素数さん:2008/06/16(月) 18:55:45
どうみても16*16の正方形だが

152 名前:132人目の素数さん:2008/06/16(月) 18:59:41
そうか?俺には縦が長く見える

153 名前:132人目の素数さん:2008/06/16(月) 19:04:12
それは行間のせいだろう

154 名前:132人目の素数さん:2008/06/16(月) 19:11:15
>>152
頭固すぎ

155 名前:132人目の素数さん:2008/06/16(月) 19:14:27
明和さん 664だよ
早く早く

156 名前:132人目の素数さん:2008/06/16(月) 20:53:57
画面がゆがんでんじゃね?

157 名前:132人目の素数さん:2008/06/16(月) 21:01:26
こんな感じか 

        /./                   | |
      _/ /-─'''"~~ ,.二.フ->      ! ,'
    / ./, '-─‐ '" ̄  / '´\\    ,' /
.  / // ニ二u__,/ // u,__ヽ 〉- 、//
  {//-===:、     |.! /  / { | r‐ノ/         /
  /,.イ u   __\、;;;||/ @ 〃,.-;=´イ,ヽ        //
. //ヾ.\ /   @ フ''| |: ミ≡彡' / _\ヽ |       ,' ,'
/ i´r\ヽu`≡==彡v{ .{._,.ノ/u,ノ u_ \!\     / /
  ヽ.ヽrヽ.} r,ラ',ニニ二´-‐''´、 ̄ ./ヽ/ヽ.ヽ ` ー-/ ./
    \`、レ',.イー' ( __ } | - ’,. ‐ヘ /  ,| |  _//    
.     `//_| rーrー┬ゝィ‐''1´ ,レ'´  /ン ! /、'´ ̄
     //  ! l‐┴‐┴‐┴‐ '"´  /ヽ/ u |、//  `丶
  ,r‐' /   \ヽ.r‐┬‐┬ ''1´.工 -‐'´u  |//\
 / /  /|   \` ┴‐''' ´ ̄ ==''___/// ヽ
/ /  ̄ ̄|   |    `ー、'''"~~´ ̄ ̄   //
./    |   |      \-───‐- 、//


158 名前:132人目の素数さん:2008/06/16(月) 21:53:20
いや、縦横比がってこと

159 名前:132人目の素数さん:2008/06/16(月) 21:58:19
>>137
ちなみに計算時間とか使用メモリはどれぐらい?


160 名前:132人目の素数さん:2008/06/16(月) 22:09:47
20秒

161 名前:132人目の素数さん:2008/06/16(月) 22:11:14
速ッ


162 名前:132人目の素数さん:2008/06/16(月) 22:12:11
□に全て異なる正の整数(何桁でも可)を入れ,
縦横対角線の□の積を全て同じにして下さい
ただし積ができるだけ小さくなる様にして下さい.同じ数字2度使いは不可

□─□─□─□
│\│ │/│
□─□─□─□
│ │×│ │
□─□─□─□
│/│ │\│
□─□─□─□

163 名前:132人目の素数さん:2008/06/16(月) 22:20:05
4x4の魔方陣の座標x,yの値をA_x_yとおいて、
各座標x,yに2^A_x_yを入れりゃいーんじゃね。


164 名前:132人目の素数さん:2008/06/16(月) 22:31:29
はて?偶数の魔方陣ってあったかな。

165 名前:132人目の素数さん:2008/06/16(月) 22:33:26


166 名前:132人目の素数さん:2008/06/16(月) 22:41:45
積?


167 名前:132人目の素数さん:2008/06/16(月) 22:44:05
積!

168 名前:132人目の素数さん:2008/06/16(月) 22:51:13
いやいやまってくれ。俺が悪かった。
わかるように説明してくれ。

>縦横対角線の□の【積】を全て同じにして下さい

ここが積だといってるんだな?            
で、そのせいで>>163が間違っているといっている?

169 名前:132人目の素数さん:2008/06/16(月) 22:57:17
1が使えるから2で割れよ

170 名前:132人目の素数さん:2008/06/16(月) 23:11:32
素因数分解に注目すれば完成させるのは余裕
あとは最小にするにはどうするか

171 名前:132人目の素数さん:2008/06/16(月) 23:48:24
素数を、2の倍数、3の倍数を除いた自然数として、
双子の素数が無限にあることを証明せよ

172 名前:132人目の素数さん:2008/06/16(月) 23:53:04
2の倍数を除いたら、全部とびとびじゃんか

173 名前:132人目の素数さん:2008/06/16(月) 23:54:54
6ずつ足せよ

174 名前:132人目の素数さん:2008/06/17(火) 00:01:04
http://science6.2ch.net/test/read.cgi/math/1085836534/l50

175 名前:132人目の素数さん:2008/06/17(火) 03:48:41
反転、回転で重なりあうものをひとつに数えると68解になった
うち、対称性のない解が 60個、180°の回転に対する対称性のあるものが 8個
全ての解が、ひとつの辺に4個、それに向かい合う辺にも4個、
残りの2辺に5個のピースが接している

対称性のある解の例
┏━━━┳━━━┳━━━┳━━━┓
┃    ┏┫    ┏┻┓    ┣┓    ┃
┣━┓┃┗━┓┃  ┃┏━┛┃┏━┫
┃  ┣┛    ┣┫  ┗┫    ┗┫  ┃
┃  ┗┳┳━┛┃┏━┻━┳┳┛  ┃
┃┏━┛┃    ┗╋┓    ┃┗━┓┃
┣┫    ┗┳┳━┛┃┏━┫  ┏┻┫
┃┗━┳━┛┃    ┗┫  ┃  ┃  ┃
┃  ┏┻┓  ┣━━━┫  ┗┳┛  ┃
┃  ┃  ┃  ┣┓    ┃┏━┻━┓┃
┣┳┛  ┣━┛┃┏━┻┻┓    ┣┫
┃┗━┓┃    ┗╋┓    ┃┏━┛┃
┃  ┏┻┻━┳━┛┃┏━┻┻┓  ┃
┃  ┣┓    ┣┓  ┣┫    ┏┫  ┃
┣━┛┃┏━┛┃  ┃┗━┓┃┗━┫
┃    ┗┫    ┗┳┛    ┣┛    ┃
┗━━━┻━━━┻━━━┻━━━┛

>>160 うちのペン2のPCで1秒だけど

176 名前:132人目の素数さん:2008/06/17(火) 09:37:38
>>175
> 全ての解が、ひとつの辺に4個、それに向かい合う辺にも4個、 
> 残りの2辺に5個のピースが接している 

この性質は16×16に限らず
向かい合う辺にn個、もう一組の向かい合う辺にn+1個になる。
また正方形の辺の長さは4の倍数に限る。

177 名前:132人目の素数さん:2008/06/17(火) 09:58:43
なるほど角の4つと、その長さ2の辺がわの隣の4つの
計8個は固定なのか。

178 名前:132人目の素数さん:2008/06/17(火) 17:38:49
集合A⊂Nは、ある自然数Mと、空でないある集合B⊂{1,2,…,M}に対して
A=∪[k=0〜∞]Ak , Ak={b+kM|b∈B}
と書けるとき、周期的であるという。MをAの周期と呼ぶ。
Aが周期的であるとき、Aの周期Mの中で最小のものを、Aの基本周期と呼ぶ。
例:偶数全体の集合は周期的であり、その周期は2m (mは任意の自然数)である。
また、基本周期は2である。

[問題]A⊂Nが周期的であるとする。Aの基本周期をpとおく。もしpが
素数ならば、Aの任意の周期Mについてp|Mが成り立つことを示せ。

179 名前:132人目の素数さん:2008/06/17(火) 19:52:38
>>162
3, 70, 7, 30
5, 42,105,2
210,1,10,21
14,15, 6,35
のとき積が210^2で最小か?

180 名前:Queen ◆xeS.CIM.Jk :2008/06/17(火) 20:31:04
平面上の3個以上の点について条件Aを以下で定める。
条件A:どの2点間の距離も整数値で、どの3点も同一直線上にない。

(1)条件Aを満たす4個の点の配置を考えなさい。

(2)
(1)で配置した4点は動かさず、もう1個点を追加して条件Aを満たす配置を考えなさい。

(3)
(1)で配置した4点は動かさず、もう2008個点を追加して条件Aを満たす配置を考えなさい。

難しくはないがこういうの好きです。

181 名前:132人目の素数さん:2008/06/18(水) 01:06:37
>>145
64個までは解なし
81個だと、7629解見つかった
                  ┏━━━┳━━━┳┓
                  ┣┓┏━┻━┓┏┛┃
                  ┃┗╋┓┏━┻┻┓┃
                  ┃┏┛┗┻┓┏┳┫┃
                  ┃┣┳━━┻┫┃┗┫
                  ┣┛┣┓┏┳┛┃┏┫
                  ┣┓┃┗┫┗┓┃┃┃
                  ┃┃┃┏┫┏╋╋┛┃
                  ┃┗┫┃┃┃┃┗┓┃
┏┳━━━┳━━━┫┏╋┛┣┛┃┏┻┻━┳┳━━━┳┳━━━┳┳━━━┳┓
┃┗┓┏━┻━┓┏┻┛┗┓┣┓┃┣┓┏━┛┗┳┓┏┛┗━┓┏┛┗━┓┏┛┃
┃┏┻┻━┓┏╋┻━━┳╋┛┣╋┛┗┻┳┳━┛┗╋━━━╋╋━━━╋╋┓┃
┃┣┳┓┏╋┛┗━┓┏┛┣┓┃┗┳┳━┛┗┳━━┻┓┏━┛┗┳┓┏┛┃┃┃
┣┛┃┣┛┗━┳━┻┻┓┃┃┃┏┛┗━┳━┻━┓┏╋┻━━┳┛┗┻┓┃┗┫
┣┓┃┗┳┳━┻━┓┏┫┃┗┫┣┳━━┻┳┓┏┻┛┗┓┏┳┻━━┳┫┃┏┫
┃┃┃┏┛┗━┓┏┻┛┗┫┏┻┛┗┓┏━┛┗╋━━━╋┛┗━┓┏┛┗┻┫┃
┃┗╋╋━━━╋╋━━━╋╋━━━╋╋━━━╋┓┏━┻━┳┳┻┻━┳┳┛┃
┃┏┛┗━┓┏┛┗━┓┏┛┗━┓┏┛┗━┓┏┛┗┻┓┏━┛┗┓┏━┛┗┓┃
┗┻━━━┻┻━━━┻┻━━━┻┻━━━┻┻━━━┻┻━━━┻┻━━━┻┛

182 名前:132人目の素数さん:2008/06/18(水) 01:13:16
>>181
すごいな
何がすごいって、ズレない図を描ける事だよ
よくいるじゃねえ?得意げに(そう見える)AA貼ってみせるけどズレててさっぱりな奴が

183 名前:132人目の素数さん:2008/06/18(水) 01:16:07
半角スペースは二つ以上は省略される
どうしても失敗したくないならプレビューできるソフトがある

184 名前:132人目の素数さん:2008/06/18(水) 02:00:58
まぁ等幅だし

185 名前:132人目の素数さん:2008/06/18(水) 08:14:12
この問題は数学的に何か面白い構造があるのかな?

186 名前:132人目の素数さん:2008/06/18(水) 09:50:25
パソコンにぶち込むしかない問題なんて・・・

187 名前:132人目の素数さん:2008/06/18(水) 17:00:46
いやいや、何か数学的な構造があるかもよ。
(1)一辺の長さが全て異なる有限個の正方形を組み合わせて、新たな正方形を作れるか?
(2)一辺の長さが全て異なる有限個の立方体を組み合わせて、新たな立方体を作れるか?
という問題がある。一辺の長さが全て異なるというのがポイントで、この条件を
満たさなくてもよいならば、(1)は「田」みたいなのが答えになり、(2)はルービックキューブ
みたいなのが答えになる。しかし、一辺の長さが全て異なるようにすると、これが難しい。
なんと、(2)は「解なし」になってしまう(これは簡単に証明できる)。さらに驚くべきことに、
(1)には解がある。そして、解があることの証明には、正方形の配置から、対応する
「電気回路(物理の)」が得られ、その電気回路にキルヒホッフの法則を使って…
とかやるらしい( ´д`)
ttp://www-lab15.kuee.kyoto-u.ac.jp/~t-naka/resistor/

ホモロジーみたいなものなのだろうな。

ホモロジー - ある種の図形(多様体と呼ばれる)に対して代数を対応させる行為。(はてなダイアリー)

188 名前:132人目の素数さん:2008/06/18(水) 23:12:28
□に1から9の数字を1個ずつ入れ同一直線上の数の合計が全て同じになる組み合わせは何通りありますか?(全8列)
同じ数字の2度使いは不可(※は無視して下さい

※※※※※※□
※※※※※/
□─□─□
│\│/│
□─□─□
│※※\│
□───□

189 名前:132人目の素数さん:2008/06/18(水) 23:15:49
>>188
コンピュータで解きました、じゃ面白くないぜ。
ちゃんとエレガントな回答を用意してるんだろうな?


190 名前:132人目の素数さん:2008/06/19(木) 22:54:19
      6
     /
1−9−3
│\│/│
7−4−2
│  \│
5−−−8


191 名前:132人目の素数さん:2008/06/19(木) 23:03:10
あ、途中で送っちゃったよ。 
ずれてるけど、ま読めるか。
ちゃんと検証してないけど、これ一通りしかないんじゃないか?

右上の6のとこに来るのは3の倍数だけ、

残りを3,3,2に2通り(縦323と横332)に同じ数に分けられて
さらに/の3つも作れるのは3通りしかない
そのとき\も同じ数にできるのは1通り。




192 名前:132人目の素数さん:2008/06/20(金) 00:00:25
ズレない図を作る方が難しかったわけだな

193 名前:132人目の素数さん:2008/06/22(日) 01:32:21
方程式ax^2+bx+c=0を考える。ただし、a,b,cは整数をとるものとする。
このxについての方程式が実数解をもつ確率をS、虚数解をもつ確率をT、解を持たない確率をUとしたとき、
それらの積STUはいくらになるか。求めよ。

194 名前:132人目の素数さん:2008/06/22(日) 01:36:54
0

195 名前:132人目の素数さん:2008/06/22(日) 01:39:17
同一平面上に2 つの三角形ABC,A'B'C' があり,それぞれの外接円の半径は共に1であるとする.
この2 つの外接円の中心を結ぶ線分の中点をM,線分AA',BB',CC'の中点をそれぞれP,Q,R とする.
(1)  MP ≦1,MQ≦1,MR≦1 となることを示せ.
(2)  もし三角形PQRが鋭角三角形でその外接円の半径が1 となるならば,点Mはこの外接円の中心と一致することを示せ.
さらにこのとき三角形ABC,A'B'C',PQRはすべて合同となることを示せ

196 名前:132人目の素数さん:2008/06/24(火) 08:35:05
2回コインを投げ両方表の時が勝ちなら勝率1/4
では勝率1/3にするにはどうすればいいでしょうか?
その方法を二通り以上求めて下さい

197 名前:132人目の素数さん:2008/06/24(火) 11:42:51
>>196
> では勝率1/3にするにはどうすればいいでしょうか? 

「2回コインを投げて」という条件なのか?
他に条件はあるのか?


198 名前:132人目の素数さん:2008/06/24(火) 23:06:16
サイコロそっくりの形のコインを投げて1または2が出れば勝ちとすればおk

もっときちんと問題を書け

199 名前:132人目の素数さん:2008/06/25(水) 00:25:54
>>196訂正
ここに投げれば表裏が同確率で出るコインが1枚ある
このコイン1枚だけを使って勝率1/4の賭けをするには2回投げ両方表の時に勝ちにすればいい
ではこのコイン1枚だけを使って勝率1/3の賭けをするにはどうすればいいか?
その方法を二通り以上求めて下さい

200 名前:132人目の素数さん:2008/06/25(水) 01:00:34
>>199
有限回で決着しないとダメなのか?

201 名前:132人目の素数さん:2008/06/25(水) 02:20:11
>>199
@投げる前に表1裏2の3通りの組合せから1つ選びコインを3回投げる。
表1、裏2以外だったらやり直すものとして選んだのが当たる確率
A@で表2裏1とした場合

202 名前:132人目の素数さん:2008/06/25(水) 07:42:05
Xを非負整数全体の集合とする。

(1)f:X^2→Xをf(x,y)=[(√2)*x^2]+[π*y^3] (x,y∈X)と定義する。
ただし[ ]はガウス記号とする。このとき、X−Im(f)は無限集合である
ことを示せ。すなわち、
f(x,y)=m
の解(x,y)が存在しないような非負整数mが無限にあることを示せ。

(2)f:X^n → Xをf(x(1),x(2),…,x(n))=Σ[i=1〜n] [a(i)*(x(i))^b(i)]
で定義する。ただしa(i),b(i) (i=1,2,…,n)は正の実数とする。
Σ[i=1〜n]1/b(i)<1ならば、X−Im(f)は無限集合であることを示せ。すなわち、
f(x(1),x(2),…,x(n))=m
の解x(i) (i=1,2,…,n)が存在しないような非負整数mが無限に存在することを示せ。

203 名前:132人目の素数さん:2008/06/25(水) 09:24:33
>>199

コインを裏返しに置く。 (これが0回目の結果とする)
何度もコインを投げる。 
n回目の結果が(n-1)回目の結果と一致したら賭けは終わり。
それが裏なら負け、表なら勝ち。

204 名前:132人目の素数さん:2008/06/25(水) 12:32:38
>>200-201
有限回で決着させないとダメです

205 名前:132人目の素数さん:2008/06/25(水) 13:39:48
201だけど>>201だめなの?なら>>203もだめでは?

206 名前:132人目の素数さん:2008/06/25(水) 16:59:29
だめでしょ

207 名前:132人目の素数さん:2008/06/25(水) 17:02:38
いくら1/2を組み合わせても作れるのはm/2^nばかりで
有限の回数では1/3は作れない。

なにか他の方法を考えないとダメだ。

208 名前:204:2008/06/25(水) 20:27:32
有限回が無理なら決着するまでの回数の平均値が一番低いやつを求めて下さい

209 名前:132人目の素数さん:2008/06/25(水) 21:18:54
出題者答えわかってないのかよw

210 名前:132人目の素数さん:2008/06/25(水) 21:42:25
○○○
○○×  ☆
○×○  ☆
○××  ★
×○○  ☆
×○×  ★
××○  ★
×××

>>201の@A両方を組み合わせた形
表1、裏2と表2、裏1の両方から一つずつ選ばせておく方法

1回でも3/4の確率で決着がつく

10回以内で決着がつく確率は0.99999904632568359375


211 名前:210:2008/06/25(水) 21:53:56
あ、今思いついたけど
○○○
×××
このハズレの場合でも同様に3回ずつで仕切れるからもう少し早く決着できそうだな。

212 名前:132人目の素数さん:2008/06/25(水) 22:16:51
3^n回目でn回ハズレ抽選できる

213 名前:132人目の素数さん:2008/06/26(木) 00:56:36
>>210

> 1回でも3/4の確率で決着がつく 

いやそれ3回振ってるじゃん。

もしそれを1回と認めるなら
2n回コインを振って、その結果を表を0裏を1の
二進数とみなし0〜2^(2n)-1の値を決め
それが0だったらやり直し
0以外なら、3で割った余りが0なら勝ち、0以外なら負け
というルールにしてnを大きくすれば
一度で決着のつく確率をいくらでも大きくできる。


214 名前:132人目の素数さん:2008/06/26(木) 01:03:41
ちなみに>>203の方法なら
決着が付くまでにコインを投げる平均回数は
Σ_{n=1→∞}(n/2^n) = 2 回

215 名前:132人目の素数さん:2008/06/26(木) 01:11:42
┌-┬-┬-┐
│1│2│3│
│4│5│6│
└-┴-┴-┘
好きな桝目にコインを置き
サイコロを振る。

コインを振る平均回数は0回

216 名前:132人目の素数さん:2008/06/26(木) 01:48:25
コイン「だけ」を使い・・・

217 名前:132人目の素数さん:2008/06/26(木) 04:36:19
>>216
そのコインは一枚だけしかつかってない

218 名前:132人目の素数さん:2008/06/26(木) 10:38:05
とんち問題なのか?

219 名前:132人目の素数さん:2008/06/26(木) 14:48:41
うんち問題だろう

・回答は用意されていない
・にもかかわらず2種以上の回答を要求する
・有限回でないとならないはずが、なるべく短い無限回になる。

>>213 やり直し回数は限りなく0回に近い
>>214 おそらく平均最小
>>215 振るのは0回

もうこれらでFAでないか?
あとは>>214の最小の証明くらい?




220 名前:132人目の素数さん:2008/06/27(金) 19:29:06
Aは一分間に3個の皿を洗い,Bは一分間に2個の皿を洗います
またAは一分間に9個のコップを洗い,Bは一分間に7個のコップを洗います
ここに汚れた皿とコップが合わせて134個あります
A,Bの二人が協力して,20分で全部洗い終えました
コップは何個,皿は何個あったのでしょうか?


221 名前:132人目の素数さん:2008/06/28(土) 08:48:45
二人ともきちんと洗う気ないな
どれだけ適当に洗ってるんだ

それとも居酒屋とかではコレくらいのスピードでやらないと追いつかないのかね

222 名前:132人目の素数さん:2008/06/28(土) 10:45:45
よほどの大皿でもない限りむしろ遅い。
遅すぎるくらいである。


223 名前:132人目の素数さん:2008/06/28(土) 11:09:06
自動食器洗いを導入してAとBに別の仕事をさせるか解雇する。無駄な労働と人件費を節約するべきだ。
それにコップと皿を分けるなんてナンセンス。洗いとすすぎを分担した方が効率は良いだろう。

224 名前:132人目の素数さん:2008/06/28(土) 11:21:37
皿は一枚二枚だよなあ

225 名前:132人目の素数さん:2008/06/28(土) 15:04:43
コップと皿を分けてなどいない
>>223


226 名前:132人目の素数さん:2008/06/28(土) 15:11:12
コップと皿を洗う個数について分担しているでしょ。

227 名前:132人目の素数さん:2008/06/28(土) 15:22:08
>>226

「個数について分担」というのはどういう意味だ?

228 名前:132人目の素数さん:2008/06/28(土) 16:38:32
くもはえ算ならぬ食器洗い算

229 名前:132人目の素数さん:2008/06/29(日) 01:13:13
>>228
早食い算

>>220
> Aは一分間に3個のポテトチップを食べ、Bは一分間に2個のポテトチップを食べます
>またAは一分間に9個のピーナッツ゚を食べ,Bは一分間に7個のピーナッツを食べます

230 名前:132人目の素数さん:2008/06/29(日) 01:35:56
> ピーナッツ゚

231 名前:132人目の素数さん:2008/06/29(日) 12:57:54
下の様に16個の点が4×4の格子状に並んでいます.各点の縦横の間隔は1です
16個の点を全て通る一筆書きの線を引いて下さい
ただし曲がる回数は5回にし、線の長さも出来るだけ短くなる様にして下さい
また、その時の線の長さを小数点以下第二位まで求めて下さい

・ ・ ・ ・

・ ・ ・ ・

・ ・ ・ ・

・ ・ ・ ・

232 名前:132人目の素数さん:2008/06/30(月) 17:18:17
少数点第2位ってなんだよ?
まさか代数的数にならないの?



233 名前:132人目の素数さん:2008/06/30(月) 19:50:00
短そうなのは思いつくが
それが最小であることの証明がうまくいかん。


234 名前:132人目の素数さん:2008/06/30(月) 20:14:36
12+6√2より短いのってできる?

235 名前:132人目の素数さん:2008/06/30(月) 21:34:54
13+5√2がいけるかも


236 名前:132人目の素数さん:2008/06/30(月) 23:10:02
>>235
線の引き方は?

237 名前:132人目の素数さん:2008/06/30(月) 23:23:33
座標(0,0)〜(3,3)に点があるとして

(0,3)→(0,0)→(3,0)→(3,4)→(0,1)→(3,1)→(1,3)

238 名前:132人目の素数さん:2008/07/01(火) 00:03:18

            /┃
          /  ┃
●   ●   ●   ●
┃    \ /    ┃
┃    /\     ┃
●   ●   ●   ●
┃ /      \  ┃
┃/        \ ┃
●━━●━━●━━●
┃            ┃
┃            ┃
●━━●━━●━━●

239 名前:132人目の素数さん:2008/07/01(火) 01:15:52
>>202 (2)
納m∈Im f\{0}] 1/m が収束することを示せばよい。それには
1/f(x(1),…,x(n))  (和は f(x(1),…,f(n))≠0であるようなすべての(x(1),…,x(n))∈X^nにわたる)
が収束することを示せばよい。f(x(1),…,x(n))≧1 のとき
(n+1)*f(x(1),…,x(n)) ≧ f(x(1),…,x(n)) + n ≧ 納i=1〜n] a(i)*(x(i))^b(i) だから、
右辺の和をg(x(1),…,x(n)) とおくと、結局
1/g(x(1),…,x(n))  (和は x(1)=…=x(n))=0以外のすべてのX^nの元にわたる)
が収束することを示せばよい。それには、g(x(1),…,x(n))>0 のとき、不等式
g(x(1),…,x(n)) ≧ A*{Π[x(i)≠0] x(i)}^B (☆)
(ただし、Aはx(1),…,x(n)によらない正定数、1/B=納i=1〜n] 1/b(i) )
が成立することを示せばよい。そうすれば、条件よりB>1だから、
1/g(x(1),…,x(n)) ≦ (1/A)*(1+ζ(B))^n
であることが示される。

p,q,s,t,y,zを正の実数とするとき、対数関数が上に凸であることから
log(p*y^s + q*z^t) - log(s+t) ≧ {t*log((p/t)*y^s) +s*log((q/s)*z^t)}/(s+t)、
したがって
p*y^s + q*z^t ≧ (s+t) * (p/t)^(t/(s+t)) * (q/s)^(s/(s+t)) * (yz)^(1/(1/s + 1/t))。
これから帰納的に、(p(1),…,p(k),s(1),…,s(k),y(1),…,y(k)を正の実数とするとき)
納i=1〜k] p(i)*(y(i))^s(i) ≧ C*{Π[i=1〜k] y(i)}^(1/(納i=1〜n] 1/s(i)))  (Cはy(1),…,y(k)によらない正定数)。
いまn個あるa(i)*(x(i))^b(i)から上のような不等式が2^n個できるが、それらの右辺の定数Cで最小のものをAとする。
また、右辺の指数のうち最小のものはB=1/(納i=1〜n] 1/b(i)) である。これから不等式(☆)が成り立つ。

240 名前:132人目の素数さん:2008/07/01(火) 02:44:02
>>199
方法1:部屋の中でコインを転がすとか放り投げるって方法は?
 @止まって表を向く
 A止まって裏を向く
 B壁で斜めに止まって表か裏かわからない
(ないしは部屋の中にあるモノの中や下に入り込んでわからなくなるとか…)
要はごきげん○うのサイコロ方式ねw
ただこれだとBが1/3よりかなり低そうかな^^;

方法2:コインをひん曲げて(横から見てL字型とか、かなり反則?)トス。
(手で受けると下の@かAだけになるのでこの場合は地面やテーブルに落とす形で)
 @凸型に止まる。
 A凹型に止まる。
 Bコインが立って止まる。
  
今日問題初見で、こちらもかなり遅レスだけど、
そろそろ正解出してもいいんじゃ?>>199

241 名前:132人目の素数さん:2008/07/01(火) 09:35:04
Xを非負整数全体の集合とし、Yを非負実数全体の集合とする。1≦i≦nに対して、
f i :Y → Yは狭義単調増加でf i (y)→+∞ (y→+∞)が成り立つとする。また、

limsup[y→+∞] { g1(y)*g2(y)*…*gn(y) } /y<1

が成り立つとする。ただし、g i :Y → Y はf i の逆関数とする。
[ ]をガウス記号として、F:X^n → Xを

F(x(1),x(2),…,x(n))=Σ[i=1〜n] [f i (x(i))]

で定義する。このとき、X−Im(F)は無限集合であることを示せ。すなわち、
F(x(1),…,x(n))=mの解x(i) (i=1,2,…,n)が存在しないような非負整数mが
無限に存在することを示せ。

242 名前:132人目の素数さん:2008/07/01(火) 13:00:39
>>37
あたまいいな

243 名前:132人目の素数さん:2008/07/01(火) 18:12:30
>>242
ホントだ

244 名前:132人目の素数さん:2008/07/02(水) 12:08:24
>>199
>>37の考え方を使うと、両手足のうちから3つ決めてどれかにコイン持つ又は踏んで、3つのうちどこにコインがある?と聞く。


245 名前:132人目の素数さん:2008/07/02(水) 13:18:58
>>244
215 と 本質的にどこが異なるのだ?

246 名前:132人目の素数さん:2008/07/02(水) 14:18:48
>>245
>>215はマス目をつくるってのがマズくね?
結局同じことだけど。

247 名前:132人目の素数さん:2008/07/02(水) 14:27:57
手や足ならかまわないのか?

本質的な違いがあるとしたらコインの用途。
コインで示すことと、コインを探すことの違い。

248 名前:132人目の素数さん:2008/07/02(水) 14:43:16
>>244
確率1/3に出来ていることが証明できない。
どこに持つ(踏む)をどうやって決めるのか?

249 名前:132人目の素数さん:2008/07/02(水) 15:51:22
どうやって決めても、それを当てる人に知られていなければ問題ない。

250 名前:132人目の素数さん:2008/07/02(水) 16:03:00
>>249
んなこたねえよ

251 名前:132人目の素数さん:2008/07/02(水) 16:04:37
>>249
それでOKなら、選択肢が三つならなんでもOKになるぞ。

252 名前:132人目の素数さん:2008/07/03(木) 02:12:10
期待値は1/3だが
実際に当たる確率が1/3になるとは限らないということかな?

253 名前:132人目の素数さん:2008/07/03(木) 02:15:08
同様に確からしくないんだろ

254 名前:132人目の素数さん:2008/07/03(木) 03:59:24
何が?

255 名前:132人目の素数さん:2008/07/03(木) 04:32:27
>>250,251
なぜ? 
くじを引く(右手左手足を選ぶ?)人が
どれが当たりやすい当たりにくいとかの情報を
持っていなければ同じではないのか?



3枚の封筒のうちひとつにあたりを入れる。
あたりを入れる封筒を決めるルールは、プレイヤーには知らされていない。

さて、目の前に封筒が3枚並べられた。 プレイヤーはどの封筒を選んでもよい。

このくじの当たる確率は1/3ではないのか?


256 名前:132人目の素数さん:2008/07/03(木) 05:09:40
1/3と答えたとして誰かがニヤニヤしたとしよう
それでも君は1/3と断言できるか?

257 名前:132人目の素数さん:2008/07/03(木) 06:07:22
1/3の確率で当たるくじと、当たる期待値が1/3のくじとは
ちがうという立場だな?

258 名前:132人目の素数さん:2008/07/03(木) 08:28:38
おそらくね、くじを引く人がどんな作戦を立てようが
次に引くくじも変わらず当たる確率は常に1/3という保障がないと、
つまり、過去の100回の結果が100回連続して右手にコインが入っていようと
次の回にも右手にコインが入っている可能性はやはり1/3だと言えるようなものでないと
題意を満たさないと考える人がいるということだと思う。

では、>>256の封筒のくじがその題意を満たさないのか?といえば
過去の100回がいつでも一番右の封筒にあたりが入っていたとしたら
次の回は、やっぱり右の封筒に入っている可能性が高いのだろうか
それともそんなことは決してなく、やはり1/3なのだろうか? よくわからない。

この「よくわからない」を、「保証がないのだから題意を満たしていない」と考えるのか
「わからない以上は予測が立たないのだから題意を満たしている」と考えるのか

私にはどちらが正しいのかよくわからない。


259 名前:132人目の素数さん:2008/07/03(木) 08:30:39
>>255
> くじを引く(右手左手足を選ぶ?)人が
> どれが当たりやすい当たりにくいとかの情報を
> 持っていなければ同じではないのか?

それを言うなら、そもそも
いびつなコインを使う場合でも
当てようとする人が表裏どっちが出やすいかの情報を持っていなければ
同じ理由で1/2になってしまう

なので、解答として題意に沿っていないと思う


260 名前:132人目の素数さん:2008/07/03(木) 08:51:29
>>259
前半は同意。情報を持っていなければ1/2だと思う。

後半は現状では不同意。
なぜそれだと題意に沿っていないのかを詳しく論じてほしい。


261 名前:132人目の素数さん:2008/07/03(木) 08:58:03
題意にそぐわないというひとは、ぜひ>>255の後半の封筒のくじは
なぜ題意にそぐわないのかについても論じてほしいな。

コインを使ってないからとか、封筒を使ったからというのは無しで

自分はどちらなのかがわからない。

262 名前:132人目の素数さん:2008/07/03(木) 20:48:56
>>261
・あたり封筒を決める人のクセ(あたり封筒を置くときちょっとキョドってしまう等)
・あたり封筒の外見(厚さ、透けて見える等)

などなどが、厳密には排除できないので
確率が厳密に1/3というわけではないでしょう。

263 名前:132人目の素数さん:2008/07/03(木) 21:00:39
>>262
マジレスさせてもらうが、不毛な難癖はやめにしようぜ。
正直そういうのはもう飽きてる。

264 名前:132人目の素数さん:2008/07/03(木) 21:17:30
ふむ。題意の捉え方の違いだな。
>>32>>263のいう「不毛な議論」を回避するにはどうすれば良いか、という問題だと思った


265 名前:132人目の素数さん:2008/07/03(木) 21:21:49
もしかしてゲーム理論が関係してくるのかな。
しっぺ返し戦略とか。


266 名前:132人目の素数さん:2008/07/03(木) 21:27:30
相手がなんらかの戦略に基づいてゲームをプレイするがその戦略は知らされていない。
そのとき勝率を上げることはできるか?ということだよね。


267 名前:132人目の素数さん:2008/07/03(木) 21:48:01
封筒のゲームの場合、プレーヤーとディーラーの戦略によっては
長期の勝負をしても勝率が1/3に収束しないことがありうる。
ディーラーの戦略はプレーヤーの戦略にかかわらず、
長期の勝負で勝率が1/3に収束する必要がある。
てことでしょ。


268 名前:132人目の素数さん:2008/07/04(金) 00:23:36
封筒の問題は>>216でFAでないの?
コイン以外使うな、っていう…

269 名前:132人目の素数さん:2008/07/04(金) 02:50:57
>>268
封筒を使わなくても同じことはできるだろ。

270 名前:132人目の素数さん:2008/07/04(金) 06:31:08
ディーラーのとり得る戦略に制限がなく、
有限回の勝負でプレーヤーがディーラーの戦略を特定できないのであれば、
プレーヤー側が確実に勝率を1/3以上に上げることはできない。

しかし有限回の勝負でプレーヤーがディーラーの戦略を特定できないという仮定は
ディーラーがランダムにプレーしていると仮定するのと等価ではではないのか?

識者の意見求む。


271 名前:132人目の素数さん:2008/07/04(金) 13:22:51
「有限回で特定できない」というのは
「いかなる仮定も間違っている」
ということなのじゃないかな?
だとしたらそれはランダムと何も変わらないと思うが…

単語の意味が曖昧なのですこし補足しておくと
「特定」は、ディーラーの戦略全体を知る必要はなく、
どんな小さな癖でもひとつ掴めばいい。
当然だが「仮定」は、出目の偏りがあると
仮定するものである必要がある
(「出目はランダムだ」という仮定は当たっていても意味はない)

「仮定が間違っている」というのは、
「そのような偏りはなかった」というのに他ならない。
なぜなら、ある偏りがあると仮定すると同時に
その逆の偏りがあると仮定できるからだ。

結局、仮定できるすべての偏りについて間違っているなら
それは偏りのない(つまりランダム)と等しいと考える。



272 名前:132人目の素数さん:2008/07/04(金) 13:34:14
> 「仮定が間違っている」というのは、 
> 「そのような偏りはなかった」というのに他ならない。 

ここちょっとわかりにくいな…
ある偏りを仮定したときに、その逆の偏りも仮定できるので
出目に正の偏りも負の偏りも許されないということ。



273 名前:132人目の素数さん:2008/07/04(金) 13:42:55
あー、でも、それと
有限回の勝負でその偏りを見つけられるかどうかとは関係ないのかな?

「偏りは、有限回の勝負で必ず見つけられなければならない」ということなら
有限回の勝負では見つけられない偏りが存在するディーラーの戦略
というのは、あるのかもしれない…
わからなくなってきた…

274 名前:132人目の素数さん:2008/07/04(金) 18:27:29
もし出目に偏りがあるならプレーヤーは今までで一番でた回数の多い目にかけ続ければ最終的には勝てるんじゃない?
ディーラーの戦略は偏りが無いのはもちろんのこと、それ以上のものが求められるかと。



275 名前:132人目の素数さん:2008/07/04(金) 19:34:06
てめーら期待値で考えろよ

276 名前:132人目の素数さん:2008/07/04(金) 19:53:48
期待値なんてものは同様に確からしい何かを前提とした議論だろ。
ディーラーの戦略がその前提を満たして無くても賭けが成り立つのか?というのが問題になってるんだろうが。




277 名前:132人目の素数さん:2008/07/04(金) 20:02:56
276 :132人目の素数さん :2008/07/04(金) 19:53:48
期待値なんてものは同様に確からしい何かを前提とした議論だろ。
ディーラーの戦略がその前提を満たして無くても賭けが成り立つのか?というのが問題になってるんだろうが。

278 名前:276:2008/07/04(金) 21:26:52
俺の主張はこうだ。

封筒のゲームはディーラーがどうであれ、最初の一発目のゲームの期待値は1/3になる。
なぜなら、プレーヤーがなんの情報も持っていないから。

しかし繰り返しゲームを行ったとき、プレーヤーは過去の出目の情報を持ってしまっている。
この情報を全くの無価値にするにはディーラーの戦略がランダムであらねばならない。

つまり、繰り返しゲームを行うとき、ディーラーの戦略がどんなものであれ、ゲームの期待値が1/3になるとは思えない。

こんなたとえ話はどうだ。

あなたはカジノのオーナーです。
今回、ルーレットマシンを購入することにしました。
A社のルーレットマシンは出目が完全にランダムであることが保障されています。
B社のルーレットマシンは出目があるアルゴリズムによって決定されますが、
B社の機密保持は完璧でアルゴリズムがプレーヤーに漏れることはありません。
しかし、プレーヤーは過去の出目を参考に戦略を立ててくるかもしれません。
あなたはB社のルーレットマシンを購入しますか?







279 名前:132人目の素数さん:2008/07/04(金) 22:13:28
276 :132人目の素数さん :2008/07/04(金) 19:53:48
期待値なんてものは同様に確からしい何かを前提とした議論だろ。
ディーラーの戦略がその前提を満たして無くても賭けが成り立つのか?というのが問題になってるんだろうが。

280 名前:132人目の素数さん:2008/07/04(金) 22:39:45
ナッシュ均衡

281 名前:132人目の素数さん:2008/07/05(土) 03:55:24
>>274
出目に偏りがあるというのは、どれかが多いというだけではない。

たとえば、封筒の例で言えば、「あたりを入れる封筒は右→左→中の順の繰り返し」
というディーラーの戦略だとしたら、どれかの出目が多いとということはなくなる。

しかしこのディーラー戦略には、前のあたりと次の出目の関係に強い偏りがあるので
多くのプレイヤーが簡単に勝つことが可能になってしまうだろう。

282 名前:132人目の素数さん:2008/07/05(土) 03:57:12
なんかよほど>>276の言葉に傷心しちゃったやつでもいるのか?

283 名前:132人目の素数さん:2008/07/05(土) 04:00:40
>>278
そのたとえ話はあまり役に立たんと思う。

いったいどこのカジノが、完全にランダムに出るルーレットを持っているんだ?

どこのカジノも、出目のランダムでないルーレットを使って商売をしているのは
ルーレットのルールが、完全なランダムなど要求しなくても
ディーラーが勝てるように設計されているからだよ。

284 名前:132人目の素数さん:2008/07/05(土) 06:01:01
そもそもランダムってちゃんと数学的に定義された言葉なのか?


285 名前:278:2008/07/05(土) 08:33:06
>>283
ルーレットのルールが多少ディーラーに有利になっていてもプレーヤーに出目がばれていればディーラーは負ける。
完全なランダム性は要求されないにしても、実用的な時間で出目がばれない程度のランダム性は必要だろう。
カジノのオーナーとしてB社のアルゴリズムの安全性が気になりませんか?ということだ。

それはすなわち、

「ディーラーの戦略がどんなものであれゲームの期待値が1/3になる、とは思えない。」
(ちょっと句点の位置をずらさせてもらった。)

という主張と一致する。


たとえ話から主張するところが読み取りにくいというならそうかもしれない。


286 名前:132人目の素数さん:2008/07/05(土) 08:39:27
>>284
「同様に確からしい」とほぼ同意語ということでいいんじゃないか?

287 名前:132人目の素数さん:2008/07/05(土) 08:45:15
>>285

> 完全なランダム性は要求されないにしても、実用的な時間で出目がばれない程度のランダム性は必要だろう。 
> カジノのオーナーとしてB社のアルゴリズムの安全性が気になりませんか?ということだ。 

つまり、それなりにバラけていれば、完全にランダムである証明など要らないということになってしまうので
その例は主張したいことと異ならないか?

今要求されているのは、実用的なランダム性ではなく
両手片足のどれにコインが握られて(踏まれて)いるかを決めルールが提供されていて
かつそれが厳密に1/3だと証明されていることだと思うのだが。

というわけで、主張するところが読み取りにくい。

288 名前:284:2008/07/05(土) 16:07:16
同様に確からしい、という概念自体、わかっているようでよくわからない。

さいころを振ったとき1〜6がでることが同様に確からしいというのはどういうことか?
それぞれが出る確率が1/6ということか?
ならばそれぞれが出る確率が1/6とはどういうことか?
1〜6がでることが同様に確からしいということか?

これでは循環定義で何の説明にもなっていない。

では過去の出目が未来の出目に全く影響を与えないという定義はどうだろうか?
この場合「全く影響を与えない」ということの定義がよくわからない。
ある数列が与えられたときに、この数列の項は互いに全く影響を与えていない、
あるいは何らかの関係があるということを定義することができるのか?

そもそも同様に確からしいという概念自体公理であって他の公理から定義することは無理なのか?







289 名前:132人目の素数さん:2008/07/05(土) 16:41:25
独自研究はいいから本を読んでみたまえ

290 名前:132人目の素数さん:2008/07/06(日) 00:20:21
>>241
Nを、十分大きな正整数とする。
0≦F≦Nならば、すべてのi (1≦i≦n)について
0≦[f_i(x_i)]≦N、よって 0≦f_i(x_i)<N+1 だから 0≦x_i<g_i(N+1)。
したがって、0≦F≦Nであるような(x_1,…x_n)∈X^n の個数は
高々 [g_1(N+1)+1]*…*[g_n(N+1)+1]であり、これはもちろん {g_1(N+1)+1}*…*{g_n(N+1)+1}以下。
以上より、0≦m≦Nで、Fで表現できる m の個数は{g_1(N+1)+1}*…*{g_n(N+1)+1}以下である。

条件より、ある正定数 K, L (L<1)が存在して、N>K なら g_1(N+1)*…*g_n(N+1)<L^2*(N+1)。
また、すべてのi (1≦i≦n)について、g_i(y)→+∞ (y→+∞)だから、
Kを十分に大きくとれば、N>Kのとき {1+1/g_1(N+1)}*…*{1+1/g_n(N+1)}<1/Lとなる。

P(N)を、0≦m≦Nで、Fで表現できない m の個数とすると、N>Kなら
P(N)≧(N+1)-[g_1(N+1)+1]*…*[g_n(N+1)+1]
 >(N+1)-{g_1(N+1)*…*g_n(N+1)}/L
 >(1-L)*(N+1)
であるから、P(N)→+∞ (N→+∞)。

291 名前:132人目の素数さん:2008/07/07(月) 07:24:28
確率の定義から堂々巡りしてる男の人って・・・

まあ普通の計算問題を解いてる分にはあんまりボロが出ないのかもしれんが
教科書の最初の方だろ

292 名前:132人目の素数さん:2008/07/07(月) 08:48:53
高校の教科書の最初には定義なんてありませんよ

293 名前:132人目の素数さん:2008/07/09(水) 00:55:14
1等賞金支払い後も「くじ」販売、契約違反と州を提訴 米国
http://www.cnn.co.jp/usa/CNN200807080024.html

>くじの1等賞金を得る確率が非常に低いこととゼロでは、まったく意味が違うと主張

なるほど、提訴は正しい

294 名前:132人目の素数さん:2008/07/16(水) 12:56:46
22×26の長方形を16個の正方形に分割する方法は何通りあるか?
正方形の一辺は整数に限る

295 名前:132人目の素数さん:2008/07/17(木) 23:54:42
□□□□□□□
□◎◎◎◎◎□
□◎◎◎◎◎□
□◎◎◎◎◎□
□◎◎◎◎◎□
□◎◎◎◎◎□
□□□□□□□

以下の条件で上図の25個の◎を取り除いて下さい
(1)◎は他の◎を1つだけ飛び越して縦横斜めに移動できる(移動先に◎がある場合は不可)
(2)飛び越された◎は取り除かれる
(3)最後に◎が1つだけ中央に残る
(4)最短手数で取り除く
(5)1つの◎が連続して飛ぶ場合は一手と数える

296 名前:132人目の素数さん:2008/07/20(日) 11:41:38
ABCD×EFGH=DFFDIDGH

同じ文字に同じ数字、違う文字に違う数字を入れ等式を成立させて下さい

297 名前:132人目の素数さん:2008/07/20(日) 12:03:57
正12面体の頂点を赤、白で塗る塗り方の数はいくつでしょうか?
回転して同じになる塗り方は同じとします。

298 名前:132人目の素数さん:2008/07/22(火) 20:56:53
>>297
17824

299 名前:132人目の素数さん:2008/07/22(火) 21:16:03
>>296
4781 * 2095 = 10016195
5601 * 2378 = 13319178

300 名前:132人目の素数さん:2008/07/23(水) 00:22:55
正多角形(何でも良い)を用いて3.1<π<3.2を証明せよ

301 名前:132人目の素数さん:2008/07/23(水) 02:00:50
面白そうだったので「わからない問題はここに書いてね 676」スレより転載

数列a[n]=cos((n-1)π/2)がある。a[n]を次のように並べて、群の中の項数が1ずつ増えていく群数列b[n]を考える。

b[n]=a[1],|a[2],a[3],|a[4],a[5],a[6],|a[7],a[8],a[9],a[10],|a[11],a[12],・・・・|

(1)第n群の初めの項b[n,1]をnで表せ。
(2)第n群のm番目めの項b[n,m]をnとmで表せ。(m≦n)
(3)第n群に含まれるすべての項の和を求めよ。
(4)b[100,50]からb[200,100]までの和を求めよ。

質問者は(3)以外はわかったらしい。俺は(1)(2)しかわからなかったorz
件のスレに(3)の解法も載ってるのだが、それを読んでなお理解できなかったよ・・・
その質問者もその後、理解できたのかどうか音沙汰が無いしなあ

302 名前:132人目の素数さん:2008/07/26(土) 22:04:43

│\
◎─◎
│\│\
◎─◎─◎
│\│\│\
◎─◎─◎─◎
│\│\│\│\
◎─◎─◎─◎─◎

以下の条件で上図の◎を取り除いて下さい
(1)最初に◎を1つ取り除く
(2)◎は線にそって他の◎を1つだけ飛び越して移動できる(移動先に◎がある場合は不可)
(3)飛び越された◎は取り除かれる
(4)最後に◎が1つだけ残る
(5)最短手数で取り除く
(6)1つの◎が連続して飛ぶ場合は一手と数える

303 名前:132人目の素数さん:2008/07/27(日) 11:16:15
>>301
a[n]={i^(n-1)+(-i)^(n-1)}/2と書けば、等比数列の和の公式から
n群の和=納k=n(n-1)/2+1,n(n+1)/2]a[k]は書けることはかける・・・
あとはn=4m+sっておいて、s=0,1,2,3で場合わけでいけるんじゃない?

304 名前:132人目の素数さん:2008/07/31(木) 08:18:37
AからGに正の整数(何桁でも可)を入れて等式を成立させて下さい.同じ整数の2度使いは不可
値が最も小さくなるものを答えて下さい
Aの3乗=Bの3乗+Cの3乗−Dの3乗=Eの3乗+Fの3乗−Gの3乗

305 名前:132人目の素数さん:2008/08/01(金) 19:46:59
微分の応用問題
V cm3 の水を入れると,深さが3√ (V^ 2)cm(Vの2乗の3乗根) になる容器がある。この容器に,毎秒一定の割合で水を注ぎ入れると
き,水面の上昇する速度は,水の深さの平方根に反比例することを証明せよ。
これどうなりますかね??よろしくお願いします!!

306 名前:132人目の素数さん:2008/08/01(金) 20:42:26
ガソリンが200円になったら需要が激減して、原油先物市場が暴落する。
それまでの原油価格の変動曲線を計算しなさい。

307 名前:132人目の素数さん:2008/08/01(金) 20:42:42
>>305
容器の形状に依るのでは?

308 名前:132人目の素数さん:2008/08/01(金) 20:49:04
>>305
応用ってか、微分するだけじゃねえの?

309 名前:132人目の素数さん:2008/08/01(金) 21:45:31
レスの時刻をフーリエしなさい

310 名前:132人目の素数さん:2008/08/01(金) 22:02:58
面白半分で数学スレに来たんだが、おまえら、いや貴方たちすごいわ、

311 名前:132人目の素数さん:2008/08/02(土) 06:06:07
>>307
条件を満たす容器は形状によらず一定にならないか?

312 名前:132人目の素数さん:2008/08/02(土) 08:35:50
>>305
毎秒一定の割合で水を注ぎ入れる
ってことから、dV/dt=a (aは正の定数)とおける
また、体積Vの水が入ってるときの深さをhとおくと、
h=V^(2/3)
水面の上昇する速度はhをtで微分したものだから
dh/dt=(dh/dV)*(dV/dt)=(2a/3)*V^(-1/3)=(2a/3)*(1/√h)

って感じでいいんじゃね?
あと、条件を満たす容器の形状は一つに決まらないと思うのだが
ラッパ型になるのは分かるが、円が積み重なったラッパ型も
正方形が積み重なったラッパ型等も考えられる


313 名前:132人目の素数さん:2008/08/04(月) 16:49:00
適当にいじって変形しただけで特に背景とか脈絡とかはない問題。
問.計算せよ。

  ∞   1
(1) ――――
  n=1 n*2^n

  ∞      1
(2) ――――――――
  n=0 n!*(2n+1)(2n+3)

  ∞  16n^2+28n+11
(2) ――――――――
  n=0    (4n+4)!

314 名前:132人目の素数さん:2008/08/04(月) 16:51:08
問題を間違えた・・・

  ∞     6n+5
(2) ――――――――
  n=0 n!*(2n+1)(2n+3)
です。


315 名前:132人目の素数さん:2008/08/04(月) 17:43:15
それは面白いの?

316 名前:132人目の素数さん:2008/08/04(月) 19:22:16
>>313
(1) log(2)
(2) e
(3) 問題間違えてないか?初等関数で表せない気がする。

317 名前:132人目の素数さん:2008/08/04(月) 20:03:28
>>316
すばやいですね〜。(1)も(2)も当たりです。
(3)も初等関数を用いたシンプルな値になりますよ。
今googleで検算しましたが、n=1で既に相当近い値になりますね。

  ∞  16n^2+28n+11
(3) ――――――――
  n=0    (4n+4)!

318 名前:132人目の素数さん:2008/08/04(月) 20:26:53
>>317
1 - cos(1)

やっぱり面白くねえな。

319 名前:132人目の素数さん:2008/08/05(火) 19:24:55
微分の問題ありがとうございました!!助かりました

320 名前:132人目の素数さん:2008/08/05(火) 22:39:46
tan(π/24)の値を求めよ。

そんなに難しいわけじゃないけど、
結果は面白いと思うんだ。

321 名前:132人目の素数さん:2008/08/06(水) 08:30:17
tan(x) = sin(x)/cos(x),
sin(π/24) = (√(2-√(2+√3)))/2,
cos(π/24) = (√(2+√(2+√3)))/2,
これらを使って、
tan(pi/24) = (√(2-√(2+√3)))/(√(2+√(2+√3)))
分母分子に√(2-√(2+√3))を掛けると
(分子) = 2-√(2+√3)
(分母) = √(4-(2+√(3))) = √(2-√3)
さらに分母分子に√(2-√3)を掛けて
(分子) = (2-√(2+√3))√(2-√3)
      = 2√(2-√3) - √(4-3)
      = 2√(2-√3) - 1
(分母) = 2-√3
更に分母分子に2+√3を掛けて
(分子) = (2√(2-√3) - 1)(2+√3)
      = 4√(2-√3) - 2 + 2√(3)√(2-√3) - √3
(分母) = 1
あとは分子をなるべく簡単な形にする。
(与式) = √(2-√3)(4+2√3) - (2+√3)
      = (2+√3)(2√(2-√3) - 1)
ここで、2√(2-√3) = √(8-2√12) = √((√(6)-√(2))^2) = √(6)-√(2) より
(与式) = (2+√3)(√6 - √2 - 1)
      = 2√6 - 2√2 - 2 + 3√2 - √6 - √3
      = √2 - √3 + √6 - 2
答え. tan(π/24) = √2 - √3 + √6 - 2
これ以上簡単に出来るかどうかはわかりませんでした。

322 名前:320:2008/08/06(水) 11:06:56
>>321
合ってます。これ以上簡単にできるか?という点ですが、
√2-√3+√6-2=(√3-√2)(√2-1)
と変形できます(どちらが簡単かは好みの問題だと思いますが)。

sin(π/24)やcos(π/24)は二重根号が現れるのに対し、
tan(π/24)は分数にもならず意外ときれいな形で求められるところが
個人的に面白いと思いまして。

323 名前:132人目の素数さん:2008/08/06(水) 18:38:09
>>322
じゃあtan(π/48)はどうなの

324 名前:320:2008/08/07(木) 00:25:42
>>323
な、なんと!その質問は予想できなかった。
ちょっと計算してみます。

tan^2(x)=(1-cos(2x))/(1+cos(2x))
の公式で求めることができるのですが、
いかんせんcos(π/24)の値が二重根号入っているので、
ためらってしまう……。

325 名前:132人目の素数さん:2008/08/07(木) 01:58:52
腕力の訓練だな.若干の計算によって,
a = 2+√3, b = 2+√a, c = 2-√b とおいて
tan(π/48) = a b c^2 となる.この多重根号は外れない.

326 名前:132人目の素数さん:2008/08/07(木) 18:26:06
一辺が1の正四面体OABCにおいてOA、OB、OC上に点P、Q、Rが
四面体OPQRの体積が正四面体OABCの1/3になるように動く。
このとき三角形PQRの周および内部が通過する領域の体積を求めよ。

327 名前:132人目の素数さん:2008/08/07(木) 21:38:07
>>326
http://science6.2ch.net/test/read.cgi/math/1217503374/
こっちで終了してる

328 名前:132人目の素数さん:2008/08/07(木) 23:16:42
直径5の円の中に10個の点をどのように取っても必ず互いの距離が
2より小さい2個の点があることを示せ

使う道具はわかるけどどう使うかに苦慮する問題です

329 名前:132人目の素数さん:2008/08/07(木) 23:25:26
>>328
直径2の同心円、およびその外側の領域を放射状に8等分でどうよ?

330 名前:132人目の素数さん:2008/08/07(木) 23:43:19
>>329
正解です。頭いいですね。

331 名前:132人目の素数さん:2008/08/09(土) 07:54:30
ax^3+bx^2+cx+d=0
をxについて解け

332 名前:132人目の素数さん:2008/08/09(土) 13:49:14
>>331

x=N-(p/3),p=b/a、q=c/a
m=(-1/3)p^2+q、n=(2/27)p^3-(pq/3)+r
N=u+v、ωu+(ω^2)v、(ω^2)u+ωv (ω=[3]√1=(-1+i√3)/2)
u=[3]√[-(n/2)+√{(n^2)/4+(m^3)/27}]
v=[3]√[-(n/2)-√{(n^2)/4+(m^3)/27}]


333 名前:132人目の素数さん:2008/08/10(日) 04:35:20
次の虫食い算を解いてください。

 KYOTO + OSAKA = TOKYO

334 名前:132人目の素数さん:2008/08/10(日) 05:30:00
00000+00000=00000.
01010+09000=10010.
14131+17010=31141.
27252+25020=52272.
41373+32040=73413.
54494+40050=94544.


335 名前:132人目の素数さん:2008/08/10(日) 15:02:20
シュワルツの不等式を用いて次の不等式を証明せよ。(sqrtは√を表す)

sqrt[ Σ[k=1,n]{x(k) - y(k)}^2 ] <= sqrt{ Σ[k=1,n]x(k)^2 } + sqrt{ Σ[k=1,n]y(k)^2 }

336 名前:132人目の素数さん:2008/08/10(日) 15:05:21
>>335
どこが面白いの?


337 名前:132人目の素数さん:2008/08/10(日) 15:18:59
>>335
腹抱えてワロタ

338 名前:132人目の素数さん:2008/08/11(月) 08:24:08
>>333
それは一般的には虫食い算とは言わないと思う。

339 名前:132人目の素数さん:2008/08/11(月) 19:48:07
この図形を合同な二つの図形に分割して下さい
(×はずれないように書いてるだけだから無視して下さい)

×□□□□□□
□□□□□□□
□□□□□□□
□□□□□□□
□□□□□××

340 名前:132人目の素数さん:2008/08/11(月) 20:35:58

×■■□□□□
■■□□□□□
■■□■■□□
■■■■□□□
■■■■□××

なかなかおもしろい

341 名前:132人目の素数さん:2008/08/12(火) 00:38:58
いろんな問題あるからよかったら来てね
http://changi.2ch.net/test/read.cgi/jsaloon/1202402941/l50


342 名前:132人目の素数さん:2008/08/12(火) 02:50:06
699 名前:132人目の素数さん[sage] 投稿日:2008/08/11(月) 20:44:20
3√(5+2√5)-√(25+10√5)を√(A+B√5)の形にせよ。ただし、AとBは整数とする。

700 名前:132人目の素数さん[] 投稿日:2008/08/11(月) 21:14:02
>>699
さすがにそれはない

701 名前:132人目の素数さん[] 投稿日:2008/08/11(月) 21:18:34
>>699
これはひどい

704 名前:132人目の素数さん[sage] 投稿日:2008/08/11(月) 21:40:14
>>700-701
あれ?わかると思ったけどなぁ・・・
別のスレに書いたほうが良かった?


343 名前:132人目の素数さん:2008/08/13(水) 04:02:31
ドラえもんの「4次元ポケット」には「どこでもドア」は入らないのではないでしょうか?
(「四次元ポケット」「数学」でぐぐるとこの話は色々出てきますが、数学板では未出っぽいので出してみる)

4次元ポケットは、縦・横・厚さは(伸び縮みしますが、高々)それぞれ30cm以下に見えます。(*1)
「4次元」というからには、縦・横・厚さ以外にあと一つ「何か」があるんでしょう。その「何か」の大きさは問いません。
4次元ポケットのなす空間を P ⊂ R^4 とおくと、(*1)より、開区間 (0,30) に対して
P ⊂ (0,30)×(0,30)×(0,30)×R (*2)
とできる思われます。

一方、どこでもドアは、縦(高さ)は1.5m以上、横も80cm以上でしょう。厚さは5cm以上として
どこでもドアのなす空間を D ⊂ R^3 とおくと、同様に
D ⊃ (0,150)×(0,80)×(0.5) (*3)
とできるでしょう。

(*2)(*3)の条件から、 DはPに「入らない」ような気がします。証明できてませんけど。
ここで、「入る」の定義は…えーと…えーと…、
  f: R^3→R^4 で、任意の2点間の距離を保つような写像f が存在して、
  f(D) ⊂ P であるならば「DはPに入る」と定義します。

問題ていうか課題としては、
(1)DはPに入るかどうか。
  (fが存在すると仮定して、fは線分を同じ長さの線分に写す、ってことは簡単に示せそう。その後は…?)
(2)もしDがPに入らないならば、上記Pの決め方や「入る」の定義などをうまく変更して、DがPに入るようにしてください。
  「4次元ポケット」という名称やその形状ともうまくマッチするような、面白い考え方はありますかね?
  (例えば P ⊂ C^4 ? R^8 と解釈すれば簡単に入るけどいまいち面白くない)

344 名前:132人目の素数さん:2008/08/13(水) 04:36:17
1.4次元ポケットを裏返す。ドラえもんものびたも、どこでもドアも、これでポケットの中。

345 名前:132人目の素数さん:2008/08/13(水) 04:43:06
2.4次元ポケットはスモールライト装備である。

346 名前:132人目の素数さん:2008/08/13(水) 04:45:11
>>343 空間Dを細かく分解して各パーツを違うtに配置すればいい。

347 名前:132人目の素数さん:2008/08/13(水) 04:45:54
3.もう一つの次元は「スケール」である。

348 名前:132人目の素数さん:2008/08/13(水) 04:54:33
厚さが5cmなんだから、縦にすれば、あと必要なのは横だけじゃないか?
80cmは4次元方向を用いればよい。

349 名前:132人目の素数さん:2008/08/13(水) 05:01:33
4.ポケットの口がゴム製だった。

350 名前:132人目の素数さん:2008/08/13(水) 05:05:49
5.ポケットの入り口で空間が歪んでいた。

351 名前:132人目の素数さん:2008/08/13(水) 05:13:18
6.ドラえもんの作者が実は赤塚不二夫だった。


これでいいのだ。

352 名前:132人目の素数さん:2008/08/13(水) 07:45:29
30x30x30x∞って、また、ものすごく狭い4次元空間だな。

353 名前:132人目の素数さん:2008/08/13(水) 09:25:24
ガリバートンネル的な機能が付いてるんじゃないのか?

354 名前:132人目の素数さん:2008/08/13(水) 12:10:55
>>352
無限大に狭いも広いもない

…ないことはないが

355 名前:132人目の素数さん:2008/08/14(木) 12:21:22
半径15cm,深さ15cmの中華丼ぶりにスープが深さ3cmのこっているとき、どんぶりをてでもって
傾けて回すとき、スープの水面が作る空間の体積は?

356 名前:132人目の素数さん:2008/08/14(木) 16:02:17
>>355 断面図書いて回転体の体積求める。

357 名前:132人目の素数さん:2008/08/14(木) 23:05:28
>>356
> 断面図書いて
問題は↑ココだろ

358 名前:132人目の素数さん:2008/08/15(金) 06:16:44
体積条件でエンベロープの接平面群を出すのがものすごく難しい。数値解析に回す。
バリエーショナルならいけるかも。
学部2年の演習問題レベル。

359 名前:132人目の素数さん:2008/08/15(金) 06:18:11
解をしっているのは、ラーメン屋のおやじぐらいだ。

360 名前:132人目の素数さん:2008/08/15(金) 06:31:10
どんぶりにある長さの箸を縁から滑らせればいい。それが接平面

361 名前:132人目の素数さん:2008/08/15(金) 06:33:26
いっきに高3レベルに落ちてしまった・・・中3でもできるかも。

362 名前:132人目の素数さん:2008/08/15(金) 06:34:32
体積を箸の長さで計算してから体積条件で長さを決める。

363 名前:132人目の素数さん:2008/08/15(金) 08:26:22
>>355
意味不明

364 名前:132人目の素数さん:2008/08/15(金) 08:30:18
U(t):v*(p-s)=0
v=(cost,sint)
p=(x,y)
s=12(cost,sint)
Ut:vt*(p-s)-v*st=0
vt*p=0
(-s,c)*(x,y)=-sintx+costy=0
cost(x-12cost)+sint(y-12sint)=costx+sinty-12=0
(y^2+x^2)cost=12x cost=12x/(x^2+y^2)
sint=12y/(x^2+y^2)
(12^2)/(x^2+y^2)=1
x^2+y^2=12^2
途中から円であとは直線の壁・・・


365 名前:132人目の素数さん:2008/08/15(金) 08:58:17
微分方程式からフィボナッチの一般項求める問題が面白かった。
有名なんかな?
f(x)=蚤_nx^n を弐階まで微分して求める。

366 名前:132人目の素数さん:2008/08/15(金) 19:35:52
1,2,3 の 3つの数字で演算子(重複無しで高校レベルまで)を使って出来る限り大きな数を作ってください
123など繋げるのもありです
()はいくつつかってもいいです

367 名前:132人目の素数さん:2008/08/15(金) 19:55:28
>>365
線形の場合に母関数を考えるのは常套手段。

368 名前:132人目の素数さん:2008/08/15(金) 19:55:57
(3^21)!

369 名前:132人目の素数さん:2008/08/15(金) 19:57:52
>>368
不正解
(2^31)!の方が大きいしね


370 名前:132人目の素数さん:2008/08/15(金) 20:02:03
1/(3-2-1)

371 名前:132人目の素数さん:2008/08/15(金) 20:05:50
>>370
1を二つも使ってる

372 名前:132人目の素数さん:2008/08/15(金) 20:16:19
なんかわろた
>>370せめて-log(3-2-1)と書けよ


373 名前:132人目の素数さん:2008/08/15(金) 20:43:32
(3^21)!の方が大きいぜ!

374 名前:132人目の素数さん:2008/08/15(金) 23:46:52
おまえが作った値がxだとしたら俺は(x)!を提出してやる。

というわけで、いくらでも大きくできるでFAだろう。

375 名前:132人目の素数さん:2008/08/15(金) 23:54:53
>>374
チッチッ!
>演算子(重複無しで・・・

376 名前:132人目の素数さん:2008/08/15(金) 23:58:21
2^31!かな。

377 名前:132人目の素数さん:2008/08/16(土) 00:00:16
>>376
正解です
31!>2^31 ですしね

378 名前:132人目の素数さん:2008/08/16(土) 00:03:49
エクセルの画面出して、どこでもいいからセルを右クリックして
ハイパーリンクを選んで「ファイル・ウェブページ」の
「ブラウズしたページ」をクリックする。

379 名前:132人目の素数さん:2008/08/16(土) 00:21:47
2^(31!) だろう。

380 名前:132人目の素数さん:2008/08/16(土) 00:29:48
3^(21!)と比べるとどうなんだろう?
そもそも >>374 の (3^21)! は明らかにダメなのか?

381 名前:132人目の素数さん:2008/08/16(土) 00:30:45
>>379
そうですね 書き方の問題です
それで正しいです


382 名前:132人目の素数さん:2008/08/16(土) 00:31:32
>>380
これは対数をとればいいですね
21!log3 31!log2
後者が大きいのは自明です

383 名前:132人目の素数さん:2008/08/16(土) 00:36:12
9を3つ使って作れる最大の整数を求めよ。
ただし階乗は使っちゃダメ。

384 名前:132人目の素数さん:2008/08/16(土) 00:37:16
>>383
当然、9を三つ使いさえすればいいんだから
9999999999999999999999999999999
とかもOKなんだな?

385 名前:132人目の素数さん:2008/08/16(土) 00:40:15
>>383
9^99

386 名前:132人目の素数さん:2008/08/16(土) 01:10:36
9^9^9

387 名前:132人目の素数さん:2008/08/16(土) 01:17:07
次の極限値を求めよ。(ガウス記号を使用してもよい)

 lim [x + h] (xは実数、[]はガウス記号)
h→1-0

388 名前:132人目の素数さん:2008/08/16(土) 01:27:31
[x+1](x∈R-N)
x(x∈N)

389 名前:132人目の素数さん:2008/08/16(土) 01:38:43
xの場合分けで来ましたか。まんまやねw
ちなみに、場合分けなしの以下の表現を想定してました。
-[-x]

390 名前:132人目の素数さん:2008/08/16(土) 19:05:51
今、自分を含めて100人の死刑囚がいます.この100人に対して悪い王様から次の様な問題が出されました

「1から100までの数字(整数に限る)の中から一つの数字を紙に書きなさい
次に紙に書かれた100人全員の数字の平均の1/2を予想(整数に限る)しなさい」

最も近い数字(整数)を予想した人だけが釈放され、残りはその場で処刑される事になっています
釈放される確率をできるだけ高くするにはどんな数字を書き、どんな数字を予想したらいいでしょうか?
ただし100人の死刑囚は互いに相談する事はできず、全員が死刑は嫌だと考え合理的な判断をするものとします

391 名前:132人目の素数さん:2008/08/16(土) 19:09:47
>>390
1かな?

392 名前:132人目の素数さん:2008/08/17(日) 01:39:12
>>386
正解

393 名前:132人目の素数さん:2008/08/17(日) 02:28:44
>390
単純に25かと考えてしまうけど、違うんだろうなぁ。分からない。

394 名前:132人目の素数さん:2008/08/17(日) 20:48:14
A君(自分)とB君は階段(段数は20)を使って勝負をしています
じゃんけんをしてグーで勝てば3段,チョキで勝てば5段,パーで勝てば6段上れます
じゃんけんを繰り返して先に階段の一番上に到達すれば勝ちです
今,二人は階段の下にいます.A君は勝つ確率を高める為にはグーチョキパーをどう出すのが最良でしょうか?
ただしB君も勝つ為に最良の手を出すものとします

395 名前:132人目の素数さん:2008/08/18(月) 16:40:34
グーチョキパーをそれぞれ以下の確率で出すときに進める期待値は315/196歩で最大
グー⇒5/14、チョキ⇒3/14、パー⇒6/14


396 名前:132人目の素数さん:2008/08/18(月) 16:59:46
>>390
数字、予想ともに1

もしそうでない考えをする人が何人かいたときに
もっとも予想が外れにくいから
(平均の1/2を整数で予想というのがミソだな)

397 名前:132人目の素数さん:2008/08/20(水) 10:25:05
>>395
相手がそういった確率出すとして
こちらも対策を練れば変わってくるのでは
そしてそれに相手が対策を練れば…

398 名前:132人目の素数さん:2008/08/20(水) 10:31:48
永遠にあいこですね

399 名前:132人目の素数さん:2008/08/20(水) 12:03:51
>>397
そういうことを言うなら、変わることを示せよ。

400 名前:132人目の素数さん:2008/08/20(水) 12:15:42
>>395
には全部チョキで対抗するのがいいだろうな

401 名前:132人目の素数さん:2008/08/20(水) 12:16:40
>>399
変わらないことを示してくださいよベイベ〜

402 名前:132人目の素数さん:2008/08/20(水) 12:20:12
>>400
そしてそれには全部グーで返してループするのね

403 名前:132人目の素数さん:2008/08/20(水) 14:24:32
そもそも勝つための最良の手とは何だろう?
>>395は進める歩数の期待値を最大にしたが、はたしてそれでいいのだろうか?
進める歩数の期待値がどんなに大きくても、自分と相手のその期待値が等しいならば
勝つのは五分五分、ということはそれは勝つための手とはいえないのではないか?

勝つための最良の手とは、(自分の期待値-相手の期待値)が最大になるような手ではないのだろうか?

ここで、「相手も自分もどちらも最良の手を出す」ということを考えると
(自分の期待値-相手の期待値)は0にしかならないのではないか?





404 名前:132人目の素数さん:2008/08/20(水) 15:08:57
>>394
これって、今いる段数によって
各手を出す確率の配分が変わる?
17,18,19段目にいればどんな手で勝ってもいいし

いやそもそも、相手のいる段数も関わってくるのかな

そうすると、グーチョキパーそれぞれを出す最適な確率Pg,Pc,Ppは
Pg(n,m), Pc(n,m), Pp(n,m)というように
自分の今居る段数nと相手のいる段数mの関数にならなきゃいけないのかな

405 名前:132人目の素数さん:2008/08/20(水) 15:19:29
>>394 こういう問題いっつもみるけど
 相手も最良の手考えるなら結局相子しかでなくね?

406 名前:132人目の素数さん:2008/08/20(水) 15:31:20
>>405
ランダムという要素を入れればあいこではなくなる。
もちろん統計的に長い目で見れば引き分けかもしれないが

407 名前:132人目の素数さん:2008/08/20(水) 15:33:06
もしくは裏のかき合いになって結局運否天賦
確率の問題としては適してない題材だな。

408 名前:132人目の素数さん:2008/08/20(水) 18:15:41
もし最強の手というものがあるとしたら
あいても自分も同じ手がとれる以上
相手も自分も同じ勝率にしかならない。

このことから、以下のことがわかる。
・最強の手があるとしてもたかだか50%の勝率である。

409 名前:132人目の素数さん:2008/08/20(水) 18:25:34
>>408
いや逆だろ。最低50%だ。
相手も同じ手を使ってくるとは限らないし。

410 名前:132人目の素数さん:2008/08/20(水) 18:51:14
>>409
同じ手を使ってこない時点で最強の手ではないことになるね

411 名前:132人目の素数さん:2008/08/20(水) 19:01:26
最強の手が"あったら"その勝率は50%だな

412 名前:404:2008/08/20(水) 20:29:35
自分がn段,相手がm段のとき
両者が最適戦略をとったときの自分の勝率を V(n,m) とする

V(n,m)を再帰的に算出し、その過程で、最適な戦略( Px(n,m)の値 )を得る。

・再起計算の初期値
V(n,m) =1 ( x=g,c,p,  20≦n, m<20 )
V(n,m) =0 ( x=g,c,p,  n<20, 20≦m )

・V(n,m)の再帰計算
V(n,m)
 = (PgPg'+PcPc'+PpPp') V(n,m)
 + PgPc' V(n+3,m) + PcPp' V(n+5,m) + PpPg' V(n+6,m)
 + PcPg' V(n,m+3) + PpPc' V(n,m+5) + PgPp' V(n,m+6)
≡ (sum[x] PxPx')V(n,m) + (sum[x,y] PxP'y C(n,m,x,y))

( ただし >>404の Px(n,m) を Pxと略記, 相手の手の確率をPx'と書く,  x,y は g,c,pを渡る変数 )
C(n,m,x,y) は (x≠yのとき) 右辺PxPy'の係数, x=y の時 0 と定義
C(n,m,x,y) は 再帰計算の過程ですでに算出されている

以上から
V(n,m) = (sum[x,y] PxP'y C(n,m,x,y))/(1-sum[x] PxPx')

上記のV(n,m)を Px, Px' の関数V(Pg,Pc,Pp,Pg',Pc',Pp')とみなし
sum[x]Px=sum[x]Px'=1 の拘束条件のもと
VがPxに関して極大,Px'に関して極小となるようなPg,Pc,Pp,Pg',Pc',Pp'を見つけることで
最適戦略が得られる、と思う

413 名前:132人目の素数さん:2008/08/21(木) 07:59:15
戦略をいつ決定するかとか、どのような戦略がありうるかとかを
定めない以上、数学の問題としては不備。

414 名前:132人目の素数さん:2008/08/21(木) 12:12:04
>>394
AはA:B:C(A+B+C=1)の確率でグー、チョキ、パーを出すとすると、
(Aが進む歩数の期待値)
=3(Aがグーを出す確率)(Bがチョキを出す確率)
+5(Aがチョキを出す確率)(Bがパーを出す確率)
+6(Aがパーを出す確率)(Bがグーを出す確率)
=3Ab+5Bc+6Ca
(Bが進む歩数の期待値)
=3(Aがチョキを出す確率)(Bがグーを出す確率)
+5(Aがパーを出す確率)(Bがチョキを出す確率)
+6(Aがグーを出す確率)(Bがパーを出す確率)
=3Ba+5Cb+6Ac
よって(Aが進む歩数の期待値)=(Bが進む歩数の期待値)のとき、
3Ab+5Bc+6Ca=3Ba+5Cb+6Ac
(6C-3B)a+(3A-5C)b+(5B-6A)c=0
6C-3B=0かつ3A-5C=0かつ5B-6A=0
A:B:C=5:6:3
よって5:6:3の確率で出せばよい

415 名前:132人目の素数さん:2008/08/21(木) 21:51:52
>>414
の通り5:6:3でグーチョキパーを出す予定の相手には
全部グーで挑ませてもらおう。


416 名前:132人目の素数さん:2008/08/22(金) 02:39:20
A=5/14, B=6/14, C=3/14
a=1, b=0, c=0

Aの進む歩数の期待値
3Ab+5Bc+6Ca=6Ca=18/14

Bの進む歩数の期待値
3Ba+5Cb+6Ac=3Ba=18/14

>>415
ジャンケンそのものの勝率は上がっても
歩数の期待値の意味では相打ちじゃね?

417 名前:132人目の素数さん:2008/08/22(金) 04:42:27
>>416
Bの進む歩数の期待値は
5Ba+6Cb+3Ac=5Ba=30/14

418 名前:132人目の素数さん:2008/08/22(金) 07:17:01
>>416
期待値を競ってるわけじゃないので>>415の勝ち。

>>417
Baはグーで勝ち。

>>355
0。


419 名前:132人目の素数さん:2008/08/22(金) 19:44:10
>>418
期待値が同じならゲームの勝率も同じじゃね?

420 名前:416:2008/08/22(金) 21:33:51
ゲーム勝利条件は
どちらが先に20段以上まで登ったか、だろうから
厳密には歩数の期待値が多ければ
勝率も高いとは言えないだろうけどね

実際、双方ともに17段から19段でゴール間近のときは
>>414 の戦略に対しては >>415 のようにグーを出した方がいい

やっぱりここは
自分と相手の段数に対して各々
グーチョキパーの割合を変えて決めなければいけないと思う

421 名前:132人目の素数さん:2008/08/22(金) 23:15:49
>>420
相手が過去に選んだ手によって現在の手を変える戦略は考えないの?

422 名前:132人目の素数さん:2008/08/23(土) 03:56:02
>>419
>>414がグーを出す場合は影響がないので省ける。
チョキかパーを出した場合10回の内
パーが4回以上なら>>414の勝ちで
3回以下なら>>415の勝ち。


423 名前:132人目の素数さん:2008/08/23(土) 10:59:40
あ、なるほど。 20段以上は無駄になるからか。



424 名前:132人目の素数さん:2008/08/23(土) 21:54:56
最強戦略が存在することの(不完全な)証明
π1,...,πnを全ての戦略をとし、V(πi,πj) を 戦略πjが戦略πiに勝つ確率とする

ゲーム開始時に確率Piで戦略πiを選択し、以後その戦略を突き通すという戦略を新たにπ(P1,...,Pn)とする
この戦略はP1,...,Pnのパラメータを適当に調整することでどんな戦略もエミュレートできる

自分が戦略π(P1,...,Pn)を用い、相手が戦略π(Q1,...,Qn)で挑んでくるとする。Qiのパラメータの選び方によらず勝率50%を達成するPjの存在を示す

行列V を i,j成分が V(πi,πj) である行列と定義する
P,Qをそれぞれ、P1,...,Pn、Q1,...,Qnを並べた縦ベクトルとする
P = V^(-1) [1/2,...,1/2]' と、すべての要素が1/2の縦ベクトルにVの逆行列をかけたものを選ぶ
(Vの正則性、Pi>0,sum[i]Pi=1が成り立つ事の証明は…まだしていない、多分 "V+V'=1を並べた行列" 等の性質を使う)

このときの自分の勝率は
V(π(Q1,...,Qn),π(P1,...,Pn)) = sum[i,j] Qi V(πi,πj)Pj = Q'VP= Q' [1/2,...,1/2] = sum[i] Qi (1/2) = 1/2
とQjの値に依存することなく常に50%になる。つまりπ(P1,...,Pn)は最強の戦略になる。

425 名前:132人目の素数さん:2008/08/24(日) 12:30:44
>>424
「戦略」を定義せずに議論しても全く無意味でしょ.

(1)
> π1,...,πnを全ての戦略をとし
ここから破綻している。「戦略」は有限個しかないの?
実際 π(P1,...,Pn) は任意のパラメタに対して「戦略」ではないの?

(2)
> Vの正則性
普通に「戦略」を定義するとVは正則にならないと思われる.
実際,ある「戦略」が複数の「戦略」の和で書けている場合を考えれば
逆が無いのはかなり直感的に分かる.

426 名前:132人目の素数さん:2008/08/24(日) 12:45:04
>>424
結局君の証明は,戦略集合 C が凸集合(「戦略」の凸結合が取れる)
ことを踏まえたうえで,
 『C の端点が高々有限個(基本的な「戦略」π1, ..., πn が存在)』
を仮定している(こうすれば,その証明を正当化できる).

しかしこの仮定は相当強くて,C がコンパクトになってしまう.
よって勝率関数 V: C×C → R が最適解を持つのはほとんど自明.

凸集合であることは通常認められる仮定だが,
端点が有限個という仮定は,ほぼ絶望的だろう.

427 名前:132人目の素数さん:2008/08/27(水) 19:16:45
次のような非常に制限の強いプログラム言語X_1を考える。
1.使用できるデータ型(変数、定数とも)はC言語で言うところのunsigned charのみである。
2.使用できる演算は代入、足し算、引き算、論理演算(AND,OR,NOT,XOR,右シフト、左シフト)のみである。
3.if,while,goto,関数呼び出し等の制御構造は一切なし。プログラムは上から下へ順番に実行されるのみである。
4.unsigned char型の変数を使うことが出来る。個数に制限はない。
5.一ステップで代入一回、演算一回行うことが出来る。つまり1ステップは以下のどれか。
 a=b;
 a=~b;
 a=b+c;
 a=b-c;
 a=b&c;
 a=b|c;
 a=b^c;
 a=b<<c;
 a=b>>c;
 (※単項の-は禁止) 
6.プログラムの終わりに次のような値を返す文を入れる。
 return a;
 この値をプログラムの値と呼ぶ。
7.プログラム中で使用できる定数は0x01のみである。

問題1
プログラム言語Pにおいてプログラムの値がaになるもので
最小ステップのプログラムを、Pにおけるaを返すエレガントなプログラムと呼ぶ。
0〜255についてそれぞれその値を返すX_1におけるエレガントなプログラムを一つ挙げよ。

問題2
プログラム言語X_1に対してプログラム中で使用できる定数を0x01では無く、他の値aに変えたものをプログラム言語X_aと呼ぶ。
また、エレガントなプログラムのステップ数が最も大きくなるような値をそのプログラム言語の最悪の値と呼ぶ。
プログラム言語X_0〜X_255の内、最悪の値に対するステップ数が最も小さくなるプログラム言語はどれか。


428 名前:132人目の素数さん:2008/08/27(水) 19:35:52
×プログラム言語X_0〜X_255の内、最悪の値に対するステップ数が最も小さくなるプログラム言語はどれか。
○プログラム言語X_0〜X_255の内、最悪の値に対するエレガントなプログラムのステップ数が最も小さくなるプログラム言語はどれか。

429 名前:132人目の素数さん:2008/08/27(水) 20:17:31
問題1って、回答は256個書かないといけないの?


430 名前:132人目の素数さん:2008/08/27(水) 20:21:03
256個が面倒だったら、X_1における最悪の値どれか一つとそのエレガントなプログラム1個でいいや。


431 名前:132人目の素数さん:2008/08/27(水) 20:36:55
よく分からんけど
「エレガント」ってのは計算機科学では
何か厳密な定義があるの?

>>428見る限りはあるんだろうね。
そういう出題の仕方だから。

432 名前:132人目の素数さん:2008/08/27(水) 20:38:09
>>431
出題者じゃないが、問題文はちゃんと読もうぜ。
> 最小ステップのプログラムを、Pにおけるaを返すエレガントなプログラムと呼ぶ。

と定義がある。

433 名前:427:2008/08/27(水) 20:57:06
>>431
グレゴリーチャイティンて言う人の知の限界って本を読んだらエレガントなLisp式という言葉が出てきた。
これはその本を読んでみて作った問題。




434 名前:132人目の素数さん:2008/08/27(水) 21:09:49
ああ、Kolmogorov計算量とかそういう話題ね

435 名前:427:2008/08/27(水) 21:16:44
ちなみに俺も答え知らないので宜しくw
ひょっとしたらめちゃくちゃ難しい問題なのかもしれない。
とくに問題2は。



436 名前:132人目の素数さん:2008/08/27(水) 21:29:49
2項演算子で枝刈りがうまくできん。キレイに解く方法あるんだろうか。


437 名前:427:2008/08/27(水) 21:58:16
とりあえず。
「エレガントなプログラムを作るためには一度値を代入した変数に再度値を代入する必要はない。」
多分正しいと思う。


438 名前:427:2008/08/27(水) 22:05:52
あと、もう一つ。
「エレガントなプログラムの実行過程で異なる変数が同じ値になることはない。」
これも多分正しいと思う。


439 名前:132人目の素数さん:2008/08/27(水) 22:28:23
X_1 で 0 を返すエレガントなプログラムってこうならない?
 a=0x01;
 b=0x01;
 c=b-a;
 return c;
以下みたいな別解もあるけど、上のもエレガントだよね? >>438って正しいかなあ。
 a=0x01;
 b=~a;
 c=b&a;
 return c;


440 名前:427:2008/08/27(水) 22:38:17
紛らわしかったかもしれませんが、一応演算の引数に定数を書くのはありとします。
だから0を返すエレガントなプログラムは

a=0x01-0x01;
return a;

ということで。
return文の中での演算は禁止としましょう。




441 名前:427:2008/08/27(水) 22:39:46
あと、2項演算の引数に同じ変数を用いるのも可とします。
a=b<<bとか。


442 名前:427:2008/08/27(水) 23:29:30
もう一つあった。
「X_1におけるエレガントなプログラムのステップ数が14を越えることはない。」

証明は各ビットが立った変数を用意するのに7ステップつかって、
さらに各ビットのオアをとるのに7ステップ使えばどんなデータも生成できる。
14と言う数字はとっても荒くて、ちょっと頑張ればもっと良い数字が得られると思う。


443 名前:132人目の素数さん:2008/08/28(木) 09:18:36
プログラムが豊富すぎて解析が難しいが、
さらにプログラム言語を制限し、加減とビットシフトのみが許される言語で、
数値 n が k 手以下で作れるかどうかの判定がNP完全になる。
(STOC2005くらいだったと思う)

なので、きっとこの問題も、エレガントな解答があると
P=NPの解決に近づく問題だと思われる。

444 名前:132人目の素数さん:2008/08/28(木) 21:09:43
X_1におけるエレガントなプログラムのステップ数は7以下。
返したい値のビット表示において1の個数が5個以上の時は、
最後にNOT演算を使えばよい。
以下説明が面倒なので例だけw

(例)55(=00110111)を返す
a=0x01<<0x01
b=a|0x01
b=b<<a
b=b<<a
b=b|a
b=<<a
b=~b
return b

ちなみに必要な演算は<<と|と~のみ。

445 名前:132人目の素数さん:2008/08/28(木) 21:45:53
X_aにおけるエレガントなプログラムのステップ数は多めに見積もって11以下。
何故なら定数をCと置いて
a=C-C
b=~C
b=b+C
a=a-b
により1を生成できるため。

446 名前:132人目の素数さん:2008/08/29(金) 07:03:47
>>445
a=C-C
b=~a
a=a-b
で1を生成できる。
よって、
X_aにおけるエレガントなプログラムのステップ数は10以下。

447 名前:132人目の素数さん:2008/08/29(金) 22:10:48
今休刊になってる古いパズル雑誌の問題ですが

あなたは役人で7種類の通貨の単位を設定できる
(1円玉、2円玉、30円玉など)

一度に使用する硬貨を三枚以内で(同じ額の硬貨複数使用も可能)で1円〜70円の70通りを表現できるように
硬貨を設定するには、7種類の単位をどうするべきだろう?

(考え中)
最低単位の1円玉は絶対必要。また、24円玉以上が一つはないと70円が表現できないですね

448 名前:132人目の素数さん:2008/08/29(金) 22:46:46
>>447
>>427の問題とかなり近い気がする。



449 名前:132人目の素数さん:2008/08/29(金) 23:16:33
>>447
ボトムアップでやっていけば解けないかな?
ちゃんと確かめてないけどイメージはこんな感じ。

for(i=1;i<=70;i++)
{
if(iを今までの硬貨3枚以内で表現できない)
 {
iを通貨の単位として設定する。
}
}

450 名前:449:2008/08/29(金) 23:30:48
ごめん、なんか全然駄目っぽい。
>>449は忘れて。



451 名前:132人目の素数さん:2008/08/29(金) 23:35:46
>>447
1円玉の次に低い単位が
2円玉か3円玉か4円玉かのどれかになるんですが
分岐が多くなりそうで

452 名前:132人目の素数さん:2008/08/29(金) 23:37:15
まぁ樹形図書いて二時間くらいしらみつぶしにやってればできるね。
うまい解法はなさそうだし。

453 名前:132人目の素数さん:2008/08/30(土) 00:03:21
35円玉は必要になる予感。


454 名前:132人目の素数さん:2008/08/30(土) 00:13:56
この流れで言ってしまってはミもフタもないが
たとえ造れたとしても現在流通している以外の貨幣(紙幣)って要らないよな
一と五のみで構成された金額体系は秀逸すぎる

だからこそ五万札や十万札でなく、二千札考えた奴にはポカーンとしてしまう
ミレニアムのしょぼい魔力にとり憑かれただけだろうな


455 名前:132人目の素数さん:2008/08/30(土) 00:23:14
>>454
> 一と五のみで構成された金額体系は秀逸すぎる

そろばんやってりゃ誰にでも思いつくと思う。

> だからこそ五万札や十万札でなく、二千札考えた奴にはポカーンとしてしまう
> ミレニアムのしょぼい魔力にとり憑かれただけだろうな

これには、同意。

456 名前:132人目の素数さん:2008/08/30(土) 00:52:25
>>447
1, 4, 5, 15, 18, 27, 34で作れるな.
また,多分これ以外では不可能.

1,1+1,1+1+1,4,5,1+5,1+1+5,4+4,4+5,5+5
1+5+5,4+4+4,4+4+5,4+5+5,15,1+15,1+1+15,18,1+18,5+15
1+5+15,4+18,5+18,1+5+18,5+5+15,4+4+18,27,1+27,1+1+27,15+15
4+27,5+27,1+5+27,34,1+34,18+18,5+5+27,4+34,5+34,1+5+34
5+18+18,4+4+34,4+5+34,5+5+34,18+27,1+18+27,5+15+27,15+15+18,15+34,1+15+34
15+18+18,18+34,1+18+34,27+27,1+27+27,4+18+34,5+18+34,4+27+27,5+27+27,15+18+27
27+34,1+27+34,18+18+27,15+15+34,4+27+34,5+27+34,15+18+34,34+34,1+34+34,34+18+18

457 名前:132人目の素数さん:2008/08/30(土) 00:57:07
>>456
エレガントな解法?
どう解いたかぜひ教えていただきたい。


458 名前:132人目の素数さん:2008/08/30(土) 01:10:37
>>457
期待させて申し訳ないが,全くエレガントでない.
プログラムを書いて探索した.

参考までにソースコード(C言語):
http://kansai2channeler.hp.infoseek.co.jp/cgi-bin/joyful/img/7662.c

459 名前:132人目の素数さん:2008/08/30(土) 01:20:33
>>458
数学はもうコンピュータ無しでは立ち行かない時代になってしまったのだなぁ。(大げさか

460 名前:447:2008/08/30(土) 01:25:21
皆様ありがとうございました

461 名前:132人目の素数さん:2008/08/30(土) 07:58:06
>>454
全くだ。ユーロ圏に住んでるが
2セント、20セント、2ユーロコインは無駄すぎる

462 名前:132人目の素数さん:2008/08/30(土) 16:10:17
>>461
ユーロ圏で日本語版のWindowsだかMacintoshだかが使える事に驚いたわ

人間の行動そのものが無駄でしかない。損得勘定無しに動ける故の欠陥かね

463 名前:132人目の素数さん:2008/08/30(土) 16:38:28
そもそも無駄とは何か?
文学は無駄か?
芸術は無駄か?

464 名前:132人目の素数さん:2008/08/30(土) 22:38:00
>>427
適当に枝刈しながら、総当りでやってみた。

問題1の最悪のステップは 7

最悪のステップの例として、例えば 71 を求めるには。

1: a = ~1 [254]
2: b = a - 1 [253]
3: c = 1 - b [4]
4: d = c << c [64]
5: e = d + c [68]
6: f = e - b [71]
7: return f

ステップ数が 7 ぐらいなら数分で解けるから、問題2も2日ぐらいあれば
解けると思う。

465 名前:427:2008/08/30(土) 23:09:31
>>464
解けましたか。乙です。

X_1における最悪の値になにか特徴は見出せますか?
もしかしてunsigned char じゃなくてunsigned shortでもいけそうですか?




466 名前:132人目の素数さん:2008/08/31(日) 02:23:21
>>464
ステップ数は6の間違い?

467 名前:132人目の素数さん:2008/08/31(日) 07:30:27
>>465
> X_1における最悪の値になにか特徴は見出せますか?

ぱっと見、特に特徴はなさそう。

各ステップの分布は以下の通り。

Step 1: 1
Step 2: 0, 2, 254
Step 3: 3, 4, 8, 127, 252, 253, 255

Step 4:5, 6, 7, 9, 10, 12, 16, 24, 31, 32, 63, 64, 125, 126, 128, 129, 130, 240, 244, 247, 248, 249, 250, 251

Step 5: 11, 13, 14, 15, 17, 18, 19, 20, 21, 22, 23, 25, 26, 27, 28, 29, 30, 33, 34, 36, 40, 48, 60, 61, 62, 65, 66, 68, 80, 96, 120, 122, 123, 124, 131, 132, 160, 176, 190,
191, 192, 193, 194, 195, 196, 208, 216, 220, 223, 224, 225, 226, 228, 230, 231, 232, 233, 234, 235, 236, 237, 238, 239, 241, 242, 243, 245, 246

Step 6: 35, 37, 38, 39, 41, 42, 43, 44, 45, 46, 47, 49, 50, 51, 52, 53, 54, 55, 56, 57, 58, 59, 67, 69, 70, 72, 73, 75, 76, 78, 79, 81, 82, 84, 85, 88, 90, 91, 92, 93, 94, 95,
97, 98, 99, 100, 101, 102, 104, 108, 110, 111, 112, 113, 114, 115, 116, 117, 118, 119, 121, 133, 134, 135, 136, 137, 138, 139, 140, 141, 142, 144, 152, 155, 156, 157, 158,
159, 161, 162, 163, 164, 165, 166, 168, 171, 172, 174, 175, 177, 178, 180, 181, 183, 184, 186, 187, 188, 189, 197, 198, 199, 200, 201, 202, 203, 204, 205, 206, 207, 209,
210, 211, 212, 213, 214, 215, 217, 218, 219, 221, 222, 227, 229

Step 7: 71, 74, 77, 83, 86, 87, 89, 103, 105, 106, 107, 109, 143, 145, 146, 147, 148, 149, 150, 151, 153, 154, 167, 169, 170, 173, 179, 182, 185

> もしかしてunsigned char じゃなくてunsigned shortでもいけそうですか?

俺の頭とマシンじゃとても無理。

468 名前:132人目の素数さん:2008/08/31(日) 07:33:21
>>466
どういう意味?

469 名前:132人目の素数さん:2008/08/31(日) 08:31:31
>>468
return文をステップ数にカウントするかどうかってことじゃない?


470 名前:132人目の素数さん:2008/08/31(日) 08:52:04
>>464
71 って 6 手で作れない?ビットシフトの仕様が違うのかしら。

1: a = ~1 [254]
2: b = 1 + 1 [2]
3: c = b << b [8]
4: d = a >> b [63]
5: e = c + d [71]
6: return e

471 名前:470:2008/08/31(日) 10:33:53
俺の手元の結果では >>467 のリストと
71 103 143 151 153 185 が違う(これらが6手で作れる)

あと,問題2は,最悪に対するステップ数最小は
使える定数が 29 67 98 99 101 163 226 227 の場合で,
そのステップ数は5手になった.
ちなみに最悪ステップ数最大は定数が 0 の場合で9手.

472 名前:132人目の素数さん:2008/08/31(日) 11:20:53
右シフトの挙動ってunsignedとsignedで変化するからなぁ。

473 名前:>>467:2008/08/31(日) 12:10:04
>>470-471
すまん、バグってて刈り込みすぎてた。orz
修正したら、>>471 の言う通りだった。

>>472
今回の場合は、unsigned が前提でしょ。

474 名前:427:2008/08/31(日) 12:31:26
チャイティンさんの本によると、問題1の最悪の値はランダム性の高いビット列になるはずです。
半丁博打の親をやるとき最悪の値のビット列によって出す手を決めると負けにくいかもしれません。

そして使用するレジスタのビット幅を8から16、32と大きくしていって、レジスタ幅を無限大へ飛ばしたときの
最悪の値こそ真の乱数列と呼ばれるものにひょっとしたらなるかもしれません。

>>464さんの作成しているプログラムが完成したら、それは真の乱数検定アルゴリズムと呼べるかも?
万が一そうだとしたらちょっとすごいですね。



475 名前:132人目の素数さん:2008/08/31(日) 12:52:28
乱数は定数とは違うだろ

乱数の種にルドルフ数を使おうとネイピア数を使おうと勝手だけど、それは乱数とは言いがたいと思われ

476 名前:132人目の素数さん:2008/08/31(日) 13:59:57
何か誤解しているか、文章が不正確だと思う。とりあえず

> 問題1の最悪の値はランダム性の高いビット列になるはずです。

これは嘘。最悪の値 = 7 = 111b のランダム性は低い。

477 名前:132人目の素数さん:2008/08/31(日) 14:51:40
最悪の値となる数値 (74, 77, 83, ...) も複数あるから、その並び順によって
ランダム性も違うし。

とにかく、>>474 はそのチャイティンさんの本の題名ぐらい示すべきかと。

478 名前:132人目の素数さん:2008/08/31(日) 15:04:18
どうでもいいけどステップ数の数え方を統一しようよ。
>>444はreturn文を数えてない。
>>427の5番目のルールを見る限り、return文は数えなくていい。

479 名前:474:2008/08/31(日) 15:34:35
>>476-477
>>433に書きましたが知の限界という本です。
たしかにこの本の内容は私には難しめなので全部は理解してないし誤解があるかもしれません。
ランダム性の高いビット列といってるのは>>477の方の意味です。

480 名前:474:2008/08/31(日) 15:43:06
真の乱数というキーワードは刺激が強すぎてフレームの元かもしれませんね。
十分良い乱数、ぐらいに弱めておいたほうが無難でしょうか。


481 名前:474:2008/08/31(日) 15:58:40
これです。
http://www.amazon.co.jp/知の限界-G-J-チャイティン/dp/443401238X/ref=sr_1_1?ie=UTF8&s=books&qid=1220165773&sr=8-1


482 名前:132人目の素数さん:2008/08/31(日) 16:28:01
>>479
>>477 に書いた通り最悪の値になる数値は複数あるからその並び順が問題になる
と思うが、そう言うことには言及してないの?
あと、そもそも 0 より 1 のビットの方が多いし。(0 は、89個、1 は 95個)
74: 01001010
77: 01001101
83: 01010011
86: 01010110
87: 01010111
89: 01011001
105: 01101001
106: 01101010
107: 01101011
109: 01101101
145: 10010001
146: 10010010
147: 10010011
148: 10010100
149: 10010101
150: 10010110
154: 10011010
167: 10100111
169: 10101001
170: 10101010
173: 10101101
179: 10110011
182: 10110110

483 名前:474:2008/08/31(日) 16:41:43
>>482
乱数の種(プログラムに使用できる定数が0x01であること)が関係しているのかもしれません。
この定数は0x00のほうがよりふさわしいかもしれません。
あと演算もNOTを廃止してNORやNANDを追加したほうがより綺麗な議論になるかもしれません。
プログラムによって圧縮不能なビット列をランダムであると呼ぶ、と本に書いてありました。
もっともこの場合のプログラムはチューリング完全なプログラム言語での話なのですが。

私はX_1においても最小ステップ数の大きい値ほどランダムであると予想しました。



484 名前:474:2008/08/31(日) 16:49:42
あと、真の乱数が複数あってもおかしく無いと思ってますし、
レジスタ幅が大きくなるほど0より1のビットのほうが多いといった誤差のようなものは小さくなっていくと期待しています。

どっちにしろ真の乱数は言いすぎですかね。



485 名前:132人目の素数さん:2008/08/31(日) 17:03:09
そこまで機能を限定したいなら、Malbolgeのcrzで同じ事をやろうか

486 名前:132人目の素数さん:2008/08/31(日) 17:03:54
>>484
真の乱数ってのは有限の状態から導き出せるのかい?

487 名前:132人目の素数さん:2008/08/31(日) 17:07:25
>>484
ビット数が 10 個ぐらいのところで chaitin の議論が適用できると思うのが間違い。

488 名前:474:2008/08/31(日) 17:11:44
>>486
無限長の真の乱数を有限の状態から導き出すのは無理だと思います。
今回の方法でもレジスタ幅が有限のうちは破綻せずにプログラムを走らせられますが、
レジスタ幅を本当に無限にしてしまったら破綻してしまいます。

所詮は乱数列の長さが有限の場合に限って通用する方法だとは思います。
やっぱり真の乱数はフレームの元ですね。すいません。


489 名前:132人目の素数さん:2008/08/31(日) 17:47:25
問題いい?

(1)平面上に4つ以上の幾つかの点を置く。
どの3点も1直線上になく、どの4点も同一真円周上になく、どの2つの点の距離も無理数である。
平面上には幾つ点が置けるか?

(2)もしも平面上に(1)の置き方で置ける点の個数に上限があるとき、立体に拡張したらどうなるか。
ただしどの5点も同一真球上にないものとする。

490 名前:489:2008/08/31(日) 17:49:46
あぁ、書き忘れた。
どの点のx座標、y座標、存在するならz座標をとっても
その値は有理数であるものとする。

例えば(1,-3)はOK、(√3,1)、(5,π)等は駄目

491 名前:132人目の素数さん:2008/08/31(日) 18:08:38
>>489
(1)(n,n^2)で表される点(nは非負整数)を取っていけばいくらでも。

492 名前:132人目の素数さん:2008/08/31(日) 18:23:18
>>483
つまり >>474
> チャイティンさんの本によると、...
というのは嘘で
「『知の限界』を読んで427が考えたところ,...」
が正しいんだよな.しばらく探してしまった.

493 名前:474:2008/08/31(日) 18:32:41
>>492
申し訳ないです…。


494 名前:132人目の素数さん:2008/08/31(日) 19:12:26
>>483
> 私はX_1においても最小ステップ数の大きい値ほどランダムであると予想しました。

この予想をちゃんと書くとどうなるかに依存するが,
厳密な意味で解釈すると,これが成り立たないことが示せる.

何より,こんな読み物読んで下手な予想をする前に,
ちゃんとした本を読んだほうがよい.

495 名前:474:2008/08/31(日) 19:19:58
>>494
そうでしたか。すいません。
後学のためにどこがまずいのか解説していただけるとありがたいです。
たとえば、レジスタ幅が大きくなったとき0と1の割合が大体半々にならずに、
大きく違ってしまうといたことが起こるのでしょうか。


496 名前:132人目の素数さん:2008/08/31(日) 20:03:25
>>495
一番まずいのは言語がチューリング等価でないこと.
>>483 の予想を厳密に解釈し,それが成り立つと仮定すると
X_1 とチューリングマシンが等価になることが示せるが,それはない.

あと,マイナー(でもないが)な考え違いを指摘しておくと,
今の意味のランダム性には
> レジスタ幅が大きくなったとき0と1の割合が大体半々
という性質は不要。これは統計的(一様)ランダム性の条件だが,
情報論的ランダム性の条件ではない。

497 名前:474:2008/08/31(日) 20:17:28
もうちょっと食い下がらせてもらいます。

X_1がunsigned charしか使えないのであればチューリング等価でないのは明らかです。
しかし、レジスタ幅が任意の有限長になることを許すならば私にはチューリング等価の可能性も捨て切れません。
そもそも論理回路はチューリング完全だと思っていました。

あと、統計的ランダム性と情報論的ランダム性は情報論的ランダム性のほうがずっと強い制約なのだと思っていました。
情報論的ランダム性を満たすならば、統計的ランダム性は当然満たされるはず、と思っていたのですが…。



498 名前:132人目の素数さん:2008/08/31(日) 20:45:07
未だによく分からんのだけど、チューリング等価性の証明ってどうやるん?

499 名前:132人目の素数さん:2008/08/31(日) 20:46:09
>>497
関数も作れない、ジャンプも出来ない
どうやってループすんのさね

500 名前:132人目の素数さん:2008/08/31(日) 20:52:03
>>497
ねぇ、昔はどうやって8bitマイコンとかで32bit演算をやってたか知ってる?
あと、どうやってパソコンが負の数を扱ってるか知ってる?

X_1の場合、次の条件を満たせばunsigned intだろうが、signed intだろうが、
扱えるようになるでよ
・無数のレジスタがある
そう、これだけ。

どうしても分からなかったら「多倍長演算」とか「2の補数表現」とかでググってみなよ。
あぁ、この2つのキーワードは全く別物だから単体でね

501 名前:497:2008/08/31(日) 21:12:51
たしかにループできませんね…。
ループが出来なければ任意の大きさの入力を捌くことは出来ないし…。
結局、事前に入力の大きさに上限がある必要がありますね。





502 名前:497:2008/08/31(日) 21:32:49
しつこくてすいません。
もうちょっと教えてください。
出来ればこの問題はすっきり理解したい。

>X_1 とチューリングマシンが等価になることが示せるが

ここはどうやるのでしょう。

503 名前:497:2008/08/31(日) 22:31:37
>>483 の予想を厳密に解釈し,それが成り立つと仮定すると
X_1 とチューリングマシンが等価になることが示せるが

すいません。引用が足りないですね。
正しくはこうですね。


504 名前:132人目の素数さん:2008/08/31(日) 23:05:54
>>503
アルゴリズム的情報理論の基礎事項。適当な本読め。
あといいかげんスレ違い。

505 名前:132人目の素数さん:2008/09/01(月) 07:19:08
n次空間上に2つの点A(a1,a2,...,an)点B(b1,b2,...,bn)がある。
・2点AB間の距離が超越数になる
・a1,...,an,b1,...,bnのいずれかは超越数である
は同値か?

506 名前:132人目の素数さん:2008/09/01(月) 09:19:14
距離の定義はどれ?

507 名前:132人目の素数さん:2008/09/01(月) 09:47:48
>>505 n=1のとき、a=π+1,b=πとおくと
 d(a,b)=1だがbは超越数である。よって命題は偽。

508 名前:132人目の素数さん:2008/09/01(月) 22:16:13
ちょー面白い問題だな

509 名前:132人目の素数さん:2008/09/02(火) 10:36:40
次の( )にそれぞれアラビア数字(何桁でも可)を入れて文を成立させて下さい
漢数字やローマ数字等は不可(全通り答えて下さい)

「この文には
0が( )個
1が( )個
2が( )個
3が( )個
4が( )個
5が( )個
6が( )個
7が( )個
8が( )個
9が( )個
含まれています」

510 名前:132人目の素数さん:2008/09/02(火) 11:25:41
「この文には
0が( 1)個
1が( 11)個
2が( 2)個
3が( 1)個
4が( 1)個
5が( 1)個
6が( 1)個
7が( 1)個
8が( 1)個
9が( 1)個
含まれています」

511 名前:132人目の素数さん:2008/09/02(火) 11:31:33
頻出

512 名前:132人目の素数さん:2008/09/03(水) 05:18:34
各辺の長さが1で底面が正三角形の三角柱ABC-DEFがある。
この三角柱をAEF,BDF,CDEをそれぞれ通る3つの平面で切断する。

問1
平面DEFを含む立体の体積を求めよ。

問2
平面DEFを含む立体の展開図を作図せよ。



513 名前:132人目の素数さん:2008/09/03(水) 05:57:25
問2は平面ABCを含む立体の展開図の方が面白いかも。


514 名前:132人目の素数さん:2008/09/03(水) 06:10:43
>>510
これ以外に解はないの?

515 名前:132人目の素数さん:2008/09/03(水) 11:03:22
>>513
DEFのときとなにか違うのか?

516 名前:132人目の素数さん:2008/09/03(水) 20:45:02
空間上に正三角柱をねじったような6つの頂点を持つ立体があり、
その頂点ABCDEFが次を満たすように並んでいる。

ABの長さは1
三角形ABCは正三角形
三角形DEFは正三角形
三角形ABDは正三角形
三角形BCEは正三角形
三角形CAFは正三角形
三角形ADFは正三角形
三角形BEDは正三角形
三角形CFEは正三角形

このとき、立体ABCDEFの体積を求めよ。

517 名前:132人目の素数さん:2008/09/04(木) 00:40:02
>>516
一辺の長さが2の正四面体から4つの角を取った形になるのかな?


518 名前:132人目の素数さん:2008/09/04(木) 00:43:50
であれば、一辺の長さが1の正四面体の体積をVとして>>516の答えは4V。



519 名前:132人目の素数さん:2008/09/04(木) 05:58:44
( ゚д゚)ポカーン

520 名前:132人目の素数さん:2008/09/04(木) 07:45:24
一辺が2の正四面体の、4つの角からそれぞれ一辺が1の正四面体を切り取った形だよな。
それ以外の形で>>516を満たすものがあるのかもしれんが

521 名前:132人目の素数さん:2008/09/04(木) 09:46:45
成分が0と1だけの3x3の行列Aに対して行または列を任意にひとつ選び
0と1を入れ替える操作をRとします。
任意回の操作Rで移りあう行列を「同値な行列」と言うことにすると、
2^9個の可能な3x3行列のうち「同値でない行列」は何種類あるでしょうか?

522 名前:132人目の素数さん:2008/09/04(木) 12:48:43
>>516
△ABC を底面とすると、立体の高さは √(2/3)
底面からの高さ (√(2/3))t での断面積は
((√3)/4)(1+2t-2t^2)  (0≦t≦1)
立体の体積は
√(2/3) * ((√3)/4) * ∫[0,1](1+2t-2t^2)dt
= (√2)/3

523 名前:132人目の素数さん:2008/09/04(木) 15:19:30
>>521
16通り。

略解:
[STEP1]操作Uiと操作Vjを次のように定義する。
U1:1行目の0と1を入れ替える
U2:2行目の0と1を入れ替える
U3:3行目の0と1を入れ替える
V1:1列目の0と1を入れ替える
V2:2列目の0と1を入れ替える
V3:3列目の0と1を入れ替える
次に、行列の各成分はZ/2Zの元であると見なす。こうすると、

1行目の0と1を入れ替える ⇔ 1行目の各成分に1を足す …*

が成り立つことが分かる。他の行や列についての操作も同様である。
また、このことから、各操作の順番は可換であることが分かり、
また、同じ操作を2回繰り返すと「何もしない」のと同じである
ことが分かる。よって、任意の操作は

Uiを行うか否か(i=1,2,3)
Vjを行うか否か(j=1,2,3)

の6つを決めるだけで決まる。そして、T=(u1,u2,u3)×(v1,v2,v3)∈(Z/2Z)^3×(Z/2Z)^3に対し、
ti=1 ⇔ 「i行目の0と1を入れ替える」(i=1,2,3)
uj=1 ⇔ 「j列目の0と1を入れ替える」(j=1,2,3)
という同一視を行うことで、任意の操作はTと同一視できる。
行列A=(aij)と操作T=(u1,u2,u3)×(v1,v2,v3)を任意に取るとき、
Aに操作Tを施した行列をTAと書くことにすると、*より、
(TAのi行j列成分)=aij+ui+vj
と書けることが分かる。

524 名前:132人目の素数さん:2008/09/04(木) 15:20:26
[STEP2]任意の行列に対し、適当な操作をすることで

000
0ab
0cd

という形に変形できるから、初めからこの形の行列だけを考えればよく、
この形の行列の中で、同値でないものの個数を求めればよい。実は、この形の
行列は全て同値でなく、よって答えは16通りとなる。そのためには、

A   B
000 000
0ab 0ef
0cd 0gh

という2つの行列A,Bが同値であるとしたとき、A=Bとなることを言えばよい。
STEP1を踏まえれば、AとBが同値 ⇔ あるT=(u1,u2,u3)×(v1,v2,v3)に対しTA=B
となるが、TAの各成分とBの各成分を実際に比較すると、u1=u2=u3=v1=v2=v3
となることが分かるので、u1=0であってもu1=1であってもTA=Aとなることが
分かり、A=Bが従う。

525 名前:132人目の素数さん:2008/09/04(木) 15:22:23
>ti=1 ⇔ 「i行目の0と1を入れ替える」(i=1,2,3)
>uj=1 ⇔ 「j列目の0と1を入れ替える」(j=1,2,3)

↓訂正

ui=1 ⇔ 「i行目の0と1を入れ替える」(i=1,2,3)
vj=1 ⇔ 「j列目の0と1を入れ替える」(j=1,2,3)

526 名前:132人目の素数さん:2008/09/04(木) 20:07:36
>>521

「行列が同値 ⇒ 行列Aの任意の2x2小行列の中の1または0の個数の偶奇は不変」
なので3x3行列の4隅の偶奇のパターンだけつまり、2^4=16種類存在する。

4x4行列の場合は独立な2x2小行列が何個あるのかな?4隅と真ん中の1個で2^5種類?

527 名前:132人目の素数さん:2008/09/04(木) 23:52:23
ここでちょっと雑談を。
なんで数学の問題を面白いと感じたり詰まらないと感じたりするんだろう。
面白い問題と詰まらない問題の間にはどんな差があるのか?



528 名前:132人目の素数さん:2008/09/05(金) 00:05:48
つまらない問題
解法が自然に分かる、総当りで解ける、問題文がやけに長い

面白い問題
解法が非自明、視点を変えるとあっさり解ける、問題文が簡潔

こんなところか

529 名前:132人目の素数さん:2008/09/05(金) 00:25:09
ただ、問題の「良い」解き方、汎用性の高い解き方、
というのはアクロバティックな解き方じゃなくて、
少々証明が長くなっても非自明な命題を自明なステップに分解して
こつこつ進んでいく「詰まらない」解き方だったりするんだよね。

530 名前:132人目の素数さん:2008/09/05(金) 00:35:27
こつこつ進んでいくって言うのは自分の手持ちの思考方法が通用してる間の場合だよね。
自分の手持ちの思考方法がどれ一つ全然通用しなくなってこれ以上一歩も進めなくなったときこそ
数学者としての真価が問われるというか。



531 名前:132人目の素数さん:2008/09/05(金) 01:02:04
>>521
こういう問題は掃き出し法で機械的に解けるのだ。

532 名前:132人目の素数さん:2008/09/05(金) 01:16:12
>>530
つまり俺には数学をやる資格がないんですね、分かります

533 名前:132人目の素数さん:2008/09/05(金) 02:23:42
> 総当りで解ける

総当たりで解けることはわかっていても
そうでない方法で解けるかもしれなさそうな問題は面白いぞ。

534 名前:132人目の素数さん:2008/09/05(金) 03:00:02
総当りで原理的には解けるけど
実際は計算時間的にほとんど無理、みたいな問題もあるよね。

ルート2の10進小数展開の小数第 1 位から 100,000,000 桁までに
60,00,000 桁以上同じ数字が連続して並ぶことは無いことを
(電子計算機を使わずに)示せ、とか。

535 名前:132人目の素数さん:2008/09/05(金) 03:07:04
このスレに良く出てくる虫食いみたいな奴のことを言ってるんじゃないの?

536 名前:132人目の素数さん:2008/09/05(金) 07:51:59
>>526
4×4の場合は2^9通り。523〜524と同じ方法が使える。
というか一般のn×nでも使えて、2^{(n−1)^2}通りになる。

537 名前:132人目の素数さん:2008/09/05(金) 10:24:04
>>526
n×n 行列を F_2 上の n^2 次元ベクトル空間と考える。
与えられた U1,...,Un,V1,...,Vn を生成系とする部分空間 W がこのベクトル空間に作用すると考える。

U1+...+Un+V1+...+Vn=0 だから、部分空間 W の次元は 2n-1 以下。
一方、n×n 行列の 1 行目と 1 列目のなす部分空間の次元は 2n-1 で、
W はこの部分空間に可移に作用しているので、W の次元は 2n-1 以上。

よって、2^(n^2)/2^(2n-1)=2^((n-1)^2) が求める同値類の数。

538 名前:132人目の素数さん:2008/09/07(日) 22:27:32
M 個の石の山と N 個の石の山がある。
二人で交互に一度ずつ石を取っていく。
片方の山から石を取るか、或いは両方の山から同数ずつ石を取れ、
最後の石を取ったほうが負けとなる。

後手必勝となるのはどのような場合か?

539 名前:132人目の素数さん:2008/09/08(月) 16:13:22
M〜Nがある数値の時

540 名前:132人目の素数さん:2008/09/08(月) 18:19:20
M〜Nって何?M-Nなら分かるが。

541 名前:132人目の素数さん:2008/09/08(月) 20:48:16
MとNで大きい方から小さい方を引く

542 名前:132人目の素数さん:2008/09/08(月) 23:17:32
違う。(0,1)は後手必勝なので、
(M,M+1) (M, 1) (1, M)但し M ≧ 1 は先手必勝。
そうすると(2,2)はそこからどのような手をとっても
次の手番に先手必勝の状態にしかならないので後手必勝。なので
(M+2, M+2) (M+2, 2) (2, M+2)は先手必勝。
そうすると(3, 5) (5. 3)はそこからどのような手をとっても
次の手番に先手必勝の状態にしかならないので後手必勝。なので(以下略

543 名前:132人目の素数さん:2008/09/08(月) 23:20:18
一応ageて宣伝してみよう

544 名前:132人目の素数さん:2008/09/09(火) 04:10:32
N, M ≦ 100 の範囲で後手必勝になるものの一覧:

(0,1) (1,0) (2,2) (3,5) (4,7) (5,3) (6,10) (7,4) (8,13) (9,15)
(10,6) (11,18) (12,20) (13,8) (14,23) (15,9) (16,26) (17,28) (18,11) (19,31)
(20,12) (21,34) (22,36) (23,14) (24,39) (25,41) (26,16) (27,44) (28,17) (29,47)
(30,49) (31,19) (32,52) (33,54) (34,21) (35,57) (36,22) (37,60) (38,62) (39,24)
(40,65) (41,25) (42,68) (43,70) (44,27) (45,73) (46,75) (47,29) (48,78) (49,30) (50,81) (51,83) (52,32) (53,86) (54,33) (55,89) (56,91) (57,35) (58,94) (59,96)
(60,37) (61,99) (62,38) (65,40) (68,42) (70,43) (73,45) (75,46) (78,48) (81,50)
(83,51) (86,53) (89,55) (91,56) (94,58) (96,59) (99,61)

545 名前:544:2008/09/09(火) 04:12:19
× ≦ 100
○ < 100

546 名前:132人目の素数さん:2008/09/09(火) 08:41:54
お、頑張ったw

で比とかを取って見れば大体一定値になるのではないか?と
予想が付くよね。でそれを証明。

547 名前:132人目の素数さん:2008/09/09(火) 10:06:06
最終局免から帰納法でやってみようか

548 名前:132人目の素数さん:2008/09/10(水) 01:46:51
答えだけ書いとくと、

a = (-1 + √5)/2 = 1.61803398874989484820458683436564 = (-1 + √5)/2
(黄金比の大きいほう)、[ ]をガウス記号(整数部分)として

(0,1),(2,2),
([na], [na] + n)及びその逆のときに後手必勝、その他のとき先手必勝となる。

549 名前:132人目の素数さん:2008/09/10(水) 07:26:13
有名問題: Wythoff game

550 名前:132人目の素数さん:2008/09/10(水) 07:35:24
>最後の石を取ったほうが負けとなる。
ってのが微妙に改題してるわけだけどね。

551 名前:132人目の素数さん:2008/09/10(水) 08:07:30
misere にしたところで小さなところのGN関数書けば一致するのはすぐ見える。

けど、面白いのは確かだね。

552 名前:132人目の素数さん:2008/09/10(水) 20:11:49
2の常用対数を0.30103、3の常用対数を0.47712とします。
この2値を元に、7の常用対数になりうる値の範囲をなるべく正確に求めてください。


log21 = log3+log7 > log2+1 = log20
∴log7 > log2+1-log3 = 0.82391

553 名前:132人目の素数さん:2008/09/10(水) 20:46:01
2^28073<7^10000<2^28074


554 名前:132人目の素数さん:2008/09/10(水) 23:21:08
あれ、なんか見たことある気がする
どこの問題?

555 名前:132人目の素数さん:2008/09/14(日) 08:28:33
四角形abcdで
∠bac=30
∠cad=20
∠adb=105
∠bdc=35のとき、
∠dbc=?

(求め方も詳しく答えて下さい)

556 名前:132人目の素数さん:2008/09/14(日) 09:38:33
誰か面白い問題して。そのとき序でに『面白い問題』の定義も。

557 名前:132人目の素数さん:2008/09/14(日) 11:45:40
>>555
21.389°とかにならない?

558 名前:132人目の素数さん:2008/09/14(日) 21:15:35
>>557
解説ください

559 名前:132人目の素数さん:2008/09/14(日) 21:28:53
30゚じゃないか?

560 名前:132人目の素数さん:2008/09/14(日) 21:58:46
>>555
AB上に∠ADE=80°になるように点Eをとる。
∠DCA=∠DAC=20°よりDC=DA
∠DAE=∠DEA=50°よりDA=DE
∠EDC=60°でDC=DEなので、DC=DE=EC、∠CED=60°
∠EDB=∠EBD=25°よりDE=EB
よって、EB=EC
∠BEC=70°より∠EBC=∠ECB=55°
∠DBC=30°

561 名前:132人目の素数さん:2008/09/14(日) 22:00:04
また凧?

562 名前:132人目の素数さん:2008/09/14(日) 22:01:46
>>555
出典おせーて

563 名前:132人目の素数さん:2008/09/14(日) 22:07:50
>>560の解法見て感動した
1本の補助線であとは2等辺三角形がいっぱい
すごいわ

564 名前:132人目の素数さん:2008/09/14(日) 22:20:10
ttp://www.himawarinet.ne.jp/~rinda/framepage1.html

ラングレーの問題とか、フランクリンの凧って言われるたぐいの問題だよ
多分な。問題読んでないからわからんwww

565 名前:132人目の素数さん:2008/09/14(日) 23:05:49
>>563
> 1本の補助線であとは2等辺三角形がいっぱい

だけ読んで、凧だとおもた。

566 名前:132人目の素数さん:2008/09/14(日) 23:26:42
よく分かったな、見直したぞおまいら

567 名前:132人目の素数さん:2008/09/14(日) 23:28:24
以前ラングレーの問題ばっかり沢山集めて分類してるようなサイト見た事あるなぁ。
初等幾何って奥が深いというかなんというか、変にマニアックなんだよね

568 名前:132人目の素数さん:2008/09/14(日) 23:30:21
>>567
>>564のサイトじゃないのか?

569 名前:132人目の素数さん:2008/09/14(日) 23:30:40
「ずけひろ」ってHPは消滅したの?
受験が終わったらジックリ見ようと思ってたら、なくなっていた。
待つこと数年、未だにみつからんけど

570 名前:132人目の素数さん:2008/09/15(月) 00:10:32
>>568
そうみたい。てか564のリンクまだ見てなかったのよ。今見たらそうだった。

571 名前:132人目の素数さん:2008/09/15(月) 18:58:08
どうせならこんなの

△ABCの内部に点Dがあり、
∠ABD=21°
∠DBC=67°
∠DCB=16°
∠ACD=32°
∠CAD=?
簡単だよね?

572 名前:132人目の素数さん:2008/09/15(月) 20:44:39
>>571
さすがにちょっとハードルを上げすぎたので、修正。

△ABCの内部に点Dがあり、
∠DAB=7°
∠ABD=21°
∠DBC=67°
∠DCB=16°
∠CAD=?

前のと照らし合わせれば、答えはバレてしまうわけだが、
そうなることを証明してやってくれ。

573 名前:132人目の素数さん:2008/09/19(金) 18:09:26
こたえまだぁ?

574 名前:132人目の素数さん:2008/09/19(金) 19:12:15
>>573
kingにでも聞けば?

575 名前:KingMind ◆KWqQaULLTg :2008/09/19(金) 21:08:57
Reply:>>574 どうしろという。

576 名前:132人目の素数さん:2008/09/19(金) 21:18:00
>>king
上にある未解決な問題の解法について質問されているので
余裕があれば教えてあげれば良いのではないでしょうか

577 名前:132人目の素数さん:2008/09/20(土) 08:24:26

http://academy6.2ch.net/test/read.cgi/philo/1216049566/541-1001

よかったら物理学/数学/情報工学その他の方からの意見、情報、修正、整理、その他募集します。
もちろんこのスレでもレスOKです。

文系の混乱した思考をスパッと解明してください。

追伸

具体的には、エンドゲーム(特に将棋)でこれが解決できないか考え中です。

578 名前:577:2008/09/20(土) 10:41:40
追伸

http://science6.2ch.net/test/read.cgi/sci/1159171809/302-1001

上記スレも参照される事を願います。物理板のスレです。

物理が神の運動を法則化したとしてその動きを悪用するブルジョアのド阿呆を封じ込めるために、
将棋というエンドゲームの「取った駒を活用する」という考え方と、
王将(玉将)を絶対取らない(殺さない)という仕組みを「数学的倫理」として示せれば、
地球に格差や戦争が起こってもこれで解決できはしないかという妄想がありますが、
私の力量ではとても理論化できないです。

力を貸してください。

579 名前:132人目の素数さん:2008/09/20(土) 11:26:03
はい、はい。

580 名前:132人目の素数さん:2008/09/20(土) 13:27:24
>>578
よしんばそのような理論が構築できたとして
同じ仮定の下で誰が実践してくれるんだ?

581 名前:132人目の素数さん:2008/09/20(土) 16:37:54
石の山が一つあり、二人が交互に石を取っていく。
最後に石を取ったほうが勝ちである。
最初の人は一つ以上の石を取る。ただし全部の石を取ることは出来ない。
次からは交互に一つ以上で、前の人が取った数の二倍以下の石を取る。

石の数が n ≧ 4 のとき、先手必勝であることを示せ。

582 名前:132人目の素数さん:2008/09/20(土) 17:50:19
>>581
どうみてもn=5で後手必勝なのだが...

583 名前:132人目の素数さん:2008/09/21(日) 02:59:58
>>581
n>4 の場合、残った石の個数をフィボナッチ数に「した」方が勝ちで、「された」方が負け。
正確には、残りがフィボ個の状態で手番を「渡された」プレーヤーは、直後に自分が全部を
取れない限り、負ける。つまり初期値が4以上のフィボ個だったら後手必勝。

略証
f(1)=2, f(2)=3, f(3)=5, ,,, ,f(k)=f(f-1)+f(k-2) とする。
n=f(1)=2, n=f(2)=3のときは主張は正しい。i.e. その時点で手番を持っている方は、
そこで全部取れない場合は負け。n=f(2),f(3),,,,f(k-1) の全てでそうだと仮定する。

さて、現在f(k)個の石が残っていて、プレイヤーAの手番だとする。
これを直近のフィボ数 f(k-1) にした者が勝ち。それにはf(k-2)個の石を取ればよい。
しかしいきなりf(k-2)個を取ると、次の手番で相手Bに残り全部を取られて即死するから、
それはできない∵ f(k-1) < 2f(k-2)。
つまりこれは、残りがf(k-2)個のゲームと同等であるが、仮定から、
それは相手Bの勝ちであるゆえ、Aは負ける。■

残りがフィボ数以外の時に全て先手必勝になるかどうかは、これだけではわからない。

584 名前:132人目の素数さん:2008/09/21(日) 07:10:29
>>583
n (≧4)がフィボナッチ数なら後手必勝、それ以外は先手必勝と言えそうです。

自然数nについて、次のような「フィボナッチ数展開」とでも言うものを
考える。(世に知られているものがあるかどうかは知らないので、仮に。)

f(1)=1, f(2)=2, f(j)=f(j-1)+f(j-2) (j≧3)とする。

任意の自然数nは、有限個数のフィボナッチ数の和として、
次のような形に1通りに表される。(証明は略)

n=f(p_1)+f(p_2)+…+f(p_k)
ただし、p_j(j=1,…,k)は自然数で、
2≦j≦kにおいてp_j≦p_{j-1}-2を満たす。
(つまり、{p_j}は単調減少で、なおかつ、隣り合う数字の差は2以上)

必勝法
・nがフィボナッチ数以外で、先手の時
 常に、残り個数のフィボナッチ数展開の最小項の個数だけ取ればよい。
 (そうすると、相手はフィボナッチ数展開の最小項は取れず、
  次の自分の番では必ずまたフィボナッチ数展開の最小項が取れる。)
・nがフィボナッチ数で、後手の時
 1手目で相手が1/3以上取った時、残りを全部取ればよい。
 それ以外の場合は、上記と同様。

585 名前:132人目の素数さん:2008/09/21(日) 10:56:00
>>583
> つまりこれは、残りがf(k-2)個のゲームと同等であるが、仮定から、
> それは相手Bの勝ちであるゆえ、Aは負ける。■
f(k-2)個のゲームが終わった次の手番で、f(k-1)個を取ることができると
Aが勝つから、取れないことを言わないと不十分では?
(簡単に言えそうだけど)

586 名前:577:2008/09/21(日) 11:42:28
文系的発想からひとつ

将棋において、王将は、正確には「王」(格上)と玉(格下)があります。
さらに、ゲーム開始までに、どちらが上座に座るか、下座に座るかに単なる(格)だけでない駆け引きや気遣いがあります。

もし将棋の順位戦がピラミッドだとしたら(エジプトを想起せよ)、そのピラミッドが二等辺三角形として、もう一つの二等辺三角形を「デモス(民衆)」の位相と考えれば、正方形の地図に本来的な政治と民衆の関係がまとめられませんか?

あと石のゲームに、この石だけは取ったとしても、「相手には返さないといけない」か「(「相手と私がその石と交換しても良いと合意した石」)を代わりに返さないといけない」代わりに、勝った側はただ名誉か生活保障が与えられる仕組みを作れれば、もしかして
これが石器時代から始まる貨幣の歴史に戻る事で「デモス」と「官僚+代議士制度」の戦いが「ポスト資本主義」として、「代理戦争」の仕組みにならないでしょうか?

「デモス」に関しては本日付朝日新聞書評欄の柄谷行人の書評とその本をご覧になられてください。

また思いついたら参加します。

587 名前:577:2008/09/21(日) 11:55:37
さらにひとつ

地球の地図をメルカトル図法としてそれを正方形に圧縮し、その上で右上から左下、あるいは左下から右下に線を引いて、必ず海だけを横切って正方形内にできた二つの領域の面積が同じになるような境界線は引けないでしょうか?

その上で、二つの世界国家の『王将』と『玉将』を取り合うゲーム、あるいはそれを抑止するゲーム。

これを構築するための基本を固めてください。

今日のところはここまで。

588 名前:132人目の素数さん:2008/09/21(日) 14:44:01
>>577 >>586 >>587
巣に戻れ。関係ないスレを荒らすな

589 名前:577:2008/09/21(日) 14:48:33
>>586に問題設定に間違いがありました

王将と玉将は、王を玉=金=貨幣で買う事ではないです。

王将と王将'はGAMEの前に合意ができていて、必ず、それを交換する事。

これが基本だと思います。

そういえば9.11のあと、黒人系でイスラムにも関わりある大統領が生まれつつあるのは、
もしかしたらビン・ラディンが死を賭けて要請した「王将」(駒ではなく王将位の棋士をご想像ください。)の交代ではなかったか??

しかしこのタイム・ラグを解決するには地球の自転に斜線を入れないといけません。

この問題設定に誤りがあれば、また修正、意見等お願いします。

590 名前:577:2008/09/21(日) 14:50:54
>>588
これを荒らしと感じるようでは、
現実から遊離した宗教的=数学的階級の象牙の塔はまるで壊される事がないかのようではないか?
まぁそう思うならそれでもいい。

ならば数学板でこの話につきあえる方、スレッドのご用意あるいは誘導願います。

591 名前:132人目の素数さん:2008/09/21(日) 15:03:08
>>590
思考盗聴厨 1stVirtue とは、何者か?
http://science6.2ch.net/test/read.cgi/math/1218896384/
【信者】KingMind教【限定】
http://science6.2ch.net/test/read.cgi/math/1219587080/
【マダ】Kingと話し合うスレ8【ヤルカ】
http://science6.2ch.net/test/read.cgi/math/1215354212/
kingさんならといてくれるはず
http://science6.2ch.net/test/read.cgi/math/1217001041/
Kingと一緒に数学するスレ
http://science6.2ch.net/test/read.cgi/math/1217063125/

592 名前:132人目の素数さん:2008/09/21(日) 15:16:50
何でわざわざ自演で電波な構ってちゃんの相手をしなきゃあいかんのかって話だな

593 名前:132人目の素数さん:2008/09/21(日) 16:37:21
>>572 の解答例

△ABDの外心をEとする。
円周角の定理より∠DEB=2∠DAB=14°で、
ED=EBより∠EBD=∠EDB=83°
同様に、∠AED=2∠ABD=42°で、
EA=EDより∠EDA=∠EAD=69°
∠BDC=180°-∠DBC-∠DCB=97°=180°-∠EDBより、
3点CDEは同一直線上にある。
点Fが直線EBから見て点Aと逆側にくるように正三角形EFBを作ると、
∠EBC=∠FBC=150°、EB=FBより、
△BCE≡△BCF
∠FCB=∠ECB=∠DCB=16°、∠CFB=∠CEB=∠DEB=14°
∠ECF=∠ECB+∠FCB=32°、∠EFC=∠EFB+∠CFB=74°
∠AEF=∠AED+∠DEB+∠BEF=116°で、
EA=EFより∠EAF=∠EFA=32°
∠EAF=∠ECFより、4点EFCAは同一円周上にあり、
∠EAC=180°-∠EFC=106°
∠CAD=∠EAC-∠EAD=37°

594 名前:132人目の素数さん:2008/09/21(日) 16:42:31
数学板住人にも受け入れられる電波と受け入れられない電波があることがわかった
電波ヲチャを自認する俺も論理的思考能力の欠片も見受けられない>>577みたいなのはダメだわ…

595 名前:132人目の素数さん:2008/09/21(日) 19:52:34
論理的思考力のかけらも見受けられないから駄目って、それ電波ヲチャとして三流もいいところじゃねーか

596 名前:132人目の素数さん:2008/09/21(日) 20:55:11
たしかに俺も>>577はきついな。
ところで、>>594の考える魅力のある電波スレ(できれば現存する奴)を教えてくれ。


597 名前:132人目の素数さん:2008/09/22(月) 09:38:52
論理的思考力のある電波が一番面白い

598 名前:132人目の素数さん:2008/09/22(月) 10:17:27
そらまーただの精神分裂病患者を見ても面白くはない罠(>>577は片足突っ込んでるイメージ)
電波でもそこそこの論理性はもっておいてもらわないと

599 名前:132人目の素数さん:2008/09/22(月) 10:19:01
電波なりの論理展開をしている奴が面白いのだ
論理性や一貫性がないと面白くない。

600 名前:132人目の素数さん:2008/09/22(月) 15:29:56
トンデモをいじるおもしろさは論理の稚拙さを突くことにある。
論理のない戯れ言を見ても「なんだ基地外か」としか思えず、食指が動かないな。

601 名前:132人目の素数さん:2008/09/22(月) 15:39:27
面白い問題まだー?

602 名前:132人目の素数さん:2008/09/22(月) 18:20:33
みんなすごいな……。俺は論理性云々以前にあの文章が日本語に見えなかったんだが……。

603 名前:132人目の素数さん:2008/09/22(月) 19:42:07
日頃から「文(主に問題文)を文字通り解釈する」事をやってるんで
日本文を読む事については自信があるが、
多分他の多くのここの人もそうなんだろう。

604 名前:132人目の素数さん:2008/09/22(月) 21:10:55
>>602
漢字カナかな交りだし、使用している単語も文法も日本からは大きく逸脱していない。
論理構造を考えなければ、十分日本語として言葉になっている。
話し言葉などもそうだが、言語は常に論理構造的に正しいわけではないよ。


605 名前:132人目の素数さん:2008/09/22(月) 22:53:53
皮肉の通じない奴ってつまらないね。

606 名前:594:2008/09/22(月) 23:15:50
>>596
既に知っているかもしれないがいくつか紹介しておく

Rの濃度=R^2の濃度っておかしくね?
http://science6.2ch.net/test/read.cgi/math/1122625618/
→ アレフとアレフヌルの濃度が等しいと激しく主張する電波コテ、あえなく撃沈
  そのあとにも後発の電波がチラホラ、類は友を呼ぶ?

【定理?】負×負=正【定義?】
http://science6.2ch.net/test/read.cgi/math/1162652104/
→ 電波コテ「提唱者」が自説の提唱をひっさげて電波の国からコンニチハ
  コテのあまりの無知さ加減にスレ住人揃って失笑、電波は認識論の最果てへ

リーマン予想考察スレ
http://science6.2ch.net/test/read.cgi/math/1204815723/
→ 解析接続も知らないhirokuro氏がゼータ関数とリーマン予想にもの申す
  hirokuro氏は関数の定義すら知らなかった事が判明、「勉強する」の言葉をのこし姿を消す
  このスレはリーマン予想スレの2スレ目なんだが1スレ目の方が電波度は凄まじかった

四色問題とHadwiger予想。二色目。
http://science6.2ch.net/test/read.cgi/math/1141729305/
→ hadwigerたんによる四色問題の華麗な証明のご披露スレ
 証明の不備を指摘されるも「改善できる、成り立つと思う」の一点張り。結局証明は未完成のまま
 このスレも2スレ目でhadたんが立ち消えてから過疎化が著しい。
 刺激的な電波浴をしたいなら1スレ目をどうぞ

文系だがゲーデルってバカじゃね?
http://science6.2ch.net/test/read.cgi/math/1188624055/
→ 珍解釈、迷解釈のもっとも多いゲーデルの業績について文系の視点からもの申す!
  出オチに近いスレタイでスレを立てた>>1は馬鹿なのはもちろんだが。
  その馬鹿さ加減を上回る電波が現れて…

607 名前:569:2008/09/22(月) 23:41:47
>Rの濃度=R^2の濃度っておかしくね?

お、これ俺も電波側で参戦したやつだw


608 名前:132人目の素数さん:2008/09/22(月) 23:43:12
面白い問題を持ってないなら消えろよ

609 名前:132人目の素数さん:2008/09/22(月) 23:45:29
ごめん、名前間違い。
607=596≠569


610 名前:132人目の素数さん:2008/09/23(火) 00:08:33
これは
「そういうおまえらが一番電波」
というツッコミを入れたら負けというルールのゲームですか?
面白くないので、面白い問題をお願いします。

611 名前:132人目の素数さん:2008/09/23(火) 00:10:58
スレの流れを変えたいなら、自分で問題の一つでも持ってきて紹介すればいいんじゃね?

612 名前:132人目の素数さん:2008/09/23(火) 00:22:36
>>584の「フィボナッチ数展開」って、何か名前がついてますか?

613 名前:132人目の素数さん:2008/09/23(火) 12:38:25
学校でふと思いついて、某所にも1ヶ月程前に書いたんだが華麗にスルーされた問題。

a,b,c:実数とする。ただし常にa+b+c=0
また、上記の複素数を用いて関数f(x)=(a^x+b^x+c^x)/xとする

命題1:整数x,y,zを用いてf(x)*f(y)=f(z)となるような組(x,y,z)は(2,3,5),(2,5,7)以外に存在しない。
命題2:a,b,c:複素数の場合についてはどうか。

614 名前:613:2008/09/23(火) 12:40:13
ミス。
>>613
上記の複素数を用いて→上記の変数を用いて

615 名前:132人目の素数さん:2008/09/23(火) 14:21:43
で、一体何を求めるんだ?

616 名前:132人目の素数さん:2008/09/23(火) 14:35:20
エスパー検定8級の俺によると
命題が正しいことを示せ、じゃないのかな
数学検定は級なしどころのレベルではないのでさっぱり理解できないが

617 名前:132人目の素数さん:2008/09/23(火) 16:24:20
命題の真偽を証明せよって事かと

618 名前:132人目の素数さん:2008/09/23(火) 16:42:52
問1:命題1が真であることを証明せよ。
問2:a,b,c:複素数の場合に命題1と同様の真である命題を作ることは可能か。
 可能ならばその命題(命題2とする)を示せ。

というところか。
華麗にスルーされた理由がよくわかるなw
問題文が書けない。命題という言葉の意味がわかってない。

619 名前:132人目の素数さん:2008/09/23(火) 17:33:20
>>613
どこの国の方ですか? にほん は たいへん でしょうけど ことば には なれてくださいね

620 名前:132人目の素数さん:2008/09/23(火) 19:08:31
全ての自然数nについて
(n^n)/(e^(n-1))≦n!≦(n^(n+1))/(e^(n-1))
が成り立つことを証明せよ。

621 名前:132人目の素数さん:2008/09/23(火) 19:08:32
(1)
任意のa+b+c=0を満たす実数a,b,cに対し、f(x)*f(y)=f(z)を満たす整数x,y,zをすべて求めよ。
ただし、f(x)=(a^x+b^x+c^x)/xとする。

(2)
任意のa+b+c=0を満たす複素数a,b,cに対し、f(x)*f(y)=f(z)を満たす整数x,y,zをすべて求めよ。
ただし、f(x)=(a^x+b^x+c^x)/xとする。

と和訳してみました。(2)の方が簡単に見えるのはおれの眼の錯覚でしょうか?


622 名前:132人目の素数さん:2008/09/23(火) 19:12:30
俺にはどちらも同じくらい難しく見えます。

623 名前:132人目の素数さん:2008/09/23(火) 23:07:52
>>620
n=1 のとき、等号成立。
n≧2 のとき、各辺の対数を考える。
  log(n!) = log(2) + log(3) + ・・・ + log(n),
これを積分で近似しよう。
 f(x) = log(x) とおく。
 f "(x) = -1/(x^2) < 0 だから f は上に凸。割線 ≦ f(x) ≦ 接線。
 ∫[3/2, n+(1/2)] f(x)dx < f(2) + f(3) + ・・・ + f(n) < (1/2){f(2)+f(n)} + ∫[2,n] f(x)dx,
f(x) = log(x) を代入して
 [ x*log(x) -x ](x=3/2,n+(1/2)) < log(n!) < (1/2)log(2n) + [ x*log(x) -x ](x=2,n),
 (n +1/2)log(n +1/2) -(n-1) -(3/2)log(3/2) < log(n!) < (1/2)log(2n) + n*log(n) -n -2*log(2) +2
 (n +1/2)log(n) -(n-1) -(3/2)log(3/2) + (1/2) < log(n!) < (n +1/2)*log(n) -(n-1) -(3/2)log(2) +1 ・・・・・(*)
 (n +1/2)log(n) -(n-1) +(1/2)log(8e/27) < log(n!) < (n +1/2)*log(n) -(n-1) + log(e/√8),
 √(8e/27) * n^(n +1/2) / exp(n-1) * < n! < (e/√8) * n^(n +1/2) / exp(n-1)
 0.80541・・・ * n^(n +1/2) / exp(n-1) * < n! < 0.96105・・・ * n^(n +1/2) / exp(n-1)
これから与式を示される。
*) log(1+(1/2n)) = log((2n+1)/2n) = -log(2n/(2n+1)) = -log(1 - 1/(2n+1)) > 1/(2n+1),

624 名前:132人目の素数さん:2008/09/24(水) 23:06:47
俺の会心の作(と思ってる)>>512がスルーされてるのはなぜディスカ?(TT)
もしかして有名or既出問題だった?





625 名前:132人目の素数さん:2008/09/24(水) 23:13:50
大して面白くもないからだろ

626 名前:132人目の素数さん:2008/09/24(水) 23:18:34
その割には似たような>>516にはレスが付いてるんだぜ?
うー悔しい。


627 名前:132人目の素数さん:2008/09/24(水) 23:32:31
これは”ひねられてる”からだろ

628 名前:132人目の素数さん:2008/09/24(水) 23:33:38
うまいッ…のか?


629 名前:シベリアよりのお手紙:2008/09/25(木) 02:19:32
>>302
9手
o-----*-----*-----*-----*-----*-----*-----*-----*-----*-----
oo----*o----oo----**----*o----*o----*o----*o----**----**----
*oo---ooo---*oo---**o---***---*o*---*o*---*oo---*o*---o**---
oooo--oooo--*ooo--*ooo--*oo*--**o*--**o*--****--***o--****--
ooooo-ooooo-ooooo-ooooo-ooooo-o*ooo-oo**o-****o-****o-*****-

>>509
10進数 (0,1,2,3,4,5,6,7,8,9) に対する解
* 0: 1, 1: 7, 2: 3, 3: 2, 4: 1, 5: 1, 6: 1, 7: 2, 8: 1, 9: 1
* 0: 1, 1:11, 2: 2, 3: 1, 4: 1, 5: 1, 6: 1, 7: 1, 8: 1, 9: 1
2個の解が見つかりました(18.787秒)

>>618
問1: 他にも解があるよ。f(-1)f(3)=f(2), f(1)f(n)=f(1)。これで全部だったけど。

630 名前:132人目の素数さん:2008/09/25(木) 23:39:17
ある男が1万人に1人が発症する重病の検査で、陽性反応が出てしまった。
検査は99%の正確性を誇る。
この病気は、有効な治療法もなく、発病すれば確実に死亡する。
この男が助かる確率はいくらか?

631 名前:132人目の素数さん:2008/09/25(木) 23:52:47
>>630 発症者が間違って陰性と言われる場合もある、でいいんだよな?
発症者が陽性と診断される確率は99%
違う場合は1%。
よって、無作為に選んだ人間が、正しく陽性と言われる確率は(1/10000)*99/100
誤って陽性と言われる確率は(9999/10000)*1/100
∴求める確率は[(9999/10000)*1/100]/[[(9999/10000)*1/100]+[(1/10000)*99/100]]
(めんどいので計算略)

632 名前:132人目の素数さん:2008/09/26(金) 00:01:56
なんで、陽性反応が出てるのに陰性の場合の計算なんかいるんだ?
何万人に1人の病気か知らんけど、陽性が出た奴 100人つれてきたら
99人死ぬんだろ?
こいつが助かるのは残りの1人になるしかないんだから、1% だと思うが。

633 名前:132人目の素数さん:2008/09/26(金) 00:09:12
>>632 違うよ、本来死ぬやつが陽性でないことも考えると少なくとも1じゃないことは明らか。

634 名前:132人目の素数さん:2008/09/26(金) 00:22:58
>>620
積分を使わない別解
 http://science6.2ch.net/test/read.cgi/math/1179000000/553

635 名前:132人目の素数さん:2008/09/26(金) 00:45:56
>>630
前に同じような問題で、確率スレで大荒れに荒れた。
問題は、「99%の正確性」という部分の解釈。
「罹患していないものを確率aで陰性と正しく判定し、確率1-aで陽性と間違って判定する」
「罹患しているものを確率bで陽性と正しく判定し、確率1-bで陰性と間違って判定する」
という2通りの確率が想定でき、その表現でa=b=0.99という意味に解釈できるかという
部分が問題になるが、単に出題側が前提を明らかにすればいいだけのところを
なぜか納得できない奴が出現して、gdgd
そんな中に>>632のような奴も紛れ込んで、発散。

>>632
では、陰性と判断された奴のうちの1%は死ぬのか?

636 名前:132人目の素数さん:2008/09/26(金) 01:13:40
たとえば10億人に一人発症する病気で30%の正確性である場合を考えれば
>>632が間違っていることは直感的にもすぐわかるだろう。

637 名前:132人目の素数さん:2008/09/26(金) 09:28:21
検査を受けた任意の一人が陰性または陽性と判定されているものが合っている確率でないの?


638 名前:132人目の素数さん:2008/09/26(金) 09:42:13
陽性反応が出た人が真に陽性である確率が99%っていう意味ではないの?
んで、真に陽性である人が発症する確率が1/10000ってことではないの?

639 名前:132人目の素数さん:2008/09/26(金) 09:49:15
こういうのの評価法を知っている人は、
真の陽性者がいたときに、その人が陽性といわれる確率が99%、
真の陰性者がいたときに、その人が陽性といわれる確率が1%っていう意味だと思うんだろうけど、
数学の問題としてでたら、>>637-638のように解釈される気もする。

640 名前:132人目の素数さん:2008/09/26(金) 10:49:12
> 真の陽性者がいたときに、その人が陽性といわれる確率 
> 真の陰性者がいたときに、その人が陽性といわれる確率 

このふたつは足して1になるとは限らない。


641 名前:132人目の素数さん:2008/09/26(金) 18:23:48
ほとんど出題の不備だな

642 名前:132人目の素数さん:2008/09/26(金) 22:49:09
周期が2πである関数f(x)を、昇冪の順の整式で表せ。
ただし未定義の点については考慮しない。また、f(x)とは以下で表される。

f(x)=-π/4 (-π<x<0)
f(x)=π/4 (0<x<π)

643 名前:642:2008/09/26(金) 22:50:48
考慮しないってのは、どんな値が来てもいいって意味で使いました。



644 名前:132人目の素数さん:2008/09/27(土) 00:00:00
心理学で出てくる有名問題 >630

645 名前:132人目の素数さん:2008/09/27(土) 00:11:05
この種の問題は一度聞いたことがあるけど
(陽性反応が出たからと言って実際に陽性である
確率は必ずしも高くないという結果が出る)

>検査は99%の正確性を誇る。
ってのはどの出題者もこういう風に表現すんの?

もっとも、そうだとしても不備なのは変わらんけどな。

646 名前:132人目の素数さん:2008/09/27(土) 00:12:28
>>642
整式って言ったら普通は有限項の多項式を意味すると思うんだけど
本当にそれで良いの?

「考慮しない」とかそういう言葉をオリジナルな意味で使ってるようだから
どうもそういう細かい表現をきちんと考えてるとは思えないけど。

647 名前:132人目の素数さん:2008/09/27(土) 05:42:12
重さが相異なる五つの重りA,B,C,D,Eがある。これらを天秤を用いて、重い順に並べなければならない。
(1)天秤の使用回数7回以下で、確実に並べ替え可能であることを証明せよ。
(2)次の条件の時、出された可能性のある結論(並び順)を全て挙げよ。
条件
・天秤使用回数7回以下でソート可能な手順を用いた
・最初の3回は、A>B,C>D,A>Cという結果が出た
・天秤の使用回数6回で結論が出た

648 名前:642:2008/09/27(土) 11:10:56
>>646
すまん、項数が有限じゃない物も普通は整式って言うと思ってた。
大学出ておいたほうが良かったな・・・

649 名前:132人目の素数さん:2008/09/27(土) 12:22:44
>>647
色々考えてみたが

天秤のみを使うなら8回の比較が必要で、
最も重い錘と最も軽い錘を手で持った時にどちらが重いか分かるなら7回で十分。

という考察結果になったorz

650 名前:132人目の素数さん:2008/09/27(土) 13:55:24
>>645
俺が以前見た問題では、誤った陽性反応が出る確率と書いてあったよ。

651 名前:132人目の素数さん:2008/09/27(土) 15:22:24
ABDEが平行四辺形のとき?は何度になるか
http://p.pic.to/tt3ch

652 名前:132人目の素数さん:2008/09/27(土) 16:13:22
三角関数使って長さ測っていけば一発で終わりだな……
初等幾何の問題なんだろうけど

653 名前:132人目の素数さん:2008/09/27(土) 20:56:51
>>652
直線ADに対してEと対称の位置にFをとると
△DEFは正三角形。
∠DAF=∠DAE=∠ADBで、
AF=AE=DBなので、
四角形AFBDは等脚台形
∠ABF=∠BAD=∠EDA=30°
∠ABE=∠DEB=15°
よって∠FBE=45°
また、∠FEB=∠FED-∠DEB=45°
よって、FB=FE
∠BFD=∠FDA=∠EDA=30°でFB=FDより
∠FBD=75°
∠EBD=75°-45°=30°
∠BEA=∠EBD=30°

654 名前:132人目の素数さん:2008/09/27(土) 21:01:34
ではそれに関連して。

長方形でも菱形でもない平行四辺形であり、
4辺と2本の対角線を加えた6本の直線が互いになす角が全て度数法で有理数となるのは
>>651の形だけであることを証明せよ。

655 名前:132人目の素数さん:2008/09/28(日) 20:35:45
>>631
これ計算すると約99%になるから、要するに陰性が出たときの検査の正確さが
わからないような検査は、あんまり信頼性がないってことだよね。


656 名前:132人目の素数さん:2008/09/28(日) 21:07:29
>651

単純にAD間から点を垂直に二等辺三角形になるように伸ばして
180−(90+45+15)=30
ではダメですか?

低学歴の通りすがりの図形好きです。

657 名前:132人目の素数さん:2008/09/28(日) 21:10:05
AD間ではなくAE間でしたm(__)m

658 名前:132人目の素数さん:2008/09/28(日) 22:24:32
>>656
ごめん、わかるように書いて。

659 名前:132人目の素数さん:2008/09/28(日) 22:30:24
>>656
AEの中点をFとして、AEに垂直になるようFから直線をひき、その直線上に点Gを取って、△AEGが直角二等辺三角形になるようにするってこと?





















そうだとしたら、全然だめ

660 名前:132人目の素数さん:2008/09/28(日) 22:33:02
>>656
点を垂直に伸ばす・・・?

661 名前:132人目の素数さん:2008/09/28(日) 22:35:10
656ではないけれど

辺DE上にCD=CFとなるような点Fをとる
このとき三角形ACFは正三角形、また角度を見ればEF=CF
従って三角形AFEは直角二等辺三角形、よって角AECは30度

一般に、15度のところをx度、30度のところをy度、?のところをz度として
zをxとyを用いて表せ、を考えてるんだが酔ってて分からん

662 名前:655:2008/09/28(日) 23:01:30
ちょっと>>655は無視してちょ。勘違いしてた。

663 名前:132人目の素数さん:2008/09/28(日) 23:02:00
>>659の解釈は間違ってるかな?

664 名前:132人目の素数さん:2008/09/28(日) 23:37:04
656です

自分で読んでも意味がワケワカメでしたのでピクトを使いました。
言葉足らずスイマセンでしたm(__)m

http://i.pic.to/temes

665 名前:132人目の素数さん:2008/09/28(日) 23:42:56
663さん。
微妙に違ってましてFEGを作るはずだったのですが説明が無茶苦茶でしたね(´・ω・`)
詳しくは一つ上のレスを見ていただければ有り難いです。

皆さん混乱させて申し訳ないですm(__)m

666 名前:132人目の素数さん:2008/09/28(日) 23:52:14
PCから画像が見れなくてゲンナリ
必ずそのような補助線が引けるとは限らなくて更にゲンナリ

667 名前:132人目の素数さん:2008/09/29(月) 00:35:07
まあ、既に>>653>>661の2通りの解も出たことだし、
>>665のことはそっとしといてやろう...

668 名前:132人目の素数さん:2008/09/29(月) 00:42:01
>>659
その無駄な空白に憤りを感じるのだが!

669 名前:132人目の素数さん:2008/09/29(月) 01:08:47
点FからAEに対して垂直に直線を引くところまでは理解できたんだが
FE=FGなる点Gを作った時にそれがED上に来るかどうかは証明が必要かと

もし来なければ、全く意味のない補助線を引いたことになるからな

670 名前:132人目の素数さん:2008/09/29(月) 01:20:21
>>653の方法でも>>661の方法でも一般の平行四辺形では解けないのだが

671 名前:132人目の素数さん:2008/09/29(月) 01:30:30
>>670
だからなに?

672 名前:132人目の素数さん:2008/09/29(月) 01:32:33
俺は670じゃないが

>>671
>>654が解けない

673 名前:132人目の素数さん:2008/09/29(月) 01:32:52
いや、誰か解いてくれないかなーと思って
そんなにカリカリするなよ

674 名前:132人目の素数さん:2008/09/29(月) 01:42:15
>>654はさすがに初等幾何ではなく代数的に考えるんだろうな

675 名前:132人目の素数さん:2008/09/30(火) 00:57:05
これが日教組の算数の授業だ!

1 スレ立て代行 New! 2008/09/29(月) 07:21:52 神 ID:e5OaSCfm0● BE:?-DIA(120000)
http://img.2ch.net/ico/u_shingi.gif
日教組HPの小学生向け算数教室
http://www.jtu-net.or.jp/education/sansu/series/08.html


676 名前:132人目の素数さん:2008/09/30(火) 08:29:50
子供が、戦闘機の感想しか言わないのがなさけない。
教育に、そして数学に思想を入れるなとは言わないが、せめて
「1あたりの数を習うったら、戦闘機が無用に早いことがわかった。
 1あたりの数、超便利。算数、すげー大事。戦闘機いらない。」
くらいの感想が出るような使い方にしておけばいいのに
これじゃあ、偉大なる将軍様の下さった教室の広さのほうがマシだろ。

数学が、科学が、思想のために使われるのはいっこうに構わないが
これでは、子供が科学離れを起こすのも無理はないな。


677 名前:132人目の素数さん:2008/09/30(火) 08:32:44
感想を書いたのも大人

面白い問題まだー?

678 名前:132人目の素数さん:2008/09/30(火) 10:05:38
>>647(2)のヒント
3回の比較終了時点で可能性の残っている並び順を全て列挙してから、
次に比較すべき重りがどれとどれなのか検討せよ。

679 名前:132人目の素数さん:2008/09/30(火) 14:55:16
>>675
> 嘉手納町が東京(とうきょう)より混(こ)んでいるなんて、信(しん)じられない。 

信じなくていいですよ。  嘉手納町の人口密度は基地面積を差っ引いても
東京にある武蔵野市の40%です。 武蔵野市は東京と下では閑静な住宅街が続く
ゆったりとした街ですが、嘉手納町の2.5倍もの密度で人が住んでいます。
 

680 名前:132人目の素数さん:2008/09/30(火) 16:18:52
この問題、かなり難しい。
http://detail.chiebukuro.yahoo.co.jp/qa/question_detail/q1119495251

681 名前:132人目の素数さん:2008/09/30(火) 16:39:23
>>647(2)の回答(前半)
A>C>D、A>Bより、考えられる並び順は以下の15通り。
E>A>B>C>D,A>E>B>C>D,A>B>E>C>D,A>B>C>E>D
A>B>C>D>E,E>A>C>B>D,A>E>C>B>D,A>C>E>B>D
A>C>B>E>D,A>C>B>D>E,E>A>C>D>B,A>E>C>D>B
A>C>E>D>B,A>C>D>E>B,A>C>D>B>E
あと高々4回の比較で全てを特定しなければならないので、1回目の比較でこの数を8:7に分割する。
7の方に分類された物をさらに4:3に分割し、3の方に分類されたものを2:1に分割した時、
1の方に分類されたものは計6回の比較で特定された事になる。
最初の1回で8:7に分割可能な比較方法はCとEとを比較した場合のみ。
A>C>D、A>B、E>Cとした場合に考えられる並び順は以下の7通り。
E>A>B>C>D,A>E>B>C>D,A>B>E>C>D,E>A>C>B>D
A>E>C>B>D,E>A>C>D>B,A>E>C>D>B
2回目の比較でこれを4:3に分割可能な比較方法はEとAとを比較した場合(i)、BとCとを比較した場合(ii)の
2通り。
(i)E>A>C>D,A>Bとした場合に考えられる並び順は以下の3通り
E>A>B>C>D,E>A>C>B>D,E>A>C>D>B
(ii)A>B>C>D、E>Cとした場合に考えられる並び順は以下の3通り
E>A>B>C>D,A>E>B>C>D,A>B>E>C>D
よって6回目の比較で結論が出たとすると、その並び順は多く見積もっても以下の4種類のどれかである。
E>A>B>C>D,E>A>C>D>B,E>A>B>C>D,A>B>E>C>D
ここで「多く見積もって」と書いたのは、この比較方法で比較していない残りの部分について、必ず計7回目までで
比較が終了する事を示していないためである。

682 名前:132人目の素数さん:2008/09/30(火) 16:41:35
>>647(2)の回答(後半)
前半の順序で比較した場合に、片割れが必ず7回目の比較までにソートが終了する事を示す。
(i)最初に分割した、残りのC>Eの部分は以下の8通り。(A>C>D、A>B、C>E)
A>B>C>E>D,A>B>C>D>E,A>C>E>B>D,A>C>B>E>D
A>C>B>D>E,A>C>E>D>B,A>C>D>E>B,A>C>D>B>E
まずDとEとを比較する。
(i-i)D>Eの場合
A>C>D>E、A>Bより考えられる組み合わせは以下の4通り。
A>B>C>D>E,A>C>B>D>E,A>C>D>B>E,A>C>D>E>B
ここでBとDとを比較すると2:2に分割されるため、明らかにソート可能。
(i-ii)E>Dの場合
A>C>E>D、A>Bより考えられる組み合わせは以下の4通り。
A>B>C>E>D,A>C>B>E>D,A>C>E>B>D,A>C>E>D>B
ここでBとEとを比較すると2:2に分割されるため、明らかにソート可能。
(ii)2回目に分割した残りの部分について示す。(ii-i)ではAとEとの比較結果、(ii-ii)ではBとCとの比較結果について扱う。
(ii-i)2回目に分割した、残りのA<Eの部分は次の4通り
A>B>E>C>D,A>E>B>C>D,A>E>C>B>D,A>E>C>D>B
ここでBとCとを比較すると2:2に分割されるため、明らかにソート可能。
(ii-ii)2回目に分割した、残りのC>Bの部分は次の4通り
E>A>C>B>D,A>E>C>B>D,E>A>C>D>B,A>E>C>D>B
ここでAとEとを比較すると2:2に分割されるため、明らかにソート可能。

結論:(前半)の順序と方法で分割を行った時に全ての場合において7回目の比較まででソートが可能である事が示せた。

答え:E>A>B>C>D,E>A>C>D>B,E>A>B>C>D,A>B>E>C>D

683 名前:681-682:2008/09/30(火) 16:42:46
これで合ってますか?>>647

684 名前:132人目の素数さん:2008/09/30(火) 18:49:25
>>680
> 問 原点0から出発して、数直線上を通る点Pがある。
> 点Pは、硬貨を投げて表が出ると+2だけ移動し、
> 裏が出ると-1だけ移動する。
>
> このとき、
> 点Pが座標3以上の点に初めて到着するまで
> 硬貨を投げ続ける。
> このとき、投げる回数の期待値を求めよ。

(略解)
a[n] を
座標 3-n に居るときの、コインを投げる期待回数
とする
a[0] = a[-1] = 0,
a[n] = 1 + (1/2)(a[n-2] + a[n+1])  (n≧1)
が成立し、これを解くと
a[n] = 2n + (3-√5)(1 - ((1-√5)/2)^n)
求める期待値は
a[3] = (4√5) - 2 = 6.94427191

685 名前:132人目の素数さん:2008/09/30(火) 18:53:28
× が成立し、これを解くと
○ が成立し、これの a[n]→0 (n→∞) となる解を求めると

686 名前:684:2008/09/30(火) 19:10:35
訂正になってなかった

a[n] = 1 + (1/2)(a[n-2] + a[n+1])
の一般解は A,B,C を任意定数として
a[n] = 2n + A + B*((1+√5)/2)^n + C*((1-√5)/2)^n
となる。
a[n] = O(n) のはずだから B=0 となる解を求めればよい

687 名前:132人目の素数さん:2008/09/30(火) 20:24:23
>>680
「数学Aの確率の問題です」という時点で釣りだろう。
答えが1より小さいってのもナメくさっとるw
(投稿者が釣られた結果なのかもしらんが)

688 名前:132人目の素数さん:2008/09/30(火) 20:50:20
>>684
隣接4項間の漸化式なので、a[1]の値が確定しないと
それ以上の項が決定できない気がするんだが。

689 名前:132人目の素数さん:2008/09/30(火) 21:29:45
>>688
a[n] = O(n)という情報があるから、初項に関する条件は少し弱められるのでは?

690 名前:132人目の素数さん:2008/09/30(火) 21:40:46
>>686
特性方程式はx^4-2x^3+x+2=0
となり、実数解を持たないわけだが。
>a[n] = 2n + A + B*((1+√5)/2)^n + C*((1-√5)/2)^n
>となる。
の所でダウト。

691 名前:132人目の素数さん:2008/09/30(火) 21:45:20
期待値の計算が苦手な俺に
>a[n] = 1 + (1/2)(a[n-2] + a[n+1])
この式が成り立つ理由を教えてくれ(´・ω・`)

692 名前:132人目の素数さん:2008/09/30(火) 21:45:54
あ、全然違った。
>>690は無視してちょ

693 名前:132人目の素数さん:2008/09/30(火) 21:59:15
>>690
a[n] = (1/2)*(1+a[n-2]) + (1/2)*(1+a[n+1])
と書いた方が分かり易いかもしれない。

つまり、今いる位置でコインを振って、
表が出たら期待値が1増えて2つ右に移動、
裏が出たら期待値が1増えて1つ左に移動。

694 名前:647:2008/09/30(火) 23:16:21
>>681

> よって6回目の比較で結論が出たとすると、その並び順は多く見積もっても以下の4種類のどれかである。
> E>A>B>C>D,E>A>C>D>B,E>A>B>C>D,A>B>E>C>D

上の部分をよく見直してみてください。その訂正を以て、正解です。

695 名前:681:2008/09/30(火) 23:30:34
>>647
OK,E>A>B>C>Dが重複してるわ
答え:E>A>C>D>B,E>A>B>C>D,A>B>E>C>Dの3通り

696 名前:681:2008/09/30(火) 23:33:40
comment
678のヒントがなかったらあと1週間位の時間を要求していたと思う。
面白かったよ

697 名前:132人目の素数さん:2008/10/01(水) 22:39:13
>>680
ちょうどn回目に上がるパターン数をf(n)とすると、

f(n) = 0          (n=1 mod3)
    C(n+2, [n/3]+1) (n=0 or 2 mod3)

だな。[ ] はガウス記号。
あとは Σ[1→∞]n*f(n)/2^n の極限値を求めればいいわけだが‥‥
nCrヲタの出現を待つとしよう。

698 名前:132人目の素数さん:2008/10/01(水) 22:42:38
>nCrヲタ

どんなヲタだwww


699 名前:132人目の素数さん:2008/10/01(水) 22:53:39
>>697
ちょうど2回目で上がるパターンは1通りしかないが、その式によると
f(2)=C(4,1)=4となってしまう。

700 名前:132人目の素数さん:2008/10/01(水) 23:08:27
>>698
nCrヲタって群生体で不等式ヲタで三角関数ヲタでもあるらしい…

701 名前:132人目の素数さん:2008/10/01(水) 23:20:36
俺はnCrでどんぶり飯三杯はいけるぜ

702 名前:697:2008/10/01(水) 23:24:16
>>699
指摘サンクス。正しくは以下だった

f(n) = 0             (n=1 mod3)
    C(n+2, [n/3]+1)/(n+2) (n=0 or 2 mod3)

703 名前:132人目の素数さん:2008/10/01(水) 23:46:56
>>701
それは単なる nCr デブだ。

704 名前:132人目の素数さん:2008/10/02(木) 00:21:09
俺はnCrで3回はヌケる

705 名前:132人目の素数さん:2008/10/02(木) 00:23:41
>>704
それは単なるnCrフェチだ

706 名前:132人目の素数さん:2008/10/02(木) 00:57:12
俺はnCrで三回はコケる。

707 名前:697:2008/10/02(木) 01:07:40
ついでに>>684の漸化式を、a[1]=aとして解いてみた。
a[n]-a[n-1]-2 = {a[n-1]-a[n-2]-2} + {a[n-2]-a[n-3]-2}
と変形できるので、3項間に帰着される。結果、
p=(1+√5)/2、q=(1-√5)/2 とおくと

a[n] = 2n + {(2p-a)p^2(1-p^n) - (2q-a)q^2(1-q^n)}/√5

となり、確かに>>686のような形になったものの、やはりa[1]の
値が(定数部分にも)入ってきているため、a[1]の値なしには
a[3]を確定できそうにない。

708 名前:697:2008/10/02(木) 01:32:24
a[n]のオーダーがO(n)になる理由がわからん。
確かにそれを仮定すれば、p^nの項を潰すようにaを決められるな。

709 名前:132人目の素数さん:2008/10/02(木) 03:56:21
>>708
a[n]ってのは、言い換えると、原点から出発して最初にn以上の地点に到達するまでの
回数の期待値だから、たとえばa[100ぐらい]=xぐらいならば、
a[200ぐらい]は、100ぐらいに最初に到達した時点を一区切りとみなすと、
100ぐらいに最初に到達したら終わりという試行を2回繰り返すことを考えればいいので
a[200ぐらい]=2xぐらいと言える。
だから、a[n]のオーダーがO(n)という予想は悪くない。
(a[n]がnにかかわらず無限大に発散するのでないかぎり。)
ただ、オーダーってどうやって証明すればいいんだろう。

ちなみに、1回で+2移動なんてのがなければ(例えばその代わりにプラス方向の確率の方が大きいとかなら)
上記の「ぐらい」は全部消せるのだが。

710 名前:132人目の素数さん:2008/10/02(木) 11:59:43
俺には>>702が成り立つ理由も分からんぜ

711 名前:132人目の素数さん:2008/10/02(木) 19:28:42
一応出来たっぽい。

B ≠ 0 のとき a(n+1) - a(n) は指数函数的に増加。
B = 0 のとき a(n + 1) - a(n) → 2 (n → ∞)。
従ってこれを大雑把に評価すれば良い。

(初期位置の座標 3-n から)
確率 1/2 で -1 進み、確率 1/2 で +2 進むという試行を繰り返す。
k 回の試行のうち、 t 回目の試行までに
-1 が x(t) 回、 +2 が y(t) 回出たとして
試行中常に 2y - x ≦ n-1 となる確率を p(n,k) と置く。
p(n,k)は k に関して単調減少、 n に関して単調増加。
 a(n)
 = 農{k=1}^{k=∞} k(p(n,k+1) - p(n,k))
 = 農{k=1}^{k=∞} p(n,k)。
従って
 a(n+1) - a(n)
  農{k=1}^{k=∞} p(n+1,k) - p(n,k)。
2y(t) - 2x(t) ≦ 2k だから 2k ≦ n-1 つまり k ≦ (n-1)/2 のとき
n が充分でかいから p(n,k) = 1。このとき p(n+1,k) も常に 1 となる。
従って
 a(n+1) - a(n)
 = 農{k=1}^{k=∞} p(n+1,k) - p(n,k)
 = 農{k=1}^{k=[(n+1)/2]} p(n+1) - p(n,k)
 < 農{k=1}^{k=[(n+1)/2]} p(n+1) < [(n+1)/2] < (n+1)/2
ここで 0 ≦ p(n+1) ≦ 1 、[ ]は整数部分。
つまり a(n+1) - a(n) は高々 n の一次の速さでしか増大しないので B = 0。
従って実際は a(n) 〜 2n、a(n+1) - a(n) → 2。   □

Catalan数使ってどうにか出来ないか試したりして
結局帰宅後すぐ取り掛かって今になるまで掛かった。長かったー、、

712 名前:711:2008/10/02(木) 19:36:31
「n に関して単調増加」の直ぐ下を
 a(n)
 = 農{k=1}^{k=∞} k(p(n,k-1) - p(n,k))
              ~~~~~~~~~
に訂正。

713 名前:132人目の素数さん:2008/10/04(土) 02:23:35
>>711
> = 農{k=1}^{k=∞} p(n+1,k) - p(n,k)
> = 農{k=1}^{k=[(n+1)/2]} p(n+1) - p(n,k)
のところ
 = 農{k=1}^{k=∞} p(n+1,k) - p(n,k)
 = 農{k=[(n+1)/2]}^{k=∞} p(n+1,k) - p(n,k)
じゃないのか?

714 名前:132人目の素数さん:2008/10/04(土) 03:16:40 ?2BP(10)
あ、そうかも、、

715 名前:132人目の素数さん:2008/10/04(土) 19:22:32
高3だけど、問題自作してみた。

Oを原点とする座標平面上に、どちらも原点Oではない、
相異なる2点A,Bがある。
線形変換(1次変換)f は、
f (OA↑) = 2*OB↑, f (OB↑) = 3*OA↑を満たすという。
線分ABを直径とする円上の動点Pをf によって写した点をQとすると、
動点Qはどのような軌跡を描くか。 OA↑,OB↑を用いて答えよ。

716 名前:132人目の素数さん:2008/10/04(土) 21:57:16
【例題】sinθ+cosθ=1.5の時、sinθcosθはいくらか?

解答(1)
(sinθ−0.5)の2乗+(cosθ−0.5)の2乗=1.5−(sinθ+cosθ)
sinθ+cosθ=1.5を代入して
(sinθ−0.5)の2乗+(cosθ−0.5)の2乗=0
sinθ=cosθ=0.5
∴sinθcosθ=0.25

解答(2)
(sinθ+cosθ)の2乗=1+2sinθcosθ
sinθ+cosθ=1.5を代入して
1.5の2乗=1+2sinθcosθ
∴sinθcosθ=0.625

解答(1)、解答(2)より0.25=0.625

どこに矛盾があるのか?

717 名前:132人目の素数さん:2008/10/04(土) 22:00:46
仮定

718 名前:132人目の素数さん:2008/10/04(土) 22:09:01
仮定を認めるなら解答(1)の3,4行目

719 名前:132人目の素数さん:2008/10/04(土) 22:26:29
なるほど。sinθ=cosθ=1/2は
(sinθ)^2+(cosθ)^2=1満たさないのか・・・・
勉強になりました

720 名前:132人目の素数さん:2008/10/04(土) 22:29:35
違うだろ
sinθ、cosθが実数とは限らないから解答(1)の三行目からは四行目が得られないんだろ

721 名前:132人目の素数さん:2008/10/04(土) 22:52:16
>>720
同じことだと思うが

722 名前:132人目の素数さん:2008/10/04(土) 22:53:43
>>720-721
単に間違ってるという事にそれ以上の説明が必要なのか。

723 名前:132人目の素数さん:2008/10/04(土) 22:56:03
どこがクリティカルなミスかを正しく理解することは大切だと思います

724 名前:132人目の素数さん:2008/10/05(日) 14:38:10
通りすがりの者ですが>>716の解答(1)で
一行目がさっぱりわかりません
どっからこういう風になるんでしょうか
それとも何か、これは何かの冗談でしょうか

それと関係ないけど2乗や小数表記が気持ち悪いです
そこは別にどうでもいいんですが


725 名前:132人目の素数さん:2008/10/05(日) 14:56:12
カスが口をはさむな

726 名前:132人目の素数さん:2008/10/05(日) 15:00:56
今の言葉取り消してください
カスではなくクズです


727 名前:132人目の素数さん:2008/10/05(日) 15:01:34
いいえ、カスであり、かつクズである。が真。

728 名前:132人目の素数さん:2008/10/05(日) 15:08:06
いずれにせよ、式中に「の2乗」なんて書く奴の書き込みは読む気がしない。


729 名前:132人目の素数さん:2008/10/05(日) 15:18:06
>>724
左辺を展開したまえ。さすれば(sinθ)^2+(cosθ)^2=1を使って整理すれば右辺になる。

わざわざ優しすぎたかな・・・・

730 名前:132人目の素数さん:2008/10/05(日) 15:26:34
>>729
すごく・・・優しいです・・・

731 名前:132人目の素数さん:2008/10/05(日) 15:47:02
>>715
何処が面白いんだ?

732 名前:132人目の素数さん:2008/10/05(日) 16:18:17
多分自分で作った問題を自慢したいだけだと思うよ

733 名前:132人目の素数さん:2008/10/05(日) 17:48:06
俺はむしろ自作の問題を
「それを解けなくて困っている」フリをして質問スレに書いたことがあるぞ
この方がはっきりいって面白い

自分の作問能力の程度が測れるし
他人がどんな解法で攻めてくるのかも楽しみだ


734 名前:132人目の素数さん:2008/10/05(日) 18:02:11
それいいなwwww
さっそく試してみるwwww

735 名前:132人目の素数さん:2008/10/05(日) 18:32:45
じゃあ本当に解けなくて困ってる問題を。

x, y平面状の格子点(n, m) (但しn, m は 0 または自然数)を考える。
原点から右または上のみに 1 ずつ進んで (n, m) まで
到達する方法は (n + m)Cn 通りである。
ここで傾き a が正、y 切片 b も正の直線 y = ax + b を考え、
y = ax + b より下にある点のみを通り、
原点から右または上のみに 1 ずつ進んで (n, m) まで
到達する方法を f(n, m) 通りとする。
このとき、n, m が充分に大きければ f(n, m)/(n + m)Cn は充分小さくなることを示せ。

(つまり、任意の正の実数εに対してある正整数 N が存在して以下を満たす:
n > N かつ m > N ならば f(n, m)/(n + m)Cn < ε)

736 名前:132人目の素数さん:2008/10/05(日) 18:36:37
あ、遅レスだけど
>>584の「フィボナッチ数展開」は普通
Zeckendorf representationと呼ぶね。
まあ584は知ってそうだけど。

737 名前:132人目の素数さん:2008/10/06(月) 00:26:54
>>735
反例があった。a=b=1, m≦nのときf(n,m) / (n+m)Cn = 1-m/(n+1)だからn=m^2としてみる。

738 名前:132人目の素数さん:2008/10/06(月) 09:34:40
正方形を、どの2つも合同でない3つの相似な図形に分割して下さい
ただし切り分ける線の長さはできるだけ短くして下さい

739 名前:132人目の素数さん:2008/10/06(月) 12:21:40
>>738
最短かどうかは知らんが、
正方形の1辺を1とし、x^3-x^2+2x-1=0の実数解をaとすると、
(長辺,短辺)=(1,a),((1-a)/a,1-a),(1-a,a(1-a))
の3つの長方形に分割できる。切断線の合計長は2-a。
ちなみにaは約0.56984 (by Mathematica)

740 名前:132人目の素数さん:2008/10/07(火) 00:47:38
>>733
そういう目で見てしまうじゃねーかwwwwww

741 名前:132人目の素数さん:2008/10/07(火) 22:22:46
1/A+1/B+1/C+1/D+1/E+1/F+1/G+1/H+1/I+1/2007=1
0<A<B<C<D<E<F<G<H<I<2007
A〜Iはすべて自然数。A〜Iはいくらか

742 名前:132人目の素数さん:2008/10/08(水) 02:36:30
>>741
(7,72,168,223,252,446,669,1561,1784)
ふぅ、疲れた。

2007=223*9なので、分母が223の倍数となるものだけを拾うと
1から8までの整数からいくつかを選んだ組{a_n} (n=1,…,k)を使って
Σ[n=1,k](1/(223a_n)) + 1/(223*9) = (1/(2520*223))(Σ[n=1,k](2520/a_n)+280)
となり、このΣ[n=1,k](2520/a_n)+280の部分を223の倍数にする必要があるので、
{2520,1260,840,630,504,420,360,315}のうちのいくつかと280を足して
合計が223の倍数になるパターンを探したら、
2520+1260+840+360+315+280=5575=223*25となり、
{a_n}={1,2,3,7,8}とすると
Σ[n=1,5](1/(223a_n)) + 1/(223*9) = 5/504となった。
あとは、499/504をなんとかすればいいので
499=252+168+72+7ということで。

743 名前:132人目の素数さん:2008/10/08(水) 02:39:05
>>741
>>742の肝心の答えの部分が全然違った。
(2,3,7,72,223,446,669,1561,1784)
が正解。何をやってるんだ、オレ。

744 名前:132人目の素数さん:2008/10/10(金) 10:15:07
>>581 のゲームで「二倍以下」のところを「3倍以下」にしたとき、
後手必勝となるのは最初の石数がどのようなときか?

745 名前:132人目の素数さん:2008/10/11(土) 10:00:15
>この文章内には1が( )個ある
>この文章内には2が( )個ある
>この文章内には3が( )個ある
>この文章内には4が( )個ある

>この文章内には1が(3)個ある
>この文章内には2が(1)個ある
>この文章内には3が(3)個ある
>この文章内には4が(1)個ある
みたいに数字を入れてく問題がいつかあった気がするけど
何スレ目で出てたっけ

746 名前:132人目の素数さん:2008/10/13(月) 18:39:48
>>745
知らぬ
それより問題を出せ

747 名前:132人目の素数さん:2008/10/13(月) 22:24:38
だれかベクトルの問題で難しい奴知ってる?
できれば、高卒〜大学新入生レベルで。

748 名前:132人目の素数さん:2008/10/13(月) 22:57:40
>>747
超有名問題でよければ
一辺の長さが1の正四面体をある平面に正射影したとき、その正射影の面積の範囲を求めよ

749 名前:132人目の素数さん:2008/10/13(月) 23:01:18
>>747
>>715とか

750 名前:132人目の素数さん:2008/10/13(月) 23:10:22
>>715
の問題は、O,A,Bが一直線上にあるときは変換が存在しないな。
おそらく出題者はそういう状況に気づいていないのだろう。
(オレも今気づいたが)

751 名前:132人目の素数さん:2008/10/14(火) 02:03:39
6×6のマスで、対角線上のマスが2つ欠けたマスがある。
このマスを、隣り合う2マスを塗りつぶしていって
34個のマス全てが塗りつぶせない事を証明せよ。

752 名前:132人目の素数さん:2008/10/14(火) 02:11:25
>>751
6×6のマス目をチェス盤のように塗りつぶす。対角線上のマスは同色になるから、以下略。

753 名前:132人目の素数さん:2008/10/14(火) 02:53:37
鳩ノ巣原理のだろ。有名すぎだな。

754 名前:132人目の素数さん:2008/10/14(火) 03:19:17
鳩ノ巣原理なのか?

755 名前:132人目の素数さん:2008/10/14(火) 07:48:06
一応>>752の以下略の部分を突き詰めると鳩ノ巣原理だな。
でもこの問題のキモは>>752の前半部分だ。

756 名前:132人目の素数さん:2008/10/14(火) 08:29:15
>突き詰めると鳩ノ巣原理だな
ほぅ
なんか突き詰めるとほとんどの不等式が鳩ノ巣原理になりそうだな

757 名前:132人目の素数さん:2008/10/14(火) 08:35:39
>>748
それって、[√2/4, 1/2]だったりするのか?

758 名前:132人目の素数さん:2008/10/14(火) 19:27:16
>>752の以下略にはどんな文章がはいるのですか?

759 名前:132人目の素数さん:2008/10/14(火) 19:53:55
0÷0=?と言うもんだいの解に関して考察してます
よかったら見てください。
http://www.nob13.com/game/iappli/BlogTool/m/view.php?username=marques006&year=2008&month=10&day=14

760 名前:132人目の素数さん:2008/10/14(火) 22:31:00
>>759
スレちがい。
http://science6.2ch.net/test/read.cgi/math/1203116217/
へどうぞ。

761 名前:132人目の素数さん:2008/10/14(火) 22:45:53
>>751
塗りつぶせる

762 名前:132人目の素数さん:2008/10/14(火) 23:10:42
>>750
A、B、f、Oが条件を満たす場合を考えてるから
A、B、Oを決めたとき条件を満たすfが存在しなくても
B、f、Oを決めたとき条件を満たすAが存在しなくても
この問題に関係ない

763 名前:132人目の素数さん:2008/10/15(水) 06:28:58
>>739
 a = (1/3){1 + [(√{11^2 + 4*5^3} + 11)/2]^(1/3) - [(√{11^2 + 4*5^3} - 11)/2]^(1/3)}
  = 0.56984029099805326591139995811957・・・・

764 名前:132人目の素数さん:2008/10/15(水) 09:16:49
ある国では人々は生まれてくる子には男の子だけを欲しがりました。
そのため、どの家族も男の子を産むまで子供を作り続けました。
この国では男の子と女の子の人口比率はどうなりますか?

765 名前:132人目の素数さん:2008/10/15(水) 11:32:48
>>764
中絶は一切認めず、男の子を授かる確率と女の子を授かる確率が等しい
という前提ならば、直感的予想に反し、男女比は1:1で変わらない。

数学の問題ではないが、次のようなものもある。

1)中絶を認めず、男の子を授かる確率と女の子を授かる確率は等しく、
ある夫婦が次に子供を作るかどうかの判断は、それまでに生まれた
子供の性別とは独立であるという仮定で考えるとき、
ちょうど2人子供がいる夫婦全体の中から無作為に1組の夫婦を選ぶと
その夫婦の2人の子供の性別が同じである確率は1/2より高いか低いか?
理由も合わせて答えよ。

2)結婚直前のアンケート調査で、
「子供は男女どちらが欲しいですか」という質問と
「男の子」「女の子」「どちらでもない」という選択肢が用意された
項目において、「男の子」と回答した夫婦を集めたグループをA、
「女の子」と回答した夫婦を集めたグループをBとする。
これらの夫婦の10年後を追跡調査し、
各夫婦の子供に女の子が2人以上いるかどうかを調べたところ、
グループAにおいては女の子供が2人以上いる割合はa%、
グループBにおいては女の子供が2人以上いる割合はb%であった。
aとbはどちらが大きいと考えられるか?理由も合わせて答えよ。


766 名前:132人目の素数さん:2008/10/15(水) 15:28:45
>>765
それらが数学の問題でないならなんなの?

767 名前:132人目の素数さん:2008/10/15(水) 15:31:23
算数かな

768 名前:132人目の素数さん:2008/10/15(水) 15:38:31
>>765
> 中絶は一切認めず、男の子を授かる確率と女の子を授かる確率が等しい 
> という前提ならば、直感的予想に反し、男女比は1:1で変わらない。 

何人目の子供でも生まれる男女の比率が同じいう前提でも
男女比は元と変わらない。


769 名前:132人目の素数さん:2008/10/15(水) 16:04:40
>>766
少なくとも2)は数学じゃないと思うけど。
アンケートの回答(各夫婦の嗜好)による
女児の数の偏りなんて数学の問題じゃないと思うんだけど。
少なくとも与えられたデータから数学的に出て来るようなものじゃないと思う。

770 名前:132人目の素数さん:2008/10/15(水) 16:41:53
生まれたり、内診でわかってしまった性別について中絶や殺すなどの操作しない限りは
男女比は変化しないんじゃないか?



771 名前:132人目の素数さん:2008/10/15(水) 16:44:51
>>769
いや、そういう意味でなく(それは数学の問題だろ?という意味ではなく)
「できの悪い数学の問題」でなければ、そんなことをマジに考えてる学問てのは
いったいなんなのかに興味があったんだ。すまん。

772 名前:132人目の素数さん:2008/10/15(水) 17:18:20
1)は、一卵性双生児が生まれる可能性があるので、2人の性別が同じ確率の方が高い。

2)は、男の子が欲しいという夫婦が、男の子が生まれるまでは子供を作ろうとし、
女の子が欲しいという夫婦が、女の子が生まれるまでは子供を作ろうとすると仮定すると、
前者は、女の子しかいないならばまだ子供を作ろうとするので女の子が複数生まれる可能性があるが、
後者は、女の子が1人生まれたら満足するので、女の子は1人で終わる可能性が高い。
もちろん、2人生まれるまでは作り続けるという行動をとる可能性もあるので、一概には言えないが、
実際世の中で、経済的にそんなに楽ではないのに女の子ばかり3人とか作っている夫婦を見ると
「ああ、男の子が欲しかったんだな」と思う。
というわけで、aの方が大きいであろうと予測できる。

1)も2)も、男女構成比が1:1という事実とは矛盾しない。
というわけで、数学ではないでそ。

773 名前:132人目の素数さん:2008/10/15(水) 17:20:56
>>771
すまん、別の何かの学問というわけではなく、
「数学ではない」の真意は、「多湖輝的パズル」って意味だったので...

774 名前:132人目の素数さん:2008/10/15(水) 17:32:34
あげとけ

775 名前:132人目の素数さん:2008/10/15(水) 22:05:15
模範解答見つけた

模範解答
自然体で女の子が産まれる可能性をp(0<p<1)とすると、 1人目で男の子が産まれる可能性は1−p
1人女の子が産まれた後に2人目で男の子が産まれて男の子1人・女の子1人となる可能性はp×(1−p)
2人女の子が産まれた後に3人目で男の子が産まれて男の子1人・女の子2人となる可能性はp^2×(1−p)
n−1人女の子が産まれた後にn人目で男の子が産まれて男の子1人・女の子n−1人となる可能性はp^(n−1)×(1−p)
これを言い換えれば、 子どもが1人だけの場合、男の子1人で、その確率は1−p
子どもが2人だけの場合、男の子1人・女の子1人で、その確率はp×(1−p)
子どもが3人だけの場合、男の子1人・女の子2人で、その確率はp^2×(1−p)
子どもがn人だけの場合、男の子1人・女の子n−1人で、その確率はp^(n−1)×(1−p)


776 名前:132人目の素数さん:2008/10/15(水) 22:06:26
以後同様に考え、この国での男女比は、
男の子の出生数/女の子の出生数=(1−p)+p×(1−p)+p^2×(1−p)+・・・+p^(n−1)×(1−p)+・・・/p×(1−p)+2×{p^2×(1−p)}+・・・+(n−1)×{p^(n−1)×(1−p)}+・・・
分子を(1−p)で、分母をpくくると、
男の子の出生数/女の子の出生数=(1−p)×{1+p+p^2+・・・+p^(n−1)+・・・}/p×[1−p+2×p×(1−p)+・・・+(n−1)×{p^(n−2)×(1−p)}+・・・]
ここで、分母中の大括弧の中身を考え、子どもの数がn人の場合について展開し、
n×p^(n−2)−n×p×p^(n−2)−p^(n−2)+p×p^(n−2)
=n×p^(n−2)−n×p^(n−1)−p^(n−2)+p^(n−1)
=p^(n−2)×(n−1)−p^(n−1)×(n−1)
となり、これにn−1人の場合の、
p^(n−3)×(n−2)−p^(n−2)×(n−2)
とn+1人の場合の、
p^(n−1)×n−p^n×n
を加えたとき、n人の場合の
p^(n−2)についてはn−1人のそれとの合算でp^(n−2)だけが残り
(p^(n−2)×(n−1)−p^(n−2)×(n−2)=p^(n−2))、p^(n−1)についてはn+1人との合算でp^(n−1)だけが残り
(p^(n−1)×n−p^(n−1)×(n−1)=p^(n−1))、
これをすべてのnについて行えば、結局のところ分子の中括弧の中身と同様に、1+p+p^2+・・・+p^(n−1)+・・・といった数列となるので、約分可能。したがって、
男の子の出生数/女の子の出生数=(1−p)/p
であり、自然体と変わらない男の子と女の子の人口比率となる。

777 名前:132人目の素数さん:2008/10/15(水) 22:28:08
まあ、その国で新たに子供が生まれたら必ず「人口管理局」に連絡が入るとして、
その管理局の職員の目線で見れば,
「報告!新生児が誕生しました!」「性別は?」「*であります!」の
*に入るのが男であるか女であるかは、親がどういう経緯で子供をこさえようと思ったかとは
関係のない事象なわけで、>>775>>776の結果になるのは当然ではあるのだけど。

778 名前:132人目の素数さん:2008/10/15(水) 22:42:45
>>763
どうやって線を引けばいいのですか?

779 名前:132人目の素数さん:2008/10/16(木) 00:12:02
>>777
激しくガイシュツ

780 名前:132人目の素数さん:2008/10/16(木) 01:23:08
>>764
短期的にはその国本来の性比(人種や環境に依存する)、例えば105:100になる。
長期的には女の子が生まれやすい遺伝子が次の世代に多く受け継がれるため性比は低下する。

781 名前:132人目の素数さん:2008/10/16(木) 03:06:51
>>780
> 長期的には女の子が生まれやすい遺伝子が次の世代に多く受け継がれるため性比は低下する。 

ここがわからん kwsk

782 名前:132人目の素数さん:2008/10/16(木) 21:44:18
女の子を産みやすい夫婦ほど子供をたくさん作ることになるから。

783 名前:132人目の素数さん:2008/10/17(金) 03:00:36
性を決めるのは両親のうち男の側の遺伝子。
この世界の男の子はどの両親からも一人しか生まれない。
男を産みやすい両親からも、女を産みやすい両親からも。
これで本当に女の子を生みやすい遺伝子は後世に強く残るのだろうか?

男の子が生まれるまで頑張れなかった、両親もいるかもしれないことを考えると
減るかもしれない。

もっとも性を決めるのは男親の遺伝子ではあるが、どちらを受け入れるかは
女親が決めているという考え方はできる。




784 名前:132人目の素数さん:2008/10/17(金) 03:02:07
> 性比は低下する。

比が低下するってのは 比が小さくなる(1:1に近くなる)という意味だと思っていたら
比が大きくなるって意味だったのね。

785 名前:132人目の素数さん:2008/10/17(金) 04:55:00
>>783
そこまで細かいこと考えるんだったら
性決定と適応度の関係は生物学的にかなり難しいのだから
実際にシミュレーションなり実験なりしてみるしかない、としか言いようが無い。

実際統計学的には明らかに男児が生まれる割合の方が
大きいが、そうなる分子生物学的機序など何も分かっていない。

786 名前:132人目の素数さん:2008/10/17(金) 05:08:31
√2より大きく17/12より小さい有理数q/p [p, qは互いに素な正整数] のうちp+qが最小となるものを考えよ

787 名前:132人目の素数さん:2008/10/17(金) 06:39:06
たぶん、>>780が主張しているのは、
例えば、男女比はちょうど1:1であっても、
遺伝形質の中に、男児が生まれやすい(例えばそれが女性の側の
形質だとすると、受精時にY染色体を持つ精子の方に有利に働く
なんらかの条件があるとか)というものと、女児が生まれやすい
というものが存在して、それらのバランスで全体としては男女比
が1:1になっているのだと仮定して、

ある日を境に全国民一斉に
>>764のように「男の子を産むまで子供を作り続ける」という
行動をとるようになったとすると、
結果的に女児が生まれやすい形質を持った女性の方が
平均すると多産となり、
長期的にみると女児が生まれやすい形質の方が多く生き残る、
ということだと解釈した。

ただ、もしそうなら、短期的にも、生まれてくる子供の割合は
女児の方が男児よりも多くなる気がするが...。
(女児が生まれやすい夫婦の方が、2子目を作ろうとする割合が
大きいことになるので。)

つまり、分子レベルのメカニズムうんぬんはともかく、
全体で平均した男女の生まれてくる確率とは別に、
夫婦毎に異なる確率を持つというモデルを採用した時点で、
「男の子を産むまで子供を作り続ける」という意思によって
男女構成比は崩れる、と。

788 名前:132人目の素数さん:2008/10/17(金) 07:00:45
>>786
58/41ですかね。
手計算でどうやって探せばいいかはよくわからん。

789 名前:132人目の素数さん:2008/10/17(金) 07:04:32
よく分からないのになぜ出せた

790 名前:132人目の素数さん:2008/10/17(金) 07:12:49
Excel

791 名前:132人目の素数さん:2008/10/17(金) 08:33:52
>>786
これくらいなら一応手でできるね.電卓があればなお簡単.
Stern-Brocot tree を構成する.

[0/1, 1/0] → 1/1 ≦ √2
[1/1, 1/0] → 2/1 ≧ 17/12
[1/1, 2/1] → 3/2 ≧ 17/12
[1/1, 3/2] → 4/3 ≦ √2
[4/3, 3/2] → 7/5 ≦ √2
[7/5, 3/2] → 10/7 ≧ 17/12
[7/5, 10/7] → 17/12 ≧ 17/12
[7/5, 17/12] → 24/17 ≦ √2
[24/17, 17/12] → 41/29 ≦ √2
[41/29, 17/12] → √2 < 58/41 < 17/12

よって 58/41 が解

Stern-Brocot tree の説明はググれば出てくる.

792 名前:132人目の素数さん:2008/10/17(金) 19:01:55
>>785
男児が生まれる確率のが統計的に高いのは
性染色体の部分で重さが僅かに違うから、卵子まで到達するのにかかる時間がXY型の方がXX型より短くなるため

という説を生物の授業中に先生が言ってた。
真偽は知らないけどね

793 名前:132人目の素数さん:2008/10/17(金) 23:37:00
>>786
高校生的解法。
√2<q/p<17/12のとき288p^2<(12q)^2<(17p)^2=289p^2より
(17p)^2-p^2+1≦(12q)^2≦(17p-1)^2 これよりp≧34が必要と分かる。
p=34から順に探していくとp=41,q=58という解が見付かる。
あとはこれがp+qを最小にすることを示せばよいが、
42'≦p'<q'/√2を満たす任意のp',q'についてp'+q'>(√2+1)p'≧42(√2+1)=101.39・・・

794 名前:132人目の素数さん:2008/10/18(土) 08:09:18
2p^2+1,3p^2+1,6p^2+1が全て平方数となるような素数pが存在しない事を証明せよ。

ガイシュツならごめんよ

795 名前:132人目の素数さん:2008/10/18(土) 18:10:58
p^2,2p^2+1,3p^2+1,6p^2+1:平方数であるとする。(仮定より、背理法)
2p^2+1=a^2,3p^2+1=b^2,6p^2+1=c^2とすると
p^2(2p^2+1)(3p^2+1)(6p^2+1)
=(pabc)^2
∴p^2(2p^2+1)(3p^2+1)(6p^2+1):平方数・・・@
p^2=nと置くと
p^2(2p^2+1)(3p^2+1)(6p^2+1)
n(2n+1)(3n+1)(6n+1)
=(6n^2+5n+1)(6n^2+n)
=36n^4+36n^3+11n^2+n
=(6n^2+3n+1/6)^2-1/36
∴(6n^2+3n)^2-1/36<(6n^2+3n+1/6)^2-1/36<(6n^2+3n+1)^2-1/36
∴(6n^2+3n)^2<(6n^2+3n+1/6)^2<(6n^2+3n+1)^2
∴(6n^2+3n)^2<n(2n+1)(3n+1)(6n+1)<(6n^2+3n+1)^2
∴(6n^2+3n)^2<p^2(2p^2+1)(3p^2+1)(6p^2+1)<(6n^2+3n+1)^2
∴p^2(2p^2+1)(3p^2+1)(6p^2+1):平方数ではない・・・A
@、Aより矛盾。

796 名前:132人目の素数さん:2008/10/18(土) 19:25:47
>>794 そんな愚問を出す前に、kを奇数として、2k^2+1 が平方数になる事があるかチェックしたまえ

797 名前:132人目の素数さん:2008/10/18(土) 20:05:13
www

798 名前:132人目の素数さん:2008/10/18(土) 20:22:31
x^2=(3n^2+1)(6n^2+1)=18n^4+9n^2+1=9n^2(2n^2+1)+1=y^2+1

799 名前:132人目の素数さん:2008/10/25(土) 11:21:59
□に0〜9までの数字を1つずつ入れて縦と横の足し算を同時に成立させて下さい.同じ数字2度使いは不可
(全通り求めて下さい)
■は無視して下さい

■■□□│□
■■□□│□
■+■□│□
■───■■
■■□□■■

800 名前:132人目の素数さん:2008/10/25(土) 11:34:01
    □□│□
    □□│□
  +  □│□
  ───   
    □□   


801 名前:132人目の素数さん:2008/10/25(土) 12:01:00
>>799
一行目の二桁+二行目の三桁=三行目の三桁?


802 名前:132人目の素数さん:2008/10/25(土) 12:49:41
    AB│C
    DE│F
  +  G│H
  ───
    IJ

AB+DE+G=IJ
DA+GEB=HFC

803 名前:132人目の素数さん:2008/10/25(土) 16:56:05
力技で解かせたが、答えが 20個 (*1) もある上に、

14 | 5
39 | 2
+ 7 | 8
----+
60

17 | 8
39 | 2
+ 4 | 5
----+
60

のように、値を交換した (4 ⇔ 7, 5 ⇔ 8) だけの答えもそれなりにあるので、
俺的には面白い問題とは言えない。

# *1: G = 0 のケースを含んでいるから、それを除外すると 18個

804 名前:132人目の素数さん:2008/10/25(土) 20:02:32
>>803
答えを詳しく

805 名前:132人目の素数さん:2008/10/25(土) 21:57:57
自分でやれ。

806 名前:>>803:2008/10/25(土) 23:51:43
>>804
20個全部欲しいってこと?
つまんないし、ひと迷惑だからやめとくわ。

807 名前:132人目の素数さん:2008/10/26(日) 08:23:03
>>802のように総当りで解ける問題は面白くない
工夫するのが面白いと思ってるのだろうけど、正直結果も面白くない

意外な結果だとか、成り立ちそうなのに示しにくいとか、そういうのが面白い

808 名前:132人目の素数さん:2008/10/26(日) 08:57:10
じゃあ俺様が面白い問題をだしてやろう

□に入る数字はなにか
3+□=8

どうだ面白いだろwwww

809 名前:132人目の素数さん:2008/10/26(日) 09:26:37
>>807を受けて「じゃあ面白い問題を出してやろう」って言うんだから、その問題は

>意外な結果だとか、成り立ちそうなのに示しにくいとか、そういうのが面白い

という条件を満たしていなければならない。しかし>>808はこの条件を満たしていない。

810 名前:132人目の素数さん:2008/10/26(日) 09:55:56
本人面白いと思ってるんだから、そっとしておいてやるのが大人ってもんだよ。

811 名前:808:2008/10/26(日) 11:58:12
どうだ?www
俺様の問題面白かっただろ?wwww
やっぱ俺様って天才wwwwwwwwwwwwwww

812 名前:132人目の素数さん:2008/10/26(日) 13:31:49
>>807の提示したような「面白い問題の条件」を満たしていないという「意外性」が面白いんだろう
違うならもう知らん

>>807のような意見が出てきた時点でこの手のレスが来ることは想定内
エスパー検定でも9〜8級レベル



813 名前:132人目の素数さん:2008/10/26(日) 14:27:34
じゃぁ要望にこたえて、某有名、大学入試問題集から一問

次の命題の真偽を調べ、真ならば証明し、偽ならば反例を示せ。
「すべての非負整数 n について、0<a(n)<1 ならば、
lim[n→∞]a(1)a(2)a(3)****a(n)=0 」

814 名前:132人目の素数さん:2008/10/26(日) 14:35:27
出題者がAとBの二人に別々に自然数を伝えた後、こういった。
「2以上の相異なる2つの自然数に対し、Aにはその積を、Bにはその和を伝えた。」

A「私には元の二つが何か分かりません。」
B「そうでしょうね。あなたにも分からないと思ってましたよ。」
A「ほほう、ならば分かりました。」
B「そうですか、それならば私にも分かりました。」

AとBの会話から、元の2自然数を決定しなさい。
ただし、それらはともに20以下であると仮定してよい(A, Bの知る限りではない)。

815 名前:132人目の素数さん:2008/10/26(日) 15:22:34
>>813
a_n=1/2^(1/2^n))
lim[n→∞]a(1)a(2)a(3)****a(n)=1/2

816 名前:132人目の素数さん:2008/10/26(日) 15:33:57
>>815
全然驚かれませんでしたね、ガッカリです

817 名前:132人目の素数さん:2008/10/26(日) 16:25:45
こんなもんにどう驚けと。

818 名前:132人目の素数さん:2008/10/26(日) 16:32:46
ボクチンのいおなずん!

819 名前:132人目の素数さん:2008/10/26(日) 18:29:31
>>814
3と14

820 名前:132人目の素数さん:2008/10/26(日) 18:30:09
間違えた4と13

821 名前:132人目の素数さん:2008/10/26(日) 18:47:52
>>814
コンピュータ使っちゃダメ?使って良ければこんな感じで出るんだけど。
> B「そうでしょうね。あなたにも分からないと思ってましたよ。」
よりBが知った和は相異なる2つの素数の和では表せないことが分かる。
そのような和は6=2+4を除けば11,17,23,25,27,29,35,37の8通りである。
> A「ほほう、ならば分かりました。」
より和が集合{11,17,23,25,27,29,35,37}に含まれることから2数が一意に決まる。
Aが知った数は18=2*9(=3*6),24=8*3(=4*6=2*12)など多数考えられるが、
30=2*15=5*6(=3*10)や66=2*33=3*22=6*11などは除かれる。
> B「そうですか、それならば私にも分かりました。」
よりBが知った数として11=2+9=3+8=4+7や23=4+19=5+18=7+16=10+13などが排除され、
最終的に17=4+13だけが残る。ゆえに求める自然数は4と13。

822 名前:132人目の素数さん:2008/10/29(水) 20:56:40
y=□x^2,y=□x+□の交点は(□,□),(□,□)であり、2線で囲まれた部分の面積S=□である。
□に数字を埋めよ。ただし入る数字は全て1桁の自然数である。

823 名前:132人目の素数さん:2008/10/29(水) 21:31:35
>>822
はぁ?どこが面白いんだ?
センターレベルじゃないか

824 名前:132人目の素数さん:2008/10/29(水) 22:23:59
>>822
y = a x^2, y= bx + c
交点での x の値は ax^2 + bx + c = 0
x = (-b±√(b^2 - 4ac)) / (2a)

0 < a, 0 < b, 0 < c, だから、x = (-b - √(b^2 - 4ac)) / (2a) は、
負もしくは虚数にしかならない。

つまり、交点 (□,□),(□,□) の□が全て自然数はありえない。

センター試験にこんな問題はでないと思うが、とても面白いと思えない
点では >>823 に同意。

825 名前:132人目の素数さん:2008/10/29(水) 22:27:18
822で
入る数字が全て2桁の自然数の時、解は幾つ存在するか。

826 名前:822:2008/10/29(水) 22:34:07
あぁ、何か間違えてた。
誤:y=□x^2,y=□x+□の交点は(□,□),(□,□)であり、2線で囲まれた部分の面積S=□である。
正:y=□x^2,y=□x+□の交点は(-□,□),(□,□)であり、2線で囲まれた部分の面積S=□である。

827 名前:132人目の素数さん:2008/10/29(水) 22:35:02
>>825
バカか?>>824で終了。桁数の問題では無い。消えろクズ。

828 名前:132人目の素数さん:2008/10/29(水) 23:54:00
>>827
aho

829 名前:132人目の素数さん:2008/10/30(木) 00:05:17
>>827
あほがいる

830 名前:132人目の素数さん:2008/10/30(木) 00:25:19
>>828-829
はいはい自演乙

831 名前:132人目の素数さん:2008/10/30(木) 11:24:43
11111

832 名前:132人目の素数さん:2008/10/30(木) 16:38:14
100人の死刑囚がいます
100人はこれから赤か青か白の帽子を被せられます
さらに100人は階段の下に向かって一列に並ばされます
自分より下にいる人の帽子は見えますが,自分や自分より上にいる人の帽子は見れません
100人は上から順に,自分の帽子の色を言わされます
もし正しい色を言えなかった場合はその場で処刑されます
100人は帽子を被せられる前に相談をし,
なるべくたくさん確実に助かる方法を考えだしました
それはどんな方法でしょうか?
そして,何人助けられるのでしょうか?
(ただし100人は「赤」,「青」,「白」の一言しか言えず
イントネーションを変える,前の人に触るとかいうアクションも一切できません)

833 名前:132人目の素数さん:2008/10/30(木) 16:45:01
確実となると50人助けることしか思いつかんなあ。

834 名前:132人目の素数さん:2008/10/30(木) 16:46:29
>>832
有名問題:囚人と帽子

835 名前:132人目の素数さん:2008/10/30(木) 16:48:23
赤, 青, 白を 0, 1, 2 に対応させ、
最初の人が全ての人の合計を mod 3 で計算して伝えておけば
残りの人は前の人との差分で自分の色が分かる。

836 名前:132人目の素数さん:2008/10/30(木) 17:10:50
>>835
差分をどうやって知るの?

837 名前:132人目の素数さん:2008/10/30(木) 17:38:31
>>836
たとえば上から順番に 1 1 2 0 2 と並んでたら
一人目が総和を計算して 1+2+0+2 = 2 と言う
以降はこの数から自分以前の人が言った数の合計と
自分の前方の数の合計を引いたものを言えばいい

二人目:2 - (2+0+2) = 1 と言う
三人目:2 - (1) - (0+2) = 2 と言う
四人目:2 - (1+2) - (2) = 0 と言う
五人目:2 - (1+2+0) = 2 と言う

838 名前:132人目の素数さん:2008/10/31(金) 03:06:14
y = a x^2, y= bx + c
交点での x の値は ax^2 + bx + c = 0


839 名前:132人目の素数さん:2008/10/31(金) 05:49:56
つまり、それは最初の1人以外の99人は確実に助けられ、
1/3の確率で最初の1人も助けられる方法というわけだな。

ってゆーか、1番目の貧乏くじを引いた時点で、そいつに事前の申し合わせを守る義理なんて
なくなるので、後で1人目が約束通りの行動を取らなかったことが判明したら
生き残った全員でそいつをぬっころすという申し合わせも必要だな。

...で、2人目が計算ミスで処刑され、全員涙目w

840 名前:132人目の素数さん:2008/11/01(土) 00:53:01
ん?これ囚人同士の話し合いした時としない時で結果変わるの?

841 名前:132人目の素数さん:2008/11/01(土) 02:36:34
しないで、色と数字の対応付けをどうやってするんだ?
囚人全員が頭がいいという設定なら、死ぬのは6人くらいですむかもしれんが。

842 名前:132人目の素数さん:2008/11/01(土) 02:48:11
囚人全員がどんなに頭がよくても、
事前の打ち合わせなしでは全員死ぬ確率は2/3。
「自分より前の囚人の選択は、自分の帽子の色とは独立の事象である」ことを
帰納的に考えれば明らか。
逆に「自分より前の囚人はそこそこ頭が悪い」という場合の方が、
なんらかの傾向を仮定して(例えば見えている数が少ない色を選びたがるとか)
それを前提に推論することで、若干死ぬ確率を減らせるかもしれん。
まあ、全員が「自分以外は頭が悪い」と仮定するのはナンセンスだがw

843 名前:132人目の素数さん:2008/11/01(土) 02:53:51
>>842
> 「自分より前の囚人の選択は、自分の帽子の色とは独立の事象である」ことを 
> 帰納的に考えれば明らか。 

それは頭が悪い。

844 名前:132人目の素数さん:2008/11/01(土) 03:08:54
これ東大理物の人に問題として出されたことがあるんだけど、
打ち合わせをして良いという条件を一切与えられなかったから
当然話し合いは禁止だと思って、全然分からなかった覚えがある。

845 名前:132人目の素数さん:2008/11/01(土) 03:17:28
次へ情報を伝えつつ助かるには
正しい自分の色を言い、かつその色が次の人への情報になっていることが必要。
で、パリティを皆が思いつくかどうか。(他の方法がないかどうか)

思いついたとして、色と数字の対応がどうすれば推論できるのか。




846 名前:132人目の素数さん:2008/11/01(土) 03:53:04
>>843
事前に打ち合わせがない場合の話だぞ?

847 名前:132人目の素数さん:2008/11/01(土) 04:09:32
事前に打ち合わせしなきゃ

1人目:見えてる情報は、何も推論の足しにならないから、適当に答えるしかない
2人目:1人目が適当に答えたのだから、それで処刑されてようがされてなかろうが
    1人目の答えは何も推論の足しにならず、見えてる情報も同様なので、
    適当に答えるしかない
3人目以降:同様

となるわな。


848 名前:132人目の素数さん:2008/11/01(土) 04:24:02
1人目がパリティになれば2人目以降は救済される方法はわかったけど、
誰か1人がミスした場合のリカバー方法も、事前に詳細に打ち合わせておけば、
ミスが1人だけという前提なら、ミスして処刑された直後の人は処刑確率1/2で、
それ以降はまた救済されるという方法を準備しておけそうだな。

一番困るのは、1人目や2人目がミスした場合だが。
(どっちがミスしたのかわからないから困る。)

849 名前:132人目の素数さん:2008/11/01(土) 05:04:58
>>847
> 1人目:見えてる情報は、何も推論の足しにならないから、適当に答えるしかない 

ここが頭が悪い。

850 名前:132人目の素数さん:2008/11/01(土) 05:08:34
一人目がもし頭がいいなら

「 何色を答えても自分が助かる確率は同じ
 それならばランダムに答えるのではなく
 何か残る人間に情報を残す答えを選べないだろうか?」


851 名前:132人目の素数さん:2008/11/01(土) 06:55:11
相談もしてないのに、残した情報を相手がどう解釈するのかがわかるのか。
さすがエスパーが常駐する数学板だな

852 名前:132人目の素数さん:2008/11/01(土) 08:51:50
> 相談もしてないのに、残した情報を相手がどう解釈するのかがわかるのか。 

わかる可能性がないことを証明しなければ、わからないとはいえないのが数学だろ?
その方法はまったくないのか?

853 名前:132人目の素数さん:2008/11/01(土) 09:07:20
>>852
まったくないだろ。
現実的に意味のある論理体系において、
定義されていない記号1つのみから推論をするのはあきらかに不可能。

854 名前:132人目の素数さん:2008/11/01(土) 09:32:06
全員が十分に頭がいいとすると
たとえば頭のいい誰かはこんなふうに考えるかもしれない。

・囚人と帽子の問題くらい誰でも知っている。

・赤青白と012をどう対応させたのか?6通りの可能性がある。

・運がいいことに私の前にいる三色どの色も3の倍数人だとしたら
 数字と色の対応がどうなっていようと、私の後ろの彼は私の帽子の色を言うはずだ。
 つまり、前にいる三色どの色も3の倍数人だった場合は、必ず助かるのではないか?

・ということは私以前に処刑されなかった人たちは、運良く1/3の確率で自分の帽子の色があたったのではなく
 前にいる三色どの色も3の倍数人だった可能性があるのではないだろうか?

・私の直前の人が処刑されなかった。 かつ、私の前にいる人たちは2色が3の倍数人、残りの色は3の倍数−1人。
 ということは私は、残りの色である可能性が高くないか?

等など…



855 名前:132人目の素数さん:2008/11/01(土) 09:34:56
>>853
> 定義されていない記号1つのみから推論をするのはあきらかに不可能。 

このゲーム(?)は、少なくともルールについては
事前の打ち合わせなく全員共通の知識だろ?

つまり定義されているものは0ではないことにならんか?

856 名前:132人目の素数さん:2008/11/01(土) 10:00:46
こうして戦略集合とか頭が良いの意味とかが未定義のまま
議論が発散するんだよな。毎度おなじみのパターンだ。

857 名前:132人目の素数さん:2008/11/01(土) 10:35:22
定義しないと話ができないなら定義してくれてかまわんよ。


858 名前:132人目の素数さん:2008/11/01(土) 11:08:56
定義のしかたで発散するのが見えてるし、この話が面白いとも思えん。

859 名前:132人目の素数さん:2008/11/01(土) 11:41:03
普通の論理パズルは他の人の知識や推論速度が
大雑把に言って自分と同じ、というくらいの仮定で事が済むんだけど、
この問題に関しては
>・赤青白と012をどう対応させたのか?
根本的にまずこれがあるからなあ。

「頭が良」かったら赤を 0 、青を 1 に対応させるはず、とか
そんなことはさすがに言えないんじゃないか?
数学板はやたら自分の主観が絶対だと思ってる奴が居るから
若しかしたら 0 という数字の色的なイメージは同考えても赤、
そう思わない奴は莫迦、とか言い出すのかもしれないけど。

860 名前:132人目の素数さん:2008/11/01(土) 13:49:20
>>859
> 「頭が良」かったら赤を 0 、青を 1 に対応させるはず、とか 
> そんなことはさすがに言えないんじゃないか? 


まさか、その方向でやりたいのだとは思わなかった。

2番目、3番目以降は1番目がどう対応させたのかを
いかに効率よく当てて行く手順を考えるのはダメなのか?

当たっているかどうかは、1/3よりも少し多く生き残ることで
わかったりしないのか?

なにしろ可能性は6通りしかなく、完全に当たると
一人もしな名なくなるんだけど…

それともそんな方法はない、と言い切れる類のものなのか?

861 名前:132人目の素数さん:2008/11/01(土) 13:50:18
× 一人もしな名なくなるんだけど… 
○ 一人も死ななくなるんだけど… 


862 名前:132人目の素数さん:2008/11/01(土) 14:04:40
色と数字の組み合わせによるパリティ以外には生存率を上げる方法はないのか?
という疑問はあるが、とりあえずはそこは除外して考えてみたい。

* 1番目の奴は、全員の帽子の色を見てから対応を決定できる。
 つまり、2番目が何を見ているか、3番目が何を見ているか…
 全員が何を見ているのかを知ってから対応を決めることができる。

* 帽子の色の数や順番に偏りがあると、2番目以降が当てやすい
 組み合わせができたりしないか?

* 組み合わせがわからないままでも、条件(前にある色)により
 必ず生き残れるとか、生き残る確率が高い奴が出るなんてのも
 あるのかもしれない。 >>854の3番目とか








863 名前:132人目の素数さん:2008/11/01(土) 14:08:12
興味深いエレガントな答えが
あらかじめ用意されているという保証のない問題は
面白くないと思っているのかもしれないぞ。

864 名前:132人目の素数さん:2008/11/01(土) 14:28:52
ホントにそうなのだとしたら、もうこのスレには用はないかもしれん。


865 名前:132人目の素数さん:2008/11/01(土) 14:50:41
反例

・自分だけ青
・自分以外はみんな赤

で、仮に自分より後の奴らは頭が良く、かつ運もあり
最初から連続して「赤」「赤」「赤」…とみんな正解しているとする。

さてここで自分の番になったとして、「青」という正解にたどり着く手がかりは一切ないと思うのだが

866 名前:132人目の素数さん:2008/11/01(土) 14:54:26
↑これは「事前の打ち合わせが一切ない場合」ね。

867 名前:132人目の素数さん:2008/11/01(土) 15:12:16
>>865
もし本当にその状況なら自分の色がわからないかもしれないが
しかし、その状況は1/3よりはるかにおおくのひとが生き残っているので
題意は十分に満たしていると思う。つまり反例になっていない。

題意は、「自分が生き残る方法」ではなく「できるだけ沢山の人間が生き残る」であるはずだ。

868 名前:132人目の素数さん:2008/11/01(土) 15:17:48
反例をあげるなら

・ 帽子の色がどのように配分されていても、誰一人として、自分が生き残る確率を
 1/3よりほんの少しでも上げることはできない

ことを示さねばならない。

事前に相談する条件では99人以上が生き残るという劇的な方法があるせいで
事前に相談できない条件でも、いつのまにかそういうドラマチックなことを
勝手に条件にしてしまっていないか?


869 名前:132人目の素数さん:2008/11/01(土) 15:29:20
>>867 >>868
別に 865 がどうのこうのってわけじゃないんだけど、

各色の帽子が等確率で振られるなんてことはどこにも書かれてないんだけど、
なんで 1/3 を基準に使ってるの?

870 名前:132人目の素数さん:2008/11/01(土) 15:36:07
>>869
逆に聞きたいんだが、1/3の確率で生き残るのに、帽子の色が等確率でふられている必要があるの?

各人自分の帽子の色を推論する手段が全くないときには、ランダムに赤青白のどれかを言うしかない。
そしてそれが当たっている確率は1/3、つまり生き残る確率が1/3より高くできるということは
何らかの推論手段があるということだ。

この考え方は何かおかしいかな?

871 名前:132人目の素数さん:2008/11/01(土) 15:40:17
>>869
帽子の色が等確率にふられないことが事前にわかっているなら
より高い確率で生き残れそうだがな。

872 名前:132人目の素数さん:2008/11/01(土) 15:45:38
> 各人自分の帽子の色を推論する手段が全くないときには、ランダムに赤青白のどれかを言うしかない。 

各人自分の帽子の色を推論する手段と、後の人のために情報を残す手段の
どちらもが全くないときには、ランダムに赤青白のどれかを言うしかない。

に訂正。

873 名前:132人目の素数さん:2008/11/01(土) 19:59:18
いつのまにか誰が処刑されたか分かっているという仮定が付いてるね

874 名前:132人目の素数さん:2008/11/01(土) 20:11:19
>>872
> 各人自分の帽子の色を推論する手段と、後の人のために情報を残す手段の
> どちらもが全くないときには、ランダムに赤青白のどれかを言うしかない。

これは何故?
各人の帽子の色が独立同分布に従って決定されていて、
その分布が偏っていたら、一番多い色を言うのがベストの戦略だよね。
逆に、ランダムに言う戦略が合理的であるためには、
全ての色が等確率で分布していないといけない。

「情報がなかったら各人は一様分布と仮定して議論するだろう」ということ?

875 名前:132人目の素数さん:2008/11/01(土) 20:26:03
効率よく当てていくって無茶な相談だろ、、
赤とか青とかそんなビット数の少ない情報だけで
1番目の人の思考を当てるとか無理に決まってるだろ。
しかも1番目の人と次の2番目の人の考えることが
一緒である保証は全く無いし。
解答があるにしても、その推論は何ら論理的なものではなくなる。

どちらにしろ>>832の本来の出題意図とは外れているから、
この話を続けるのはエレガントな答えを誰かが見つけてからで良いんじゃないかな。

876 名前:132人目の素数さん:2008/11/01(土) 20:38:24
>>875
>どちらにしろ>>832の本来の出題意図とは外れているから、
>この話を続けるのはエレガントな答えを誰かが見つけてからで良いんじゃないかな。

「おまえは何を見てきた」by king

その話は>>837で終わっているようだが。

まあ、「相談なしで」という不毛な議論を延々としてるのも見飽きたので
そろそろだれか別の問題の投下よろ

877 名前:132人目の素数さん:2008/11/01(土) 22:51:19
不毛な議論認定キターーーーー

878 名前:132人目の素数さん:2008/11/01(土) 22:57:09
ああ、875は「相談無しで」の話ね。

879 名前:132人目の素数さん:2008/11/02(日) 00:19:27
同じ問題にいつまでもしつこく寄り集まってる様は面白くも楽しくもない

880 名前:132人目の素数さん:2008/11/02(日) 00:35:01
>>879
君にはわからんだろうが
フェルマーの最終定理を面白いと感じるプロもたくさんいるのだよ。

881 名前:132人目の素数さん:2008/11/02(日) 01:21:23
フェルマーの最終定理は面白いだろ。
それはそうと問題投下。

サイコロを振って出た目が得点になるゲームがあります。
・サイコロは最高n回まで振れる。
・もう1回振るかどうかは振った後決めれる。
・最後に出た目が得点になる。
(i)サイコロを2回まで振っていいとき(n=2)、どのような作戦にすれば得点の期待値が最高になりますか?
(ii)サイコロを3回まで振っていいとき(n=3)、どのような作戦にすれば得点の期待値が最高になりますか?

882 名前:132人目の素数さん:2008/11/02(日) 01:23:53
ぱっと見は4以上が出たらやめる。

883 名前:132人目の素数さん:2008/11/02(日) 01:26:41
実は(ii)は1回目で4以下が出たらもう1回振ったほうが得。

884 名前:132人目の素数さん:2008/11/02(日) 01:27:01
(i)3以下だったら2回目を振る。
(ii)4以下だったら2回目を振る。2回目3以下だったら3回目を振る。

885 名前:132人目の素数さん:2008/11/02(日) 01:28:12
意外と大きくなるんだな期待値。

886 名前:132人目の素数さん:2008/11/02(日) 01:31:48
>>881 (1) サイコロを1回振ったときの目の期待値は3.5だから4以上出た時にやめればよい。
(2)二回まで振っていいときの期待値を求める。
(1)より出方は4,5,6のときはその値、1.2.3のときは
もう一度振り直すのだからその期待値3.5をとって考えればよい。
よって期待値は(4+5+6+10.5)/6=4.25
よって一回目に降ったときに5以上ならやめ、それ以下の目の場合は振り直し、
残りの2回は(1)のやりかたで降ればよい。

どこが面白い問題なのか。

887 名前:132人目の素数さん:2008/11/02(日) 01:38:03
5が出ても振った方が得になることはないんだな。

888 名前:132人目の素数さん:2008/11/02(日) 01:53:39
財布の中には、1000円入っています。
100円玉、50円玉、10円玉、5円玉、1円玉がそれぞれ何枚かづつ(1枚以上)入っていて、
それぞれの硬貨の枚数は1枚、5枚、6枚、15枚、25枚のどれかだと言う事が分かっています。
どの硬貨が何枚入っているでしょうか?
ただし硬貨の値段と枚数とは1対1に対応しているものとします。

889 名前:132人目の素数さん:2008/11/02(日) 02:11:47
>>887
nを大きくすれば、どっかでなるだろ

890 名前:132人目の素数さん:2008/11/02(日) 02:18:00
n=5で期待値が約5.13になるから、5回以上追加で振れるときは6以外が出たら振りなおしたほうがいいね

891 名前:132人目の素数さん:2008/11/02(日) 02:19:37
ならねえだろ

892 名前:132人目の素数さん:2008/11/02(日) 02:22:08
>>889
ならない。

893 名前:132人目の素数さん:2008/11/02(日) 02:26:40
1回目期待値=7/2=3.5
2回目期待値=(3*7/2+4+5+6)/6=17/4=4.25
3回目期待値=(4*17/4+5+6)/6=14/3≒4.67
4回目期待値=(4*14/3+5+6)/6=89/18≒4.94
5回目期待値=(4*89/18+5+6)/6=277/54≒5.13

894 名前:132人目の素数さん:2008/11/02(日) 06:24:50
>>874
> 各人の帽子の色が独立同分布に従って決定されていて、 
> その分布が偏っていたら、一番多い色を言うのがベストの戦略だよね。 

「分布が偏っている」というのがわかっているということは、推論する手段があるということ。

> 逆に、ランダムに言う戦略が合理的であるためには、 
> 全ての色が等確率で分布していないといけない。 

そんなことはない。 「偏っている」という情報だけがあっても
どの色に偏ってるのかの情報がなければ、結局どの色が多いのかはわからない。

どうやら、「実際にそうなっていること」と「そうなっているという情報が与えられていること」の
区別が付いていないようだな。

895 名前:132人目の素数さん:2008/11/02(日) 06:25:44
>>875

>>863 >>864

896 名前:132人目の素数さん:2008/11/02(日) 06:29:14
前スレあたりからみるとずいぶんレベル落ちたな。

問題にも解答にも意外性も何もない。

897 名前:132人目の素数さん:2008/11/02(日) 07:14:31
>>894
> > 逆に、ランダムに言う戦略が合理的であるためには、
> > 全ての色が等確率で分布していないといけない。
> そんなことはない。 「偏っている」という情報だけがあっても
> どの色に偏ってるのかの情報がなければ、結局どの色が多いのかはわからない。

「偏っている」という情報だけがある場合は、
「赤のみを選ぶ戦略」か「青のみを選ぶ戦略」か「白のみを選ぶ戦略」の
どれかが最良になるはずだよね(当然、どれが良いかは分からない)。

決して「等確率で選ぶ戦略」が最良(合理的)にはならないと思うのだけれど。

898 名前:132人目の素数さん:2008/11/02(日) 07:59:11
>>897

ある一人を取り出したときに

* 「赤のみを選ぶ戦略」か「青のみを選ぶ戦略」か「白のみを選ぶ戦略」の
 どれかをランダムに選ぶ。

* 「赤」「青」「白」のどれかをランダムに選ぶ。

これになにか違いがあるのか?

899 名前:132人目の素数さん:2008/11/02(日) 08:27:25
> 決して「等確率で選ぶ戦略」が最良(合理的)にはならない


「他のどの戦略と比べても「等確率で選ぶ戦略」が良でないほうではない」
と厳密に言って欲しいってことじゃないのか?

AとBの大きいほう という言い方を 、等しいときには大きいほうはないという人がいる。
そのため「AとBの小さくないほう」という言い方がある。

900 名前:132人目の素数さん:2008/11/02(日) 08:35:39
助かる確率は同じだが偏差(分布)は異なるといいたいのかと思ったが
最良ではないなんて言ってるところからするとそういうわけでもないらしいな。

901 名前:132人目の素数さん:2008/11/02(日) 08:37:36
>>897
> 決して「等確率で選ぶ戦略」が最良(合理的)にはならないと思うのだけれど。

それが最良でないということは、他に最良な方法が、少なくとも1/3以上の期待値で
助かるという方法があるということ?

それとも、期待値は同じでも、どちらがよい戦略なのかを決める基準が別にあるという話か?

902 名前:132人目の素数さん:2008/11/02(日) 12:06:29
>>901
・より良い戦略が存在する
・その戦略をとる基準が存在する

この二つは別ですよね。

903 名前:132人目の素数さん:2008/11/02(日) 12:18:30
>>902
その戦略を取る基準が存在しない戦略は、ないのと同じだろ。

「各自が自分の帽子の色と同じ色の名を叫ぶ」
という戦略は生存率100%だが
これを最良の戦略だと認めるつもりか?



904 名前:903:2008/11/02(日) 12:24:36
選ぶ基準が存在しない戦略は「良い戦略」ではないだろう。
良い戦略とは、少なくとも、成果を上がられるものでないとならん。

「良い戦略」の定義でもめるかもとは思ったが、まさかこんなもめ方をするとは思わなかった。




905 名前:132人目の素数さん:2008/11/02(日) 12:31:28
>>897
「偏っているが色はわからない」という仮定の下で
その 「赤のみを選ぶ戦略」と「青のみを選ぶ戦略」と「白のみを選ぶ戦略」の
3つの戦略について、生存者の人数の期待値を、それぞれ計算してごらん。 


906 名前:132人目の素数さん:2008/11/02(日) 18:26:59
>>903
認めます。

>>905
どのように計算するのでしょう?

例えば、赤のみを選ぶ戦略の期待生存率(生存人数の期待値/全人数)は
赤の出る率に等しいのですが、「偏っている」という情報だけでは
この値は計算できないように見えます。

907 名前:132人目の素数さん:2008/11/02(日) 18:38:18
今更だが>>53,54がわかりません
(2)ですが
Step4の「両方同じなら両方を反転して終了」で終了しない場合がありませんか?
たとえば
Step1,2,3を踏んで
  お--------お
  |       |
  |      |
  う---------?
ここで左上と右下を選び「?」が表だったとして両方を反転しても終了しませんよね?
私がどこで間違えているか教えて下さい


908 名前:132人目の素数さん:2008/11/02(日) 20:25:35
>>907
step2が終わった時点でチャイムが鳴らなかったら、3個が表、1個が裏の状態になってるはず
さらに、step3が終了してもチャイムがならなかったら、2個が表、2個が裏の状態になってるはず
?が表であることはない

909 名前:132人目の素数さん:2008/11/02(日) 20:52:09
>>908
なるほどありがとうございます

910 名前:132人目の素数さん:2008/11/03(月) 13:59:59
906

> 認めます。 

では、そのルールなら、この問題の最良の戦略も
もとの問題(事前相談アリ)の最良の戦略も
「各自が自分の帽子の色と同じ色の名を叫ぶ」 だな。
生存率は100%。


> どのように計算するのでしょう? 

> 赤の出る率に等しいのですが、「偏っている」という情報だけでは 
> この値は計算できないように見えます。 

どう偏っているのかを確率的に扱えばよいのではないかな?

911 名前:132人目の素数さん:2008/11/16(日) 20:25:49

次のような2人用ゲームを考える。

ルール:
ゲーム開始時において、集合Sを{0}(0のみからなる集合)とする。
各プレイヤーは自分の手番において、Sに含まれない自然数nを言う。
このとき集合{s+mn|s∈S,m∈N}を考え、これを改めてSとする。(Nは自然数{0,1,2,...})
これを先手と後手で交互におこなう。
1を言うと負け。

先手、後手のいずれかに必勝法はあるか?
あるとすればどのような方法か?

912 名前:132人目の素数さん:2008/11/17(月) 04:43:52
>>911
> 先手、後手のいずれかに必勝法はあるか? 
ない。
∵Sに含まれない最小の素数が常に存在する。

913 名前:132人目の素数さん:2008/11/17(月) 12:56:04
先手と後手が一回ずつ数を言った時点で
Sに含まれない自然数は有限個になるわけだけど。

その二数がn_1及びn_2ならばSに含まれない最大数は
n_1・n_2-n_1-n_2になるんだっけ。
ちょっと正整数だったか非負整数だったか覚えてないんで
微妙だけどだいたいこんな感じの式。

914 名前:132人目の素数さん:2008/11/17(月) 13:06:20
え?なんか俺ルールを勘違いしているかな?

915 名前:132人目の素数さん:2008/11/17(月) 13:48:40
全然まじめに考えていないが、1回ずつ数を言った時点で有限個に絞られるなら後手必勝なんじゃないか?と予想。

916 名前:132人目の素数さん:2008/11/17(月) 21:05:00
S={0}.

s < 2.
S={0,2,4,6,8,10,...}.

g < 3.
S={0,2,3,4,5=2+3x1,6=6+3x0,7=4+3x1,...}.

s < 1.
g win.


917 名前:132人目の素数さん:2008/11/17(月) 23:19:58
>>913
> 先手と後手が一回ずつ数を言った時点で
> Sに含まれない自然数は有限個になるわけだけど。
先手が 4、後手が 2 と言ったら?

918 名前:132人目の素数さん:2008/11/17(月) 23:25:02
Nは自然数ではなく非負の整数な気が

919 名前:132人目の素数さん:2008/11/18(火) 01:31:23
言葉の定義の話なら、0を含める流儀もあるとしか

920 名前:132人目の素数さん:2008/11/18(火) 20:55:04
ぶるばき

921 名前:132人目の素数さん:2008/11/18(火) 21:53:23
二百日。


922 名前:132人目の素数さん:2008/11/20(木) 03:10:38
a,bが互いに素な整数のとき、任意の自然数nに対して
ax + by = n
は整数解を持つ。
とくに0≦x≦b-1を満たすように取る事ができる。
このときy = (n-ax)/b ≧ (n-a(b-1))/b
なので、n ≧ a(b-1)ならば、とくに(x,y)が非負の解を持つ。

このことは、今までに挙げられた中に互いに素な数が一組でもあるとき、Sの補集合が有限集合になることを意味している。

また、このゲームにおいて、
(@)2が挙げられたとき、Sは3を含まない。
(A)3が挙げられたとき、Sは2を含まない。
(B)2と3がともに挙げられた時点でSに含まれないのは1のみ
また、4以上の数が挙げられても2も3もSに含まれる事は無いので、両者は完全に見合いになっており、要するにこのゲームは
「相手に2か3を言わせたほうが勝ち」である。

例えば、{初手,二手目}={4,5}のときSの補集合={1,2,3,6,7,11}であり、
ここで先手が11を言えば{1,2,3,6,7}になる。
この6と7も見合いになっているのでこれは先手の勝ち。

一般に、もし先手があげた数が素数pだったら後手はpと素な数を言わざるを得ないので
そこでSに含まれない数の全体(有限)が必ず先手勝ちになるpがあるのか、
それともどんなpに対しても後手勝ちにできる二手目があるのかとか色々考えてみたけどよーわからん。

923 名前:132人目の素数さん:2008/11/20(木) 07:23:50
>>911 計算したところまで

1手目 4 なら、2手目 6 で後手勝ち、
1手目 6 なら、2手目 4 で後手勝ち。

>>922 p=5 に限定しても分からないんだが
1手目 5 の直後、次のペアが見合いになってる。
(ペアの片方を2手目に言われたら、もう一方を3手目に言えば先手勝ちという意味)

(4,11), (6,19), (7,8), (9,31), (12,33), (13,37), …

例えば、2手目 6 なら、3手目 19 で先手勝ち、
2手目 19 なら、3手目 6 で先手勝ち。

924 名前:132人目の素数さん:2008/12/03(水) 17:07:52
636

925 名前:132人目の素数さん:2008/12/03(水) 18:57:30
378

926 名前:132人目の素数さん:2008/12/03(水) 21:39:25
>>814
まったくわからない解説をしてくれ

927 名前:132人目の素数さん:2008/12/04(木) 01:33:39
>>926
答を解説してしまっては考える楽しみがなくなってしまうと思うので
途中までというか、基本的な考え方を…

まず、ABは 十分論理的だと考える。
すると以下のような推論が成り立つ。

Aには2数の積を伝えた。
しかし Aは 「私には元の二つが何か分かりません。」 と言った。

ということは、Aに伝えられた数は、2素数の積ではなかったはずだ。
(もし伝えられた数が、ふたつの素数の積ならば、Aには2数がすぐにわかるから)
同じような理由で、伝えられた積は素数の3乗でもない。

それに対し、2数の和が伝えられたBは
「そうでしょうね。あなたにも分からないと思ってましたよ。」と言った。

まず、あなたに『も』と言ったのだから、Bにもわからないと言う事だ。
ということは、Bに伝えられた和は5や6ではない。 
(もし5や6ならば、Bにはその2数がすぐにわかるから)
さらに、その和は二つの素数の和ではない。
(でなければ「あなたにも分からない」とは限らない)

などなど…こんなふうに推論を重ねていけばいい。


928 名前:132人目の素数さん:2008/12/04(木) 05:03:19
>>814は素数じゃなくて自然数じゃないの?

929 名前:132人目の素数さん:2008/12/04(木) 05:11:40
だから 0 と 2 だな

930 名前:132人目の素数さん:2008/12/04(木) 06:21:29
>>928
2以上のな

931 名前:132人目の素数さん:2008/12/04(木) 07:22:00
>>928
うーむ。
そういう返事がかえってくるとはさすがに予想していなかった。

>>814の2数は自然数。
だが、素数でないとは限らない。

二つの数が、両方共に素数でない事は、Aが最初に「わからない」といったから
初めて確定するんだが、それがわからないだろうか?

ある2以上の自然数ふたつを掛け合わせたものだといわれて
なにか数を言われたとき、それがふたつの素数の積だったら
元の二つの数はすぐわからるだろ?
たとえば77を言われたら、元の2数は7と11以外にありえない。
2以上の自然数のうち、それ以外のどんなふたつの数を掛けても
77にはならないのだから。

だから、もしAに伝えられた数が、ふたつの素数の積だったら
Aは「わからない」とは言わずに「わかった」というはずなんだ。

Aが「わからな」いと言っている以上は、Aに伝えられた数は
ふたつの素数の積ではなかった、ということ。


932 名前:928:2008/12/04(木) 17:53:59
>>931
ごめん。全然読めてなかった。
あと俺は>>926じゃないのであしからず。

933 名前:132人目の素数さん:2008/12/05(金) 05:03:23
>>814
答えは3.4

934 名前::2008/12/05(金) 05:22:19
違かった。スルーしてくれ

935 名前:132人目の素数さん:2008/12/06(土) 15:34:23
(1)
半径1の円を9個並べて、正方形に収めたい。
正方形の1辺の長さは6よりも小さくする事は出来ない事を証明せよ。

(2)
半径1の円をn個並べて正方形に収める時、正方形の1辺の長さの上限を与える式を作れ。
ただしnは十分に大きいものとする。

936 名前:132人目の素数さん:2008/12/09(火) 16:20:46
>>814
なぜ3,4じゃいけないんだぁあ!
誰か3,4が否定される理由を詳しくおしえて〜な

937 名前:132人目の素数さん:2008/12/09(火) 16:42:25
>>936
おそらくの範疇で聞いてほしい。

Bは「あなたにも分からないと思ってましたよ」と言った。
もし、答えが3,4だとしたらBに伝えられた自然数は7になる。
7から推測される2数は、2,5と3,4の二通りだ。
それらからAに言い渡された自然数が10か12と推定できる。
もし10だとしたらAは答えを唯一2,5と判断できる。
それをBは知ることができるから、Aが分からないとは断言できない。
つまり「あなたにも分からないと思ってましたよ」という発言に反するのだ。

と、思うのだが……

938 名前:132人目の素数さん:2008/12/09(火) 17:59:02
>>937
そのとおり。
>>936
納得した?

939 名前:132人目の素数さん:2008/12/09(火) 18:52:43
Ah〜!
Oh!Yes!
I was wrong!
THANKS!
HAHAHAHAHA!

940 名前:132人目の素数さん:2008/12/09(火) 21:58:11
まって
やっぱさ
和を聞いた奴が7をきいていたとすると
2,5 3,4
の二通りの持論をもっていったことになる
あったとき初めて
二数の組み合わせは互いに素の関係
ということがわかるから
やっぱ3,4でも題意成立だとおもっちゃたりしちゃりしちゃうんだが

941 名前:132人目の素数さん:2008/12/09(火) 22:33:43
>>940
> 二数の組み合わせは互いに素の関係

そんなことはわかっていないが?

こちらで考える可能性「2,5」と「3,4」のうち
もし正解が「2,5」だったら、それは2つ共素数なので
相手はたちどころにわかってしまう。

だから相手の意見を聞く前から「あなたもわからない」とは思えないんだよ。
「もしかしたら、あいては即座に答えてしまうかなあ」と思うしかない。
さらに、相手が即座に答えられなければ、正解は「2,5」ではないのだから
「3,4」に決まってしまう。
だから、Aが最初にわからないといった後に
Bは「そうでしょうね…」とは言わず
「なるほどそれを聞いて、私にはわかりました。」と言う。
そしてAは「まいりました、わたしにはやっぱりわかりません」と答える。
Bがわかったときいても最初に「12」と聞かされたAは 「2,6」なのか「3,4」なのかはわからないからだ。


942 名前:132人目の素数さん:2008/12/09(火) 22:42:37
「3,4が正解だった場合は頭のいい二人が交わした会話はこうだったかもしれない。

A 「まいりました。
  わたしがわからないと言ったとたんあなたはわかったというんでしょうね。
   しかしそれでもわたしにはあなたの答を聞くまでわからない」

B 「そうなんです。
  しかしわたしも、あなたがわからないというまでは
  いきなりわかったといわれてしまうのではないかとひやひやしてたことも
  わかっているんでしょう?」

943 名前:132人目の素数さん:2008/12/09(火) 22:46:53
つまり
3,4
だったら
Aがわからないちゅうことですか?

944 名前:132人目の素数さん:2008/12/09(火) 22:52:26
積は12
A「私には元の二つが何か分かりません。」
和は7
B「そうでしょうね。あなたにも分からないと思ってましたよ。」←この時点でBは答えが分かる
A「ほほう、ならば分かりました。」
B「そうですか、それならば私にも分かりました。」 ←故にこれはおかしい

945 名前:132人目の素数さん:2008/12/10(水) 10:57:36
>>944
積は12 
A「私には元の二つが何か分かりません。」 
和は7 
B「そうでしょうね。あなたにも分からないと思ってましたよ。」←この時点でBは答えが分かる 
A「ほほう、ならば分かりました。」 ←これもおかしい、ここではAはわからない
B「そうですか、それならば私にも分かりました。」 ←故にこれはおかしい 

946 名前:132人目の素数さん:2008/12/11(木) 01:47:25
元の二数をx,y(x≦y)とする
Aの初めの発言から、xとyのすくなくとも一つは合成数。

また、(z,w)をx+y=z+wを満たす二以上の自然数とする。ただしz≦w
二つ目のBの発言よりともに素数となる(z,w)の組は無い。
これを素数和表示不可能数と呼ぶ。
Bが聞いた数は素数和表示不可能数に違いない。
さて、全ての素数和表示不可能数はあきらかに「奇数の合成数+2」の形をしている。
逆に奇数合成数に2を足した数は素数和表示不可能。
(∵奇数合成数に2を足した数は奇数なので、二数の和に表すと少なくとも一方の和因子は偶数になる
 その偶数が4以上なら合成数だし、2なら仮定より奇数和因子が合成数)
すると40以下の範囲では素数和表示不可能数は以下の七つ。
11,17,23,27,29,35,37

三つ目のAの発言より、xy=ab(2≦a≦b)を満たすaとbの内、a+bが上記の素数和表示不可能な数になる組は一つそしてただ一つある。
例えばAが聞いた数が30だとすると、30=5*6=2*15より、11と17の両方を作れるので、Aはこの段階でもわからないというはず。
従ってAの聞いた数は30ではない。

947 名前:132人目の素数さん:2008/12/11(木) 01:48:31
さて、ここでx+y=11だったとしよう。
在りうる可能性は2+9,3+8,4+7,5+6のいずれかでり
Aは18,24,28,30のどれかを聞いたはずである。
24を二数の積abに表す方法は2*12,3*8,4*6の三通りで、a+bが素数和表示不可能な数になるのは(a,b)=(3,8)のみ
しかし28を二数の積abに表す方法は2*14,4*7の二通りで(4,7)が唯一の素数和表示不可能な和を与える。
つまりAが聞いた数が24でも28でもAは「分かった」というはずであり、最後のBのセリフはありえない。
従ってx+yは11ではない。

同様に考えていくと、
x+y=17のとき:
xy∈{30,42,52,60,66,70,72}
x+y=23:
xy∈{42,60,76,90,102,112,120,126,130,132}
x+y=27:
xy∈{50,72,92,110,126,140,152,162,170,176,180,182}
x+y=29:
xy∈{54,78,100,120,138,154,168,180,190,198,204,208,210}
x+y=35:
xy∈{66,96,124,150,174,196,216,234,250,264,276,286,294,300,304,306}
x+y=37:
xy∈{70,102,132,160,186,210,232,252,270,286,300,312,322,330,336,340,342}

Aが「分かった」と言う数は、上記の各集合の内のどこかに一回だけ出てくる数である。
そこで重複して現れる数を削っていくと上から順に一番上だけが単元集合になる
(x,yがともに20以下と言う条件はAとBは知らないので厳密にはx+y=123までリストを作るべきだが
 どうせこのリストの重複はそんなには起こらないので多分大丈夫)
その唯一残る数は52である。
即ちx+y=17のときにAが聞く可能性のある数たちのうち、Aが分かったと言う唯一の数が52であり、
x+yが他の値のときはそのような数は無い。

∴初めの二数は(4,13)

948 名前:132人目の素数さん:2008/12/13(土) 18:40:20
>>935

まず内接円が半径1の正六角形を平面状に敷き詰める。
そのあと一つの円に注目してA1とし、A1に接する円の一つをA2とする。
A1の中心を頂点にもち、線分A1A2と辺を共有する一片xの正方形を書く。
正方形を適当な方向に距離√2だけ平行移動させ、A1と正方形が二点で接するようにする。
最後に、正方形の内部に収まらない円を消す。

以上の手続きで一片xの正方形に対して入る円をf(x)個とする。
具体的にはf(x)は以下のように考えて計算できる。
便宜上A1が正方形の左上、A2がA1の右にあるとする。
上から奇数段目には[x/2]個(ガウス記号)、偶数段目には[(x-1)/2]個の円が並び、
k段目の円の下端の高さは(√3)(k-2)+2
よって全部で[(x-2+2√3)/√3]段ある。
従って、
f(x) = Σ[k=1,[(x-2+2√3)/√3]]([x/2]((1+(-1)^(k+1))+[(x-1)/2](1+(-1)^k))/2)

以上により、与えられたnに対してf(x)≧nを満たすxの下限が可能な正方形の一辺の下限。

949 名前:132人目の素数さん:2008/12/31(水) 02:00:38
a^2 +b^2 =abc +2 なる整数a, b, cを全て決定せよ。

950 名前:132人目の素数さん:2008/12/31(水) 02:06:10
±1、±1、0

951 名前:132人目の素数さん:2008/12/31(水) 02:14:08
>>950
一瞬だったかorz
どう解きました?

952 名前:132人目の素数さん:2008/12/31(水) 11:58:56
>>951
950ではないが
2|ab|≦a^2 + b^2≦|ab| |c| + 2 より |c|≦2 は自明。

953 名前:952:2008/12/31(水) 12:00:00
ab≠0 のときね

954 名前:132人目の素数さん:2008/12/31(水) 13:42:44
>>952
不等式では評価できてない気が・・・

955 名前:132人目の素数さん:2008/12/31(水) 13:54:13
いやできてると思うが

956 名前:132人目の素数さん:2008/12/31(水) 14:36:11
なんで
2|ab|≦a^2 + b^2≦|ab| |c| + 2 

|c|≦2が自明?

957 名前:132人目の素数さん:2008/12/31(水) 16:33:47
出題者か?
これが分からないようでは(r

958 名前:132人目の素数さん:2008/12/31(水) 18:10:55
そういう解説をせねばならないようなものは自明とは言わない。

959 名前:132人目の素数さん:2008/12/31(水) 18:37:21
まあどうでもいいや。 次の問題。

960 名前:132人目の素数さん:2008/12/31(水) 18:50:41
満を辞して出題した問題が秒殺でファビョったか。

961 名前:132人目の素数さん:2008/12/31(水) 18:57:00
2|ab|≦a^2 + b^2≦|ab| |c| + 2
は|c|≧2でaとbが充分近ければ必ず成立するのに
なんで|c|≧2が自明?

この式だけから|c|≧2なんて出てこないが。

962 名前:132人目の素数さん:2008/12/31(水) 21:22:53
つりはもういいよ

963 名前:132人目の素数さん:2008/12/31(水) 22:37:10
ってか真面目に>>949わからん
不等式だと>>961が言うように解けないのだが

964 名前:132人目の素数さん:2008/12/31(水) 23:40:09
952は|c|≧2ではなく|c|≦2と言っているようだが?

いずれにせよなぜ自明なのかはわからんけど


965 名前:132人目の素数さん:2009/01/01(木) 00:08:32
5/6<sin 1<101/120 を示せ。
ただし単位はラジアン。

966 名前:132人目の素数さん:2009/01/01(木) 02:40:55
>>965
f(x)=sinx
g(x)=x-(x^3)/6
h(x)=g(x)+(x^5)/120
の大小関係について微分を使って調べればよい

967 名前: ◆BhMath2chk :2009/01/01(木) 20:00:01
文字は全て整数。
a^2+b^2=abc+2,c≠0となるa,b,cが存在するとすると
a^2+b^2=abc+2>2,|a|^2+|b|^2=|a||b||c|+2なので
a^2+b^2=abc+2,0<a,0<b,0<cとなるa,b,cが存在する。
d=min{a|∃b∃c(a^2+b^2=abc+2,0<a,0<b,0<c)}とすると
d^2+e^2=def+2,d≦e,0<fとなるe,fが存在する。
d^2+(df−e)^2=d(df−e)f+2。
d≦df−e。
d+e≦df。
de+e^2≦def=d^2+e^2−2。
0≦d(e−d)≦−2。
よってa^2+b^2=abc+2,c≠0となるa,b,cは存在しない。


968 名前:132人目の素数さん:2009/01/01(木) 23:52:12
>>967
d^2+(df−e)^2=d(df−e)f+2。
d≦df−e。

の部分がよく分かりません。

969 名前: ◆BhMath2chk :2009/01/02(金) 01:00:00
a^2+b^2=abc+2,c≠0ならば0<abcなので0<d(df−e)f。
0<d,0<fなので0<df−e。
a=df−e,b=e,c=fとすると
a^2+b^2=abc+2,0<a,0<b,0<cで
このaの最小値がdなのでd≦df−e。


970 名前:949:2009/01/02(金) 02:58:57
>>967 >>969
合ってそうです。

ちなみに一般のk>0に対して
a^2 +b^2 =abc +k
の解で(cは定数と考えて)a^2 +b^2が最小となるものを(a, b) [|a|≧|b|]とすると
( (b^2 -k)/a, b ) もまた一解なので、|b|>√kのとき
a^2 +b^2≦{(b^2 -k)/a}^2 +b^2 すなわち|a|<|b|となり矛盾します。
故に |b|≦√k がいえます。

971 名前:132人目の素数さん:2009/01/02(金) 11:36:15
>>970
そのやり方だと a^2 +b^2 の最小値の存在について言及する必要があると思う。
a^2 + b^2=abc + k において b、c、k が決まれば a は(高々2個)決まるので、
|b^2 -k|/|a|=|a| としては駄目だろうか?


972 名前:132人目の素数さん:2009/01/02(金) 16:34:46
>>971
>a^2 + b^2=abc + k において b、c、k が決まれば a は(高々2個)決まるので、
>|b^2 -k|/|a|=|a| としては駄目だろうか?
aの2解の絶対値が必ずしも等しいとは限らない気がする。

あと>>970で、a^2 +b^2 の最小値が存在しないとすると結局方程式に1解も存在しないことになるので
最小値の存在は仮定していいのでは?厳密な議論が必要だったら指摘してくれ

973 名前:132人目の素数さん:2009/01/02(金) 18:19:37
>>979
a=3,b=2,c=2,k=1 のとき a^2 +b^2 =abc +k , |a|≧|b|
は満たすが,|b|≦√k は満たさない.

( (b^2 -k)/a, b ) の組が |a|≧|b| を満たしているとして話を進めているのが
原因かな.

974 名前:973:2009/01/02(金) 18:29:12
>>973はちょっと勘違い.
仕切り直しをして
a^2 +b^2が最小の組[|a|≧|b|]の|b|を評価しても意味ないのでは?

975 名前:132人目の素数さん:2009/01/03(土) 01:34:31
>>974
a^2 +b^2 =abc +k でb, kが定まれば
a|b^2-k よりaの解は有限個であり、そして当然cの解も有限である。
ここでcは定数と考えていたから、cのとりうる値はそれに限るといえる。

例えば>>949だったら
a^2 +b^2が最小の組[|a|≧|b|]について|b|≦√2すなわちb=±1, 0 なので
[@]b=±1のとき: a|1よりa=±1なので、a^2 +b^2 =2となりc=0を得る。
[A]b=0のとき: a^2 =2 となるがこのようなaは存在しない
以上よりc=0

976 名前:132人目の素数さん:2009/01/03(土) 01:44:53
ごめんなさい>>970を少し訂正します。
|b|>√kのとき矛盾を導くとありますが、もっと強く|b|≧√kで矛盾を導きます。
従って|b|<√kが言えるのでa|b^2-k(≠0) よりaの解は有限個となります。

977 名前:132人目の素数さん:2009/01/03(土) 01:58:37
ああ、あとkは完全平方数ではないと仮定してください
kが完全平方数の時b=0ではcの解が有限個には定まらないっぽい
連投スマソ

978 名前:132人目の素数さん:2009/01/03(土) 21:54:39
>>968

 0 ≧ df-e
の場合もあり得るが、0<d, 0<f なので
 d^2 + (df-e)^2 = d(df-e)f + 2 ≦ 2,
 このとき f=0 となって矛盾。        >>950
・・・なので
 0 < df-e,
としてよい。このとき dの定義より
 d ≦ df-e.

979 名前:132人目の素数さん:2009/01/04(日) 15:32:04
>>975
>ここでcは定数と考えていたから、cのとりうる値はそれに限るといえる。
の部分はおかしいだろ。

980 名前:132人目の素数さん:2009/01/05(月) 08:10:01
面白い問題おしえて〜な 十五問目
http://science6.2ch.net/test/read.cgi/math/1231110000/


981 名前:132人目の素数さん:2009/01/05(月) 19:51:00
乙と梅

982 名前:132人目の素数さん:2009/01/05(月) 21:53:24
二百四十八日。


983 名前:132人目の素数さん:2009/01/06(火) 00:04:16
では正解は>>967のみと言うことで。

984 名前:132人目の素数さん:2009/01/06(火) 00:35:38
>>979
得られた値の集合をSとすれば
c∈Z\Sのとき(a, b)は不能
従ってc∈S

985 名前:132人目の素数さん:2009/01/06(火) 21:53:23
二百四十九日。


986 名前:132人目の素数さん:2009/01/07(水) 18:47:55
連 立 方 程 式

987 名前:132人目の素数さん:2009/01/07(水) 21:53:23
二百五十日。


988 名前:132人目の素数さん:2009/01/07(水) 23:17:57
>>984
意味不明

989 名前:132人目の素数さん:2009/01/08(木) 01:09:17
>>988
Sってのは>>975で得られたcの解の集合な。

990 名前:132人目の素数さん:2009/01/08(木) 13:07:36
誰も見てない上げ

991 名前:132人目の素数さん:2009/01/08(木) 20:27:59


992 名前:132人目の素数さん:2009/01/08(木) 21:53:24
二百五十一日。


993 名前:132人目の素数さん:2009/01/08(木) 23:41:28
このスレを見ている人はこんなスレも見ています。(ver 0.20)
世界名作劇場〜総合スレ〜part27 [懐アニ昭和]

994 名前:132人目の素数さん:2009/01/09(金) 11:47:07
生め

995 名前:132人目の素数さん:2009/01/09(金) 15:36:35


996 名前:132人目の素数さん:2009/01/09(金) 15:41:32


997 名前:132人目の素数さん:2009/01/09(金) 15:42:02


998 名前:132人目の素数さん:2009/01/09(金) 15:42:34


999 名前:132人目の素数さん:2009/01/09(金) 15:43:04


1000 名前:132人目の素数さん:2009/01/09(金) 15:43:35
平面上に半径の等しい4つの円がある。
この4つの円は互いに重なることなく自由に動くものとする。
このとき、4つの異なる円周によって作られる領域の最大値を求めなさい。

1001 名前:1001:Over 1000 Thread
このスレッドは1000を超えました。
もう書けないので、新しいスレッドを立ててくださいです。。。


戻る 全部 最新50
DAT2HTML 0.35d Converted.